Sei sulla pagina 1di 261

YouMath.

it

Pij88u9k

INTEGRALI
ESERCIZI SVOLTI

YouMath.it
YouMath.it

Pij88u9k

Autori: F. Sbranchella, S. Zungri, G. Carichino


Copyright © 2016 Math Industries Srl. Tutti i diritti riservati.
È vietata qualsiasi forma di riproduzione, ridistribuzione, o trasmissione in ogni forma o mediante qualsiasi mezzo, incluse la
fotocopiatura, la copiatura digitale, o qualsiasi altro mezzo elettronico o meccanico, senza il permesso scritto dell'editore, ad
eccezione di brevi citazioni ad esclusivo uso di recensione e/o critica.

Authors: F. Sbranchella, S. Zungri, G. Carichino


Copyright © 2016 Math Industries Srl. All rights reserved.
No part of this publication may be reproduced, distributed, or transmitted in any form or by any means, including photocopying,
recording, or other electronic or mechanical methods, without the prior written permission of the publisher, except in the case of brief
quotations embodied in critical reviews.

Acquistato su YouMath.it da Navdeep Singh - navdeep99@outlook.it - 146.241.52.118 - 2020-01-04 22:31:14


YouMath.it

INDICE

ESERCIZI

E.1) Esercizi sugli integrali indefiniti elementari

E.2) Esercizi sugli integrali definiti elementari

E.3) Esercizi sugli integrali per parti

E.4) Esercizi sugli integrali di funzioni razionali

E.5) Esercizi sugli integrali razionali con delta negativo

E.6) Esercizi sugli integrali per sostituzione

E.7) Esercizi sugli integrali di funzioni trigonometriche

E.8) Esercizi sugli integrali di funzioni fratte

E.9) Esercizi sugli integrali di funzioni particolari

E.10) Esercizi di riepilogo sugli integrali indefiniti

E.11) Esercizi di riepilogo sugli integrali definiti

E.12) Esercizi sulla media integrale

E.13) Esercizi sull'area con gli integrali


Pij88u9k
E.14) Esercizi sul volume di solidi di rotazione con gli integrali

E.15) Esercizi sul calcolo degli integrali impropri

E.16) Esercizi sulla convergenza degli integrali impropri di prima specie

E.17) Esercizi sulla convergenza degli integrali impropri di seconda specie

E.18) Esercizi di riepilogo sulla convergenza degli integrali impropri

E.19) Esercizi sulla convergenza degli integrali impropri parametrici

E.20) Esercizi sulle funzioni integrali

E.21) Esercizi sullo studio della funzione integrale

3
eBook edito da YouMath.it acquistato da Navdeep Singh
YouMath.it

ESERCIZI SUGLI INTEGRALI INDEFINITI ELEMENTARI


La richiesta di ciascuno dei seguenti esercizi prevede di individuare la famiglia di primitive di ciascuna delle funzioni integrande.

I)

II)

III)

IV)

V)

VI)

VII)

VIII)
Pij88u9k

IX)

X)

XI)

XII)

XIII)

XIV)

XV)

XVI)

XVII)

4
Navdeep Singh - navdeep99@outlook.it - 146.241.52.118 - 2020-01-04 22:31:14 r1t2e3m4u5k cikelpo
YouMath.it

XVIII)

Soluzioni

Nota: indica una costante additiva arbitraria, che individua tutte e solo le funzioni della famiglia di primitive.

I)

II)

III)

IV)

V)

VI)

VII)

VIII)

IX)

X) Pij88u9k

XI)

XII)

XIII)

XIV)

XV)

XVI)

XVII)

XVIII)

ESERCIZI SUGLI INTEGRALI DEFINITI ELEMENTARI


Il secondo blocco di esercizi sugli integrali elementari riguarda gli integrali definiti e richiede un passaggio in più: dopo aver
determinato una primitiva per ciascuna delle funzioni integrande, è necessario calcolare la differenza delle valutazioni delle
primitive agli estremi. Si tratta in buona sostanza di applicare il teorema fondamentale del calcolo integrale.

5
eBook edito da YouMath.it acquistato da Navdeep Singh
YouMath.it

I)

II)

III)

IV)

V)

VI)

VII)

VIII)

IX)
Pij88u9k

X)

XI)

XII)

XIII)

Soluzioni

I)

II)

III)

IV)

V)

VI)

6
Navdeep Singh - navdeep99@outlook.it - 146.241.52.118 - 2020-01-04 22:31:14 r1t2e3m4u5k cikelpo
YouMath.it

VII)

VIII)

IX)

X)

XI)

XII) Non si può calcolare: si confronti il dominio dell'integranda con l'insieme di integrazione.

XIII)

ESERCIZI SUGLI INTEGRALI PER PARTI


Esercizio 1. Calcolare il seguente integrale indefinito con il metodo di integrazione per parti

Svolgimento: l'integrale
Pij88u9k

si risolve con il metodo di integrazione per parti. Osserviamo che l'integranda è prodotto di due funzioni che sono rispettivamente la
funzione identità e la funzione logaritmica .

Per utilizzare la formula di integrazione per parti

dobbiamo intuire chi svolge il ruolo di fattore finitoe chi il ruolo di fattore differenziale . Una buona regola per
comprendere in che modo scegliere consiste nel ragionare come segue: deve essere una funzione facile da
derivare, mentre deve essere facile da integrare.

Nel caso che stiamo analizzando la funzione facile da integrare tra è e infatti possiede una
primitiva nota. Come fattore finito, facile da derivare, prendiamo per esclusione .

Calcoliamo la derivata del logaritmo e l'integrale di x

e sostituiamo ordinatamente le funzioni ottenute nella formula di integrazione per parti

7
eBook edito da YouMath.it acquistato da Navdeep Singh
YouMath.it

è un integrale immediato pertanto la precedente espressione diventa

Non sottovalutiamo l'importanza della costante additiva che variando nell'insieme dei numeri reali individua l'insieme delle
primitive dell'integranda ossia l'integrale indefinito e l'esercizio è concluso.

Esercizio 2. Calcolare l'integrale

con il metodo di integrazione per parti.

Svolgimento: l'esercizio chiede di determinare il seguente integrale indefinito

Pij88u9k

Notiamo subito che l'integranda è il prodotto di due funzioni: la funzione identità e la funzione esponenziale e
quando compare un prodotto, l'integrazione per parti si candida subito come metodo risolutivo.

La formula è

Naturalmente dobbiamo scegliere chi è la funzione facile da derivare, ossia , e la funzione facile da integrare, . Sia la
funzione esponenziale che la funzione identità sono ottimi candidati per diventare , in tal caso però è opportuno scegliere
.

Osserviamo infatti che

e grazie alla formula di integrazione per parti

dove è una costante reale.

Esercizio 3. Calcolare il seguente integrale

8
Navdeep Singh - navdeep99@outlook.it - 146.241.52.118 - 2020-01-04 22:31:14 r1t2e3m4u5k cikelpo
YouMath.it

Svolgimento: per affrontare l'integrale proposto, possiamo utilizzare le proprietà di additività e di omogeneità degli integrali indefiniti

Il secondo integrale è immediato e infatti si trova nella tabella degli integrali fondamentali

dove è una costante additiva arbitraria.

Il primo invece va proprio calcolato per parti, prendendo (per dirla in modo grezzo ma efficace) come primitiva e come
derivata.

Osserviamo che quindi e di conseguenza

e da qui al risultato il passo è breve

Pij88u9k
dove è una costante additiva arbitraria.

Ricomponiamo il tutto in così da poter scrivere

dove è una costante additiva arbitraria, pertanto

e ciò conclude l'esercizio.

Esercizio 4. Calcolare l'integrale indefinito

Svolgimento: l'integrale proposto può essere risolto con il metodo di integrazione per parti, a patto di utilizzare un barbatrucco che
consiste nel vedere l'integranda come prodotto tra la funzione costante e la funzione , alla fine non stiamo
modificando la traccia, è l'elemento neutro rispetto alla moltiplicazione.

A questo punto scegliamo come fattore finito (facile da derivare) e come fattore differenziale (facile da integrare)
. Deriviamo con la regola di derivazione delle funzioni composte e determiniamo una primitiva di

9
eBook edito da YouMath.it acquistato da Navdeep Singh
YouMath.it

Facciamo intervenire la regola di integrazione per parti

e sostituendo in modo opportuno i pezzi

Ora, non ci resta che semplificare il semplificabile e scrivere

Per portare a termine l'esercizio dobbiamo risolvere l'integrale del logaritmo che si calcola ancora una volta per parti e riciclando il
barbatrucco precedente

Pij88u9k
Ora che abbiamo a disposizione l'integrale del logaritmo possiamo ricomporre il risultato dell'integrale di partenza

dove è una costante additiva al variare della quale vengono individuate tutte e le sole primitive di .

Esercizio 5. Calcolare il seguente integrale indefinito

Svolgimento: l'integrale

si risolve tramite la regola di integrazione per parti

dove è la funzione facile da derivare e è la funzione facile da integrare.

10
Navdeep Singh - navdeep99@outlook.it - 146.241.52.118 - 2020-01-04 22:31:14 r1t2e3m4u5k cikelpo
YouMath.it

In questo esercizio è lampante capire quale sia la funzione facile da derivare e quale quella facile da integrare

e in base alla regola di integrazione per parti scopriamo che

Un piccolo trucco algebrico ci permette di scrivere in modo differente

e sostituendo nell'integrale avremo

Attenzione, conviene spezzare l'integranda in modo furbo così da semplificarne l'espressione

Pij88u9k

Osserviamo che una primitiva di , in accordo con la tabella degli integrali fondamentali, è , quindi il primo integrale nelle

parentesi è immediato.

Anche l'integrale

è immediato perché, se osserviamo bene l'integranda, il numeratore è quasi la derivata del denominatore, mancherebbe solo un .
L'idea qui è moltiplicare e dividere per così da costruirci al numeratore esattamente la derivata del denominatore.

Possiamo scrivere

Riprendiamo l'integrale di partenza da dove lo abbiamo lasciato

11
eBook edito da YouMath.it acquistato da Navdeep Singh
TmF2ZGVlcCBTaW5naCAtIG5hdmRlZXA5OUBvdXRsb29rLml0IC0gMTQ2LjI0MS41Mi4xMTggLSAyMDIwLTAxLTA0IDIyOjMxOjE0

YouMath.it

dove è una costante reale, dunque

e ciò conclude l'esercizio.

Esercizio 6. Calcolare il seguente integrale indefinito

Svolgimento: per svolgere l'esercizio, bisogna procedere con la formula di integrazione per parti

Dobbiamo ovviamente scegliere in modo opportuno chi svolge il ruolo di e chi il ruolo di , ma come si fa, visto che la
funzione integranda è ? Interviene un barbatrucco che consiste nel vedere l'integranda come un prodotto:
.

Prendiamo come funzione facile da derivare e come funzione facile da integrare , dunque

Pij88u9k

Grazie alla formula di integrazione per parti possiamo scrivere

A questo punto interessandoci solamente dell'integrale che resta, osservando che il numeratore è esattamente la derivata del
denominatore a meno di una costante moltiplicativa: moltiplichiamo e dividiamo per

e utilizziamo la regola di integrazione fondamentale

Osserviamo che il valore assoluto è superfluo perché è positiva indipendentemente dal valore di dunque possiamo
asserire che

12
Navdeep Singh - navdeep99@outlook.it - 146.241.52.118 - 2020-01-04 22:31:14 r1t2e3m4u5k cikelpo
YouMath.it
con costante reale. Abbiamo concluso l'esercizio.

Esercizio 7. Determinare l'insieme delle primitive della funzione

Svolgimento: non facciamoci ingannare dalla traccia, l'esercizio ci sta chiedendo di determinare l'integrale indefinito della funzione
ossia

si risolve con il metodo di integrazione per parti, ossia utilizzando la formula

dove è il fattore finito, facile da derivare, mentre è il fattore differenziale, facile da integrare. In questo caso è
conveniente scegliere la funzione logaritmica come fattore finito, e la funzione costante come fattore differenziale.

Osserviamo che la derivata di si ottiene utilizzando la regola di derivazione della funzione composta.

Utilizziamo la formula di integrazione per parti


Pij88u9k

e al numeratore dell'integrale rimasto sommiamo e sottraiamo , dopodiché eseguiamo dei passaggi algebrici il cui compito è
quello di semplificare i conti

Ci rimane solo un integrale da risolvere e può essere affrontato riconducendoci all'integrale notevole/p>

13
eBook edito da YouMath.it acquistato da Navdeep Singh
YouMath.it
Affinché l'integrale si possa scrivere in questa forma, mettiamo in evidenza il al denominatore, dopodiché facciamo intervenire le
proprietà delle potenze

Sottolineiamo che nell'ultimo passaggio abbiamo espresso e in un secondo momento abbiamo usato la proprietà del
quoziente di due potenze che hanno lo stesso esponente. Non ci siamo ancora ricondotti all'integrale fondamentale, manca la

derivata di al numeratore dell'integranda: poco male, è sufficiente moltiplicare e dividere per

Finalmente possiamo scrivere il risultato

e l'esercizio
Pij88u9kè concluso.

Esercizio 8. Calcolare l'integrale indefinito

Svolgimento: l'integrale

è un esercizio classico sull'integrazione per parti. Si risolve quindi con la formula

dove e dunque:

Grazie alla formula di integrazione per parti l'integrale di partenza diventa

14
Navdeep Singh - navdeep99@outlook.it - 146.241.52.118 - 2020-01-04 22:31:14 r1t2e3m4u5k cikelpo
YouMath.it

Procediamo nuovamente per parti ponendo

così che possiamo scrivere

come

Osserviamo che l'integrale al primo membro è identico all'integrale al secondo membro, possiamo pensare quindi di portare
quest'ultimo al primo e sommare algebricamente i due risultati

Da qui è facile giungere alla conclusione, è sufficiente dividere membro a membro per .
Pij88u9k

dove è una costante additiva e abbiamo portato a termine l'esercizio.

Esercizio 9. Calcolare l'integrale della funzione utilizzando il metodo di integrazione per parti.

Svolgimento: a conti fatti

è un classico integrale irrazionale con sostituzione trigonometrica, ma la traccia è categorica: chiede di calcolarlo con il metodo di
integrazione per parti. Adeguiamoci e procediamo per parti, prendendo come fattore finito (da derivare)

mentre

Per la formula di integrazione per parti

15
eBook edito da YouMath.it acquistato da Navdeep Singh
YouMath.it

l'integrale diventa

Aggiungiamo e sottraiamo al numeratore della funzione integranda

e spezziamo la frazione così da poter scrivere l'integrale come somma di integrali

Eseguiamo una razionalizzazione al contrario moltiplicando e dividendo l'integranda centrale per

Abbiamo scoperto che

Pij88u9k

ossia siamo di fronte ad un integrale ricorsivo che può essere affrontato con le formule di riduzione per gli integrali.

In sostanza, portiamo al primo membro

e dunque

Dalla tabella degli integrali fondamentali sappiamo che

quindi l'integrale precedente diventa

A questo punto non ci rimane altro che dividere membro a membro per

16
Navdeep Singh - navdeep99@outlook.it - 146.241.52.118 - 2020-01-04 22:31:14 r1t2e3m4u5k cikelpo
YouMath.it

con costante reale. L'esercizio può ritenersi concluso.

Esercizio 10. Utilizzando il metodo di integrazione per parti, calcolare l'integrale indefinito

Svolgimento: nel calcolo dell'integrale

riscrivere il seno mediante la formula di bisezione non è un buon modo di procedere: il metodo più conveniente prevede di
osservare che, applicando la formula di integrazione per parti e prendendo come derivata, la cui primitiva (a meno di

costanti additive) è , si ottiene

Stesso ragionamento con l'integrale

Pij88u9k

Ricomponiamo il tutto

Poniamo e trattiamolo come un'incognita

Manca la costante additiva per individuare l'intera famiglia di primitive

Possiamo asserire che

17
eBook edito da YouMath.it acquistato da Navdeep Singh
YouMath.it

e l'esercizio è concluso.

Esercizio 11. Calcolare l'integrale indefinito

Svolgimento: grazie alle proprietà delle potenze, l'integrale

può essere riscritto come

Di fronte ad un integrale di questo tipo, il metodo di integrazione per parti è perfetto per giungere al risultato. Poniamo quindi

La formula
Pij88u9kdi integrazione per parti

ci assicura quindi la seguente uguaglianza

Per risolvere l'integrale è sufficiente trasportare fuori dal simbolo di integrazione la costante moltiplicativa ed eseguire il prodotto

usando le proprietà delle potenze

Per l'ultimo integrale è sufficiente usare la formula per l'integrale di una potenza

Riscriviamo in forma normale le potenze con esponente negativo

L'integrale di partenza è quindi

18
Navdeep Singh - navdeep99@outlook.it - 146.241.52.118 - 2020-01-04 22:31:14 r1t2e3m4u5k cikelpo
YouMath.it

e l'esercizio è terminato.

Esercizio 12. Determinare la famiglia di primitive della funzione

Svolgimento: in soldoni, l'esercizio chiede di calcolare l'integrale indefinito

e possiamo farlo mediante la formula di integrazione per parti

Scegliamo come funzione facile da derivare e come funzione facile da integrare

e applicando la formula di integrazione per parti otteniamo

Pij88u9k

L'integrale rimasto non è immediato, però ci viene in soccorso un barbatrucco algebrico che renderà più agevole il calcolo:
sommiamo e sottraiamo al numeratore dell'integranda

Razionalizziamo moltiplicando e dividendo per l'integranda del primo integrale

Perfetto, dopo questi semplici passaggi possiamo asserire che

19
eBook edito da YouMath.it acquistato da Navdeep Singh
YouMath.it

Ora portiamo al primo membro , stando attenti a cambiare segno, e sommiamo algebricamente gli integrali al

primo membro

Ci rimane da calcolare l'integrale

il quale richiede un po' di trucchi algebrici, il primo dei quali consiste nel raccogliere all'interno della radice e in seguito utilizzare le
proprietà delle potenze

Pij88u9k
Sostituiamo nell'integrale così da ottenere

A questo punto è tutta discesa, dividiamo per membro a membro e otteniamo la famiglia di primitive della funzione
ossia:

dove è una costante reale e l'esercizio è terminato.

Esercizio 13. Calcolare il seguente integrale

Svolgimento: grazie alla formula di duplicazione del seno

possiamo asserire che e dunque l'integrale proposto dall'esercizio diventa

20
Navdeep Singh - navdeep99@outlook.it - 146.241.52.118 - 2020-01-04 22:31:14 r1t2e3m4u5k cikelpo
YouMath.it

A questo punto possiamo trasportare fuori dal simbolo di integrazione la costante moltiplicativa

Utilizziamo la formula di integrazione per parti

dove

Grazie ad essa possiamo scrivere

dove è una costante reale. Possiamo asserire che

Pij88u9k

e ciò conclude l'esercizio.

Esercizio 14. Calcolare il seguente integrale indefinito con il metodo di integrazione per parti

Svolgimento: grazie alle proprietà delle potenze possiamo scrivere l'integranda in modo tale da agevolare l'integrazione per parti

Integriamo per parti prendendo come funzione facile da integrare. Una primitiva di questa funzione è

La funzione facile da derivare è invece la cui derivata è ovviamente .

A questo punto è sufficiente utilizzare la formula di integrazione per parti ossia

Sostituiamo nella formula appena scritta le funzioni

21
eBook edito da YouMath.it acquistato da Navdeep Singh
YouMath.it

Non resta che calcolare l'integrale

dove basta osservare che l'integranda è proprio la derivata della cotangente cambiata di segno. Ricomponiamo il tutto e scriviamo il
risultato

L'esercizio è concluso.

Esercizio 15. Calcolare il seguente integrale indefinito

Svolgimento: l'integrale

va risolto con la formula di integrazione per parti, ossia

Pij88u9k

dove è il fattore finito (facile da derivare) mentre è il fattore differenziale (facile da integrare). Scegliamo come la
funzione perché, una volta derivata, il suo esponente diminuirà di un'unità, di conseguenza ed è facilissimo
determinare una sua primitiva

A questo punto facciamo intervenire la formula di integrazione per parti

L'integrale al secondo membro si risolve nuovamente per parti, scegliendo come fattore finito e come fattore differenziale

Abbiamo praticamente concluso perché l'integrale rimasto è immediato

22
Navdeep Singh - navdeep99@outlook.it - 146.241.52.118 - 2020-01-04 22:31:14 r1t2e3m4u5k cikelpo
YouMath.it
dove è una costante reale. In definitiva l'integrale di partenza è

e l'esercizio finisce qui.

Integrali definiti per parti

Gli ultimi esercizi riguardano il calcolo degli integrali definiti.

Esercizio 16. Calcolare l'integrale definito che va da della funzione

Svolgimento: l'esercizio chiede di calcolare l'integrale definito

dove la funzione è definita a tratti, più precisamente

Pij88u9k
Poiché la funzione è definita a tratti è necessario spezzare l'integrale in base all'intervallo di variazione della variabile .

Calcoliamo separatamente i due integrali al secondo membro iniziando dal primo:

Il secondo integrale, ossia

si risolve per parti scegliendo come fattore finito e come fattore differenziale , pertanto

Per la formula di integrazione per parti

23
eBook edito da YouMath.it acquistato da Navdeep Singh
YouMath.it

Abbiamo calcolato entrambi gli integrali definiti, e dunque l'integrale di partenza è

e abbiamo concluso.

Esercizio 17. Calcolare il seguente integrale definito con integranda data dal prodotto tra un termine esponenziale ed il seno:

Svolgimento: quello proposto è uno di quegli integrali che vanno risolti con la formula di integrazione per parti, impostando la
risoluzione in modo da ottenere un'equazione in cui l'incognita è data proprio dall'integrale.

Tale approccio è tipico degli integrali in cui è dato il prodotto di funzioni con derivate cicliche. Lavoriamo con l'integrale indefinito

Pij88u9k
e applichiamo la formula di integrazione per parti considerando il termine esponenziale come derivata, la cui primitiva è data

semplicemente da

A parte calcoliamo

integrando nuovamente per parti

Chiamiamo l'ultimo integrale e ricomponiamo . Torna tutto, perché quello che abbiamo ottenuto è un integrale identico a quello
di partenza:

24
Navdeep Singh - navdeep99@outlook.it - 146.241.52.118 - 2020-01-04 22:31:14 r1t2e3m4u5k cikelpo
YouMath.it
Trattando l'uguaglianza come un'equazione arriviamo a

Ora possiamo pure occuparci dell'integrale definito

ed è tutto!

ESERCIZI SUGLI INTEGRALI DI FUNZIONI RAZIONALI


Esercizio 1. Calcolare l'integrale della seguente funzione razionale con il metodo dei fratti semplici

Svolgimento: l'integrale
Pij88u9k

si risolve con il metodo dei fratti semplici. Il nostro intento è quello di determinare due costanti reali tale che

Determiniamo il minimo comune multiplo al secondo membro

da cui, semplificando i denominatori e raccogliendo secondo le potenze di otteniamo l'uguaglianza

Il principio di identità dei polinomi ci permette di costruire il sistema lineare

che ha per soluzione . A questo punto sostituiamo i valori di in così da ottenere

Grazie al metodo dei fratti semplici, abbiamo espresso la funzione integranda come somma di frazioni facili da integrare, pertanto
sussiste la seguente uguaglianza

25
eBook edito da YouMath.it acquistato da Navdeep Singh
YouMath.it

Il primo integrale è immediato, è infatti un integrale presente nella tabella degli integrali fondamentali

Il secondo integrale

può essere ricondotto all'integrale fondamentale

infatti possiamo osservare che il numeratore è quasi la derivata del denominatore, ci serve solo il segno meno: poco male,
possiamo usare il solito barbatrucco algebrico, moltiplicando e dividendo per .

Pij88u9k

Non ci rimane che ricomporre il risultato dell'integrale di partenza

dove è una costante reale e l'esercizio è concluso.

Esercizio 2. Calcolare il seguente integrale indefinito

Svolgimento: l'integrale indefinito

si calcola con il metodo dei fratti semplici. Il trinomio al denominatore può essere fattorizzato con la regola del trinomio con somma e
prodotto

dunque l'integrale di partenza si riscrive come

Il nostro obiettivo ora è quello di esprimere la funzione integranda come somma di frazioni più semplici da integrare, e in base al
metodo dei fratti semplici esistono due costanti reali tali che
26
Navdeep Singh - navdeep99@outlook.it - 146.241.52.118 - 2020-01-04 22:31:14 r1t2e3m4u5k cikelpo
metodo dei fratti semplici esistono due costanti reali tali che YouMath.it

A questo punto portiamo tutto a denominatore comune

Semplifichiamo i denominatori e raccogliamo secondo le potenze di al secondo membro

Il principio di identità dei polinomi ci assicura che il polinomio al primo membro è identico al polinomio al secondo membro se e solo
se i coefficienti dei termini dello stesso grado sono uguali, pertanto affinché valga l'identità deve essere soddisfatto il seguente
sistema lineare

In definitiva, possiamo riscrivere l'integrale di partenza come

Pij88u9k

Osservazione: nell'ultimo passaggio abbiamo fatto intervenire una delle proprietà dei logaritmi così da esprimere la soluzione in
forma più compatta, ad ogni modo non è da considerarsi un passaggio obbligatorio.

In conclusione, scriviamo

e l'esercizio è concluso.

Esercizio 3. Determinare la famiglia delle primitive della funzione razionale

Svolgimento: l'esercizio ci sta chiedendo di determinare la famiglia delle primitive della funzione

e per definizione di integrale indefinito, ciò equivale a calcolare il seguente integrale:

Il metodo di integrazione adatto ad affrontare questo tipo di integrali è quello dei fratti semplici: consiste nell'esprimere l'integranda
come somma di frazioni facili da integrare. Per prima cosa osserviamo che il denominatore è già scomposto in fattori

27
eBook edito da YouMath.it acquistato da Navdeep Singh
YouMath.it
irriducibili che sono .

Al fattore associamo il fratto semplice , mentre al fattore associamo la seguente somma:

Il nostro obiettivo è quello di determinare le costanti di modo che

Portiamo tutto a denominatore comune

I denominatori non servono più perché uguali membro a membro, pertanto otteniamo la seguente uguaglianza tra polinomi

Sviluppiamo i prodotti e raccogliamo secondo le potenze di

Grazie al principio di identità dei polinomi possiamo costruire il sistema lineare


Pij88u9k

che ha per soluzioni e dunque la funzione integranda si esprime come segue:

In sostanza abbiamo espresso la funzione integranda come somma di frazioni di cui è facile calcolare l'integrale. Procediamo
pertanto con il calcolo dell'integrale

Decomponiamo l'integrale della somma come somma di integrali

Il primo e il secondo integrale sono praticamente immediati

mentre

28
Navdeep Singh - navdeep99@outlook.it - 146.241.52.118 - 2020-01-04 22:31:14 r1t2e3m4u5k cikelpo
YouMath.it

è un integrale della forma

Riprendiamo il calcolo dell'integrale da così da poter concludere che

dove è una costante reale. Per mettere un punto all'esercizio scriviamo per bene il risultato

e finalmente l'esercizio è concluso.

Esercizio 4. Calcolare

con il metodo dei fratti semplici.

Svolgimento:
Pij88u9k come richiesto dalla traccia, utilizziamo il metodo dei fratti semplici per risolvere il seguente integrale:

Il primo passo consiste nello scomporre in fattori irriducibili il denominatore dell'integranda che nella fattispecie è una differenza di
cubi

Il secondo passo consiste nell'associare a ciascun fattore della scomposizione il fratto semplice: al fattore associamo

mentre al fattore associamo il fratto semplice

Ora siamo giunti al terzo passo: dobbiamo determinare le costanti reali tali che

Calcoliamo il minimo comune multiplo ed eseguiamo i prodotti

29
eBook edito da YouMath.it acquistato da Navdeep Singh
YouMath.it

I denominatori si possono semplificare perché sono uguali membro a membro, in questo modo otteniamo la seguente uguaglianza

Al secondo membro raccogliamo secondo le potenze di

e facciamo intervenire il principio di identità dei polinomi, il quale ci permette di costruire il sistema lineare con cui calcoleremo i
valori di :

Risolvendo il sistema otterremo i valori che stavamo cercando

di conseguenza

Pij88u9k

e l'integrale di partenza si riscrive come

Per risolvere l'integrale

utilizziamo dei trucchi algebrici così da costruirci la derivata del denominatore al numeratore. Moltiplichiamo e dividiamo per

e scriviamo

Ora spezziamo l'integrale come somma di integrali, ma in modo furbo:

30
Navdeep Singh - navdeep99@outlook.it - 146.241.52.118 - 2020-01-04 22:31:14 r1t2e3m4u5k cikelpo
YouMath.it

Osserviamo che

perché questo integrale è nella forma , mentre il secondo integrale deve essere ricondotto

all'integrale notevole

mediante l'uso di barbatrucchi algebrici, il primo dei quali consiste nel completamento del quadrato di binomio

Pij88u9k

Mettiamo in evidenza e grazie alle proprietà delle potenze possiamo scrivere

Osserviamo che la derivata della base del quadrato è

Moltiplichiamo e dividiamo in modo furbo l'integranda

31
eBook edito da YouMath.it acquistato da Navdeep Singh
YouMath.it

e svolgendo alcune semplificazioni algebriche otteniamo

Adesso ricomponiamo il tutto stando particolarmente attenti alle costanti moltiplicative che abbiamo tralasciato

dove è una costante reale. L'esercizio è concluso.

Pij88u9k
Esercizio 5. Utilizzare il metodo dei fratti semplici per calcolare l'integrale indefinito

Svolgimento: per calcolare l'integrale

si può procedere con il metodo di integrazione delle funzioni razionali, o metodo dei fratti semplici. Dopo aver scomposto il
denominatore con il raccoglimento parziale

si cerca una scomposizione della forma

calcoliamo il denominatore comune e raccogliamo i coefficienti dei termini in

32
Navdeep Singh - navdeep99@outlook.it - 146.241.52.118 - 2020-01-04 22:31:14 r1t2e3m4u5k cikelpo
YouMath.it
dopodiché applichiamo il principio di identità dei polinomi e confrontiamo il primo e l'ultimo termine della catena di uguaglianze: per
trovare la decomposizione richiesta vogliamo

e risolvendo il sistema si trova come unica soluzione .

Morale: possiamo riscrivere l'integrale di partenza nella forma

Il primo lo riscriviamo sfruttando la linearità dell'integrale di Riemann

e a questo punto integrare è semplice

dove è una costante reale. Scriviamo il risultato dell'integrale di partenza

e, finalmente,
Pij88u9k mettiamo un punto a questo esercizio.

Esercizio 6. Calcolare il seguente integrale con il metodo dei fratti semplici

Svolgimento: per innescare il metodo dei fratti semplici dobbiamo innanzitutto scomporre in fattori irriducibili il denominatore
dell'integranda, osservando che è una differenza di quadrati

A ciascun fattore possiamo associare un fratto semplice, in particolare al fattore:

Il prossimo passo consiste nel determinare le costanti in modo che sussista la seguente identità:

33
eBook edito da YouMath.it acquistato da Navdeep Singh
YouMath.it

Portiamo tutti a denominatore comune

Il denominatore non serve più perché uguale membro a membro. Effettuiamo i vari prodotti e raccogliamo secondo le potenze di

Grazie al principio di identità dei polinomi possiamo costruire il sistema lineare

e risolvendolo conduce all'unica soluzione

I valori ottenuti permettono di esprimere la funzione integranda come somma di fratti semplici

e l'integrale di partenza diventa

Pij88u9k

Decomponiamo l'integrale della somma come somma di integrali

dove è una costante reale. Scriviamo per bene il risultato

Perfetto! L'esercizio è concluso.

Esercizio 7. Calcolare il seguente integrale indefinito

con il metodo dei fratti semplici.

Svolgimento: per risolvere l'integrale

34
Navdeep Singh - navdeep99@outlook.it - 146.241.52.118 - 2020-01-04 22:31:14 r1t2e3m4u5k cikelpo
YouMath.it

mediante il metodo dei fratti semplici dobbiamo innanzitutto scomporre il polinomio al denominatore dell'integranda

Osserviamo che sono radici del polinomio di molteplicità . Il nostro intento è quindi quello di esprimere la funzione
integranda come

con costanti da determinare. Portiamo tutto a denominatore comune

Eseguiamo le moltiplicazioni e raccogliamo secondo le potenze di

Il principio di identità dei polinomi ci permette di costruire il seguente sistema lineare

Pij88u9k
Risolviamolo così da ottenere le costanti

quindi la funzione integranda si scrive come segue:

di conseguenza l'integrale di partenza diventa

e sfruttando la linearità dell'operatore integrale scriveremo

Indichiamo con rispettivamente il primo, secondo, terzo e quarto integrale

35
eBook edito da YouMath.it acquistato da Navdeep Singh
YouMath.it

L'integrale di partenza è uguale alla somma di , pertanto possiamo scrivere

dove è una costante reale. L'esercizio è concluso.

Esercizio 8. Calcolare il seguente integrale con integranda data da una funzione razionale

Pij88u9k
Svolgimento: siamo di fronte ad un integrale di una funzione razionale con numeratore di grado maggiore al grado del
denominatore .

Eseguiamo allora la divisione polinomiale tra numeratore e denominatore così da determinare il quoziente e il resto della divisione

pertanto

Ora, sfruttando le proprietà di linearità e additività dell'integrale abbiamo

mentre

Ricorriamo al metodo dei fratti semplici, così da esprimere l'integranda come

36
Navdeep Singh - navdeep99@outlook.it - 146.241.52.118 - 2020-01-04 22:31:14 r1t2e3m4u5k cikelpo
YouMath.it

Per il principio di identità dei polinomi deve essere

da cui , pertanto rimpiazzando i valori trovati in otteniamo

e grazie alla tabella degli integrali notevoli

Mettiamo insieme i vari pezzi e concludiamo che

dove la costante ingloba tutte le costanti.

Esercizio 9. Calcolare il seguente integrale indefinito


Pij88u9k

Svolgimento: nel caso dell'integrale

abbiamo un denominatore che è potenza di un binomio irriducibile, e si considera la seguente scomposizione della funzione
integranda

Portiamo tutto a denominatore comune

I denominatori sono uguali membro a membro quindi possono essere semplificati. Eseguendo le moltiplicazioni e raccogliendo
secondo le potenze di , ciò che rimane è la seguente uguaglianza

Il principio di identità dei polinomi permette di costruire il seguente sistema lineare

37
eBook edito da YouMath.it acquistato da Navdeep Singh
YouMath.it

da cui .

Possiamo riscrivere l'integrale nella seguente forma

e a questo punto non è difficile integrare: per prima cosa spezziamo l'integrale come somma di più integrali

Calcoliamo i tre integrali separatamente: il primo è immediato

il secondo richiede un paio di arrangiamenti algebrici

Pij88u9k

Per quanto riguarda il terzo integrale è sufficiente osservare che può essere riscritto nella forma

Grazie alle proprietà delle potenze possiamo scrivere

A questo punto ricostruiamo la derivata di che è banalmente :

Ecco fatto. Ricomponiamo il risultato dell'integrale di partenza

38
Navdeep Singh - navdeep99@outlook.it - 146.241.52.118 - 2020-01-04 22:31:14 r1t2e3m4u5k cikelpo
YouMath.it
dove è una costante reale. L'esercizio è terminato.

Esercizio 10. Calcolare il seguente integrale indefinito

Svolgimento: procediamo con la risoluzione dell'integrale

seguendo la via del barbatrucco algebrico e il primo passo consiste nello spezzare l'integrale nella somma

Risolviamo separatamente i due integrali partendo dal primo

raccogliamo un termine a numeratore

Pij88u9k

Spezziamo anche questo integrale

ossia

Il primo integrale è semplice, e il secondo anche, ma a patto di aggiustare le costanti moltiplicative

e ponendo possiamo scrivere il risultato parziale nella forma più compatta

Dedichiamoci ora al secondo integrale della somma scritta inizialmente:

39
eBook edito da YouMath.it acquistato da Navdeep Singh
YouMath.it

Raccogliamo un

aggiungiamo e sottraiamo, a numeratore, il termine

e spezziamo l'integrale come somma di integrali

ossia

Pij88u9k
e quindi

Non resta che mettere tutto assieme e scrivere il risultato dell'integrale di partenza

avendo posto . Possiamo finalmente asserire che l'esercizio è concluso.

Esercizio 11. Calcolare l'integrale indefinito della funzione razionale fratta

Svolgimento: l'esercizio chiede di svolgere

e poiché il numeratore ha grado maggiore di quello del denominatore, possiamo tranquillamente procedere con la divisione
polinomiale tra il polinomio al numeratore e il polinomio al denominatore, così da determinare il quoziente e il resto: questa è la
strada standard.

D'altro canto possiamo far uso di trucchi algebrici per bypassare la divisione: aggiungiamo e sottraiamo al numeratore.

40
Navdeep Singh - navdeep99@outlook.it - 146.241.52.118 - 2020-01-04 22:31:14 r1t2e3m4u5k cikelpo
YouMath.it

Spezziamo la frazione in modo furbo e di conseguenza spezziamo l'integrale come somma di integrali

Nel primo integrale sommiamo e sottraiamo mentre nel secondo mettiamo in evidenza

Il secondo integrale è praticamente svolto, il primo invece deve essere manipolato e spezzato ulteriormente

Pij88u9k

Gli integrali ottenuti sono facilmente risolvibili

dove è una costante reale. Riscriviamo per bene il risultato

e finalmente l'esercizio è concluso.

Esercizio 12. Calcolare l'integrale indefinito della seguente funzione razionale fratta

Svolgimento: detto in altri termini, l'esercizio chiede di calcolare l'integrale indefinito

ossia l'integrale di una funzione razionale fratta con numeratore

41
eBook edito da YouMath.it acquistato da Navdeep Singh
YouMath.it
di grado maggiore al grado del denominatore

Eseguiamo dunque la divisione polinomiale tra numeratore e denominatore così da ottenere il quoziente e il resto, rispettivamente:

pertanto

Sfruttiamo le proprietà di linearità e additività dell'integrale

e avendo ben presenti gli integrali notevoli, scriviamo

Perfetto, adesso non ci rimane altro che scrivere per bene il risultato

Pij88u9k

con costante reale, e concludere l'esercizio.

Esercizio 13. Determinare l'insieme delle primitive della funzione razionale fratta

Svolgimento: l'insieme delle primitive di una funzione è per definizione l'integrale indefinito della funzione stessa, pertanto l'esercizio
sta chiedendo di calcolare il seguente integrale

L'integranda è una funzione razionale fratta, in cui il grado del numeratore coincide con il grado del denominatore. In questi casi
possiamo procedere in due modi differenti: o eseguiamo la divisione polinomiale tra il numeratore e il denominatore, oppure
utilizziamo dei barbatrucchi algebrici.

Procediamo nel secondo modo, in particolare sommiamo e sottraiamo così da ottenere:

Spezziamo la frazione in modo furbo

e semplifichiamo

42
Navdeep Singh - navdeep99@outlook.it - 146.241.52.118 - 2020-01-04 22:31:14 r1t2e3m4u5k cikelpo
YouMath.it

Usiamo la linearità dell'operatore integrale (l'integrale della somma è la somma degli integrali)

Il primo integrale è immediato, così come il secondo, dunque

In definitiva

dove è una costante reale, è l'insieme delle primitive della funzione data e l'esercizio è concluso.

Integrali definiti di funzioni razionali

Esercizio 14. Calcolare il seguente integrale definito

Pij88u9k

Svolgimento: per affrontare l'integrale definito

utilizziamo il metodo dei fratti semplici. Per prima cosa scomponiamo in fattori irriducibili il polinomio al denominatore

dopodiché andiamo alla ricerca di due costanti reali che permettono di esprimere l'integranda come somma di frazioni più
semplici, più precisamente:

Consideriamo l'uguaglianza tra il primo e l'ultimo membro

e semplifichiamo i denominatori ottenendo quella che deve essere un'identità polinomiale

43
eBook edito da YouMath.it acquistato da Navdeep Singh
YouMath.it

Il principio di identità dei polinomi permette di costruire il sistema lineare

da cui otteniamo facilmente le due costanti: .

Morale della favola, possiamo scrivere l'integrale di partenza come

A questo punto è sufficiente valutare le differenze tra i valori che le funzioni assumono agli estremi, più precisamente dobbiamo
calcolare la differenza tra la funzione valutata all'estremo superiore e la stessa funzione valutata all'estremo inferiore

In definitiva
Pij88u9k

e abbiamo finito!

Esercizio 15. Calcolare l'integrale definito

Svolgimento: il metodo dei fratti semplici si presta bene per il calcolo dell'integrale

dunque procediamo con la scomposizione in fattori irriducibili del denominatore

Bene! Il nostro obiettivo diventa quello di determinare due costanti reali tali che la funzione integranda si scriva come
somma di frazioni più semplici, ossia

ora scriviamo il membro di destra come un'unica frazione

44
Navdeep Singh - navdeep99@outlook.it - 146.241.52.118 - 2020-01-04 22:31:14 r1t2e3m4u5k cikelpo
YouMath.it

e raccogliamo secondo le potenze di

Dobbiamo dunque confrontare il primo e l'ultimo membro della catena di uguaglianze

e l'applicazione del principio di identità dei polinomi permette di costruire il sistema lineare

che risolto conduce alla soluzione , per cui possiamo riscrivere l'integrale come segue

e quindi per le proprietà degli integrali

Abbiamo quasi terminato: i due integrali che restano sono semplici da calcolare, dacché le integrande hanno primitive logaritmiche

Pij88u9k

ossia

Ora l'esercizio può considerarsi concluso.

Esercizio 16. Calcolare l'integrale definito

Svolgimento: l'integrale proposto può essere affrontato con qualche magheggio algebrico. L'idea è quella di spezzare la frazione
nella somma di due frazioni. Grazie alla linearità dell'integrale definito possiamo scrivere l'integrale proposto come segue

Calcoliamo gli integrali indefiniti associati: il secondo integrale ha evidentemente come primitiva l'arcotangente di (a meno di
costanti moltiplicative):

45
eBook edito da YouMath.it acquistato da Navdeep Singh
YouMath.it

Il primo integrale invece ha una primitiva logaritmica

L'integrale definito vale dunque:

Scriviamo per bene il risultato

e finalmente l'esercizio è concluso.

Esercizio 17. Calcolare l'integrale definito

Svolgimento: per risolvere l'integrale

Pij88u9k
ricorriamo ad un trucco algebrico che consiste nel sottrarre e sommare al numeratore

A questo punto spezziamo la frazione e successivamente scriviamo l'integrale della somma come somma di integrali, tutto questo
grazie alle proprietà dell'integrale definito:

Osserviamo che è a tutti gli effetti una differenza di quadrati, e dunque si scompone come segue:

Sostituiamo nell'integrale

e semplifichiamo

46
Navdeep Singh - navdeep99@outlook.it - 146.241.52.118 - 2020-01-04 22:31:14 r1t2e3m4u5k cikelpo
YouMath.it

Risolviamo con tutta calma il primo integrale, che è praticamente immediato

Dedichiamoci alla risoluzione del secondo integrale

Ora possiamo calcolare l'integrale definito dato dalla traccia: è sufficiente sommare tra loro i contributi dei due integrali

e l'esercizio
Pij88u9kè concluso!

ESERCIZI SUGLI INTEGRALI RAZIONALI CON DELTA


NEGATIVO
Esercizio 1. Calcolare l'integrale indefinito

Svolgimento: per risolvere quel tipo di integrali si può procedere in molti modi differenti, il più veloce consiste riportarsi all'integrale
con primitiva un'arcotangente.

Iniziamo mettendo in evidenza il al denominatore

e per le proprietà delle potenze possiamo esprimere il precedente integrale come segue:

A questo punto osserviamo che se al numeratore ci fosse la derivata del termine , cioè , ci saremmo ricondotti all'integrale

notevole

47
eBook edito da YouMath.it acquistato da Navdeep Singh
YouMath.it

ma non disperiamo, è sufficiente moltiplicare e dividere per l'integrale che stiamo risolvendo:

Semplifichiamo in modo furbo

e portiamo fuori utilizzando le proprietà dell'integrale

Grazie a questi passaggi algebrici ci siamo ricondotti all'integrale notevole, dunque:

Pij88u9k

Possiamo quindi concludere che

con costante reale. L'esercizio è concluso.

Esercizio 2. Calcolare il seguente integrale razionale fratto

Svolgimento: osserviamo che l'integranda è una funzione razionale fratta in cui il denominatore ha discriminante minore di zero. Nel
caso di integrali con delta negativo a denominatore, il modo di procedere prevede di scrivere il denominatore come somma di un
quadrato perfetto e di una costante, in questo modo ci ricondurremo agevolmente ad una funzione la cui primitiva è un'arcotangente.

Per esprimere il denominatore come una somma di quadrati, è sufficiente completare il quadrato procedendo come segue:

quindi l'integrale proposto diventa

Osserviamo ora al numeratore abbiamo la derivata del termine , di conseguenza siamo di fronte ad un integrale del tipo

48
Navdeep Singh - navdeep99@outlook.it - 146.241.52.118 - 2020-01-04 22:31:14 r1t2e3m4u5k cikelpo
YouMath.it

Nel nostro caso, se prendiamo abbiamo e quindi

L'esercizio è concluso.

Esercizio 3. Calcolare l'integrale della seguente funzione razionale fratta

Svolgimento: dobbiamo calcolare l'integrale

osservando dapprima che il discriminante del polinomio al denominatore è negativo, e dunque il polinomio è irriducibile. In questi
casi l'idea consiste nel ricondursi ad un integrale di una funzione la cui primitiva è un'arcotangente e per farlo sarà necessario
completare il quadrato:

in questo modo l'integrale di partenza si riscrive come


Pij88u9k

Ora, raccogliamo al denominatore

e sfruttiamo le proprietà delle potenze

Aggiustiamo un po' la base della potenza, scrivendo in forma normale la frazione di frazioni

Se al numeratore avessimo la derivata di , ossia , saremmo di fronte ad un integrale che ha come risultato

un'arcotangente: poco male, è sufficiente moltiplicare e dividere per il fattore di cui abbiamo bisogno.

49
eBook edito da YouMath.it acquistato da Navdeep Singh
YouMath.it

Non ci rimane altro che semplificare le frazioni e concludere che

dove è una costante reale. Possiamo mettere un punto all'esercizio.

Esercizio 4. Calcolare l'integrale indefinito

Svolgimento: prima di buttarci a capofitto nel calcolo dell'integrale

è necessario effettuare alcune premesse. Per prima cosa, l'integranda è una funzione razionale fratta con numeratore e
denominatore dello stesso grado. In più il denominatore è irriducibile come si deduce dal calcolo del discriminante.

Il metodo per affrontare questa tipologia di integrali consiste nell'effettuare la divisione polinomiale tra numeratore e denominatore
Pij88u9k
che ci permette di esprimere l'integranda nella forma

Per conseguire lo scopo, ossia calcolare l'integrale di partenza, abbiamo bisogno:

1. di una frazione con numeratore che coincide con la derivata del denominatore;

2. di una frazione con numeratore costante.

Concentriamoci per un momento sull'addendo

e su di esso utilizziamo dei semplici trucchi algebrici, nella fattispecie facciamo comparire il coefficiente a numeratore

sommiamo e sottraiamo inoltre un a numeratore

e infine spezziamo la frazione

A questo punto si tratta di calcolare


50
Navdeep Singh - navdeep99@outlook.it - 146.241.52.118 - 2020-01-04 22:31:14 r1t2e3m4u5k cikelpo
YouMath.it

Osserviamo che nell'argomento del logaritmo il modulo non serve perché il polinomio ha delta negativo e il
coefficiente di è positivo, di conseguenza il polinomio in questione è positivo su tutto l'asse reale.

Passiamo a

proponendoci come obiettivo quello di ricondurre l'integranda alla derivata di un'arcotangente, per cui vogliamo a denominatore la
somma tra e un quadrato, ossia . Dobbiamo lavorarci su, aiutandoci con il metodo del completamento del quadrato

Raccogliamo

Il coefficiente esterno lo si può portare fuori dall'integrale, cosicché siamo giunti alla forma richiesta
Pij88u9k

dove .

Ora non bisogna fare altro che far comparire all'interno dell'integrale

la costante moltiplicativa , in accordo con il teorema di derivazione della funzione composta. Otteniamo

Finalmente abbiamo a disposizione i risultati che ci servono per risolvere l'integrale di partenza

51
eBook edito da YouMath.it acquistato da Navdeep Singh
YouMath.it

dove è una costante reale. L'esercizio è concluso.

Esercizio 5. Determinare la famiglia delle primitive della funzione razionale

Svolgimento: in altri termini l'esercizio chiede di calcolare l'integrale indefinito

dove l'integranda è una funzione razionale fratta con al denominatore il polinomio che ha delta negativo ed è
dunque irriducibile in .

Il trucco per risolvere questa tipologia di integrali è far sì che al numeratore appaia la derivata del denominatore. In questo caso, la
derivata del denominatore è , conseguentemente moltiplichiamo e dividiamo per l'integranda così da ottenere

Sommiamo e sottraiamo al numeratore e spezziamo la frazione in modo che faciliti il nostro compito
Pij88u9k

Calcoliamo separatamente i due integrali cominciando dal primo. Esso è un integrale immediato nella forma

dunque

Il secondo integrale

è più delicato da trattare: proponiamoci come obiettivo quello di ricondurlo all'integrale notevole

52
Navdeep Singh - navdeep99@outlook.it - 146.241.52.118 - 2020-01-04 22:31:14 r1t2e3m4u5k cikelpo
YouMath.it

Per fare ciò completiamo il quadrato al denominatore

e sostituiamolo in

È fatta, è sufficiente ricomporre il risultato

dove è una costante reale. Finito.

Esercizio 6. Risolvere il seguente integrale razionale fratto

Svolgimento: consideriamo l'integranda

Pij88u9k
e osserviamo che è un polinomio irriducibile in , infatti è un polinomio di secondo grado con discriminante associato
negativo. In più, il grado del numeratore è inferiore al grado del denominatore.

In questo caso l'integrale

è praticamente immediato se ci si accorge che il numeratore è quasi la derivata del denominatore: manca solo un al numeratore.
Per farlo apparire, moltiplichiamo e dividiamo per

A questo punto, l'integrale si presenta nella forma

dunque possiamo scrivere che

Osserviamo che quindi il valore assoluto è superfluo, dunque

dove è una costante reale e l'esercizio è concluso.

53
eBook edito da YouMath.it acquistato da Navdeep Singh
YouMath.it

ESERCIZI SUGLI INTEGRALI PER SOSTITUZIONE


Esercizio 1. Calcolare il seguente integrale con il metodo di sostituzione

Svolgimento: osserviamo che l'integrale proposto può essere ricondotto ad un integrale notevole, ma la traccia parla chiaro:
dobbiamo risolverlo per sostituzione. Prima di procedere, però, utilizziamo le proprietà lineari dell'integrale indefinito per trasportare
fuori dal simbolo di integrale la costante moltiplicativa

inoltre grazie alle proprietà delle potenze possiamo scrivere

A questo punto possiamo innescare il metodo di sostituzione ponendo

Sostituiamo nell'integrale di partenza così che diventi

Pij88u9k
Questo è un integrale fondamentale che ha come risultato un'arcotangente

Perfetto! Non ci resta che ripristinare la variabile tenendo a mente la sostituzione fatta, ossia

Scriviamo per bene il risultato

e mettiamo un punto all'esercizio.

Esercizio 2. Risolvere il seguente integrale indefinito con il metodo di sostituzione

Svolgimento: possiamo risolvere

effettuando la seguente sostituzione

54
Navdeep Singh - navdeep99@outlook.it - 146.241.52.118 - 2020-01-04 22:31:14 r1t2e3m4u5k cikelpo
YouMath.it

dove nell'ultimo passaggio abbiamo diviso per membro a membro così da esprimere come .

Ok, grazie alla sostituzione proposta l'integrale diventa

ma sappiamo che , pertanto l'integrale diventa

Osserviamo che il valore assoluto non è necessario perché il termine è positivo in quanto somma di quantità positive.

In definitiva

dove è una costante reale. Fine!

Esercizio 3. Risolvere il seguente integrale indefinito con il metodo di sostituzione

Pij88u9k

Svolgimento: osserviamo che l'integrale proposto può essere risolto riconducendolo all'integrale fondamentale in forma generale

ma la traccia parla chiaro, dobbiamo risolvere

con il metodo di sostituzione. Grazie alle proprietà delle potenze possiamo scrivere

e a questo punto è chiaro qual è la sostituzione che ci permetterà di giungere al risultato. Poniamo

di conseguenza l'integrale diventa praticamente un integrale fondamentale

55
eBook edito da YouMath.it acquistato da Navdeep Singh
YouMath.it

Poiché abbiamo posto allora l'integrale risulta essere

Scriviamo per bene il risultato

e l'integrale è risolto.

Esercizio 4. Utilizzare il metodo di sostituzione per risolvere il seguente integrale

Svolgimento: per calcolare

possiamo procedere per sostituzione ponendo

così che l'integrale di partenza diventi a tutti gli effetti un integrale di una funzione razionale fratta

Pij88u9k

Senza indugi, procediamo nuovamente per sostituzione ponendo da cui e dunque

Osserviamo che ci troviamo di fronte ad un integrale fondamentale, infatti l'integranda è proprio la derivata dell'arcotangente

Adesso sostituiamo a ritroso, dobbiamo infatti ritornare nella variabile , partendo dalla sostituzione scopriamo che

e ricordando che possiamo concludere che

e l'esercizio è concluso.

Esercizio 5. Calcolare il seguente integrale indefinito

56
Navdeep Singh - navdeep99@outlook.it - 146.241.52.118 - 2020-01-04 22:31:14 r1t2e3m4u5k cikelpo
YouMath.it

Svolgimento: il metodo di sostituzione si presta veramente bene per la risoluzione dell'integrale

ed è evidente che la sostituzione che fa al caso nostro è da cui e, grazie ad essa, l'integrale diventa

Spezziamo la frazione e utilizziamo le proprietà lineari dell'integrale indefinito

Osserviamo ora che quelli ottenuti sono integrali immediati: possiamo scriverne subito il risultato

ma Pij88u9k di conseguenza

dove è una costante reale. Finito.

Esercizio 6. Determinare la famiglia di primitive della funzione irrazionale fratta

Svolgimento: in parole povere l'esercizio chiede di calcolare l'integrale

che, raccogliendo diventa

Ora effettuiamo un cambio di variabile

determiniamo la sostituzione inversa elevando membro a membro al quadrato

57
eBook edito da YouMath.it acquistato da Navdeep Singh
YouMath.it
e infine calcoliamo il differenziale

Sostituiamo nell'integrale

Grazie alla linearità dell'integrale possiamo trasportare fuori dal simbolo di integrazione la costante moltiplicativa e, semplificando
, otteniamo:

L'integranda ora è proprio la derivata dell'arcotangente, e in accordo con la tabelle degli integrali notevoli

Dato che possiamo concludere che

dove è una costante reale.

Esercizio 7. Risolvere il seguente integrale indefinito mediante il metodo di sostituzione

Pij88u9k

Svolgimento: l'integrale proposto ha in effetti un'aria molto minacciosa ma con la sostituzione appropriata esso si risolve in un batter
d'occhio, o quasi. Per calcolare

procediamo con la sostituzione e da questa troviamo la sostituzione inversa, ossia , a cui associamo il
differenziale . L'integrale diventa

Ci siamo ricondotti all'integrale di una funzione razionale fratta il cui numeratore ha grado maggiore di quello del denominatore e in
questi casi, sappiamo già che è necessario effettuare una divisione polinomiale tra numeratore e denominatore, così da ottenere

Sfruttiamo la linearità dell'integrale così da esprimere l'integrale della somma come somma di integrali

Tutt'e tre sono integrali fondamentali, di cui conosciamo già il risultato

58
Navdeep Singh - navdeep99@outlook.it - 146.241.52.118 - 2020-01-04 22:31:14 r1t2e3m4u5k cikelpo
YouMath.it

Non ci rimane che ripristinare la variabile , tenendo a mente l'imposizione fatta, ossia e scrivere

Ok, riportiamo per bene il risultato dell'integrale di partenza

Osserviamo che nel risultato finale non appare il valore assoluto nell'argomento del logaritmo perché è una quantità
positiva. L'esercizio è concluso.

Esercizio 8. Risolvere il seguente integrale indefinito mediante il metodo di sostituzione

Svolgimento: come suggerito dalla traccia, procederemo con il metodo di sostituzione per risolvere

Pij88u9k

Poniamo da cui elevando membro a membro al quadrato otteniamo la sostituzione inversa, ossia e pertanto il
differenziale è . L'integrale diventa

Addizioniamo e sottraiamo al numeratore ed otteniamo

che spezziamo nei due integrali di cui sappiamo già il risultato

59
eBook edito da YouMath.it acquistato da Navdeep Singh
YouMath.it

Ripristiniamo la variabile , tenendo a mente la sostituzione effettuata, ossia

In definitiva

dove è una costante reale. In chiusura dell'esercizio, osserviamo che in questo caso il valore assoluto nel logaritmo è superfluo
perché è una somma di quantità positive, ed è dunque positiva.

Esercizio 9. Calcolare il seguente integrale indefinito con il metodo di sostituzione

Svolgimento: la sostituzione adatta a risolvere velocemente

Pij88u9k

è , da cui elevando al quadrato membro a membro si ottiene e conseguentemente il differenziale è .


Grazie alla sostituzione l'integrale diventa

e dopo le opportune semplificazioni

Usiamo un barbatrucco algebrico: sommiamo e sottraiamo per

e a questo punto spezziamo la frazione in modo furbo

60
Navdeep Singh - navdeep99@outlook.it - 146.241.52.118 - 2020-01-04 22:31:14 r1t2e3m4u5k cikelpo
YouMath.it

Utilizziamo la linearità dell'integrale: scriviamolo come somma di integrali

Ora fattorizziamo la differenza di quadrati

Nel secondo integrale semplifichiamo e contemporaneamente osserviamo che il primo è in realtà un integrale immediato

Eseguiamo il prodotto e ripristiniamo la variabile ricordando la sostituzione fatta

Osserviamo che il valore assoluto non compare nel risultato finale perché la quantità è positiva.

Pij88u9k in bellezza l'esercizio scrivendo per bene il risultato


Concludiamo

con costante reale. Fine.

Esercizio 10. Risolvere il seguente integrale mediante il metodo di sostituzione

Svolgimento: l'obiettivo è quello di calcolare il seguente integrale usando il metodo di sostituzione... ma prima è necessario
aggiustare un po' l'integranda così da semplificarci il lavoro. Il trucco consiste nell'esprimere come il prodotto tra , ma
perché lo facciamo? Semplicemente perché così facendo, apparirà nell'integrale il nuovo differenziale.

Poniamo così che da cui , non ci rimane che sostituire così da ottenere

Dividiamo termine a termine e applichiamo le proprietà degli integrali

61
eBook edito da YouMath.it acquistato da Navdeep Singh
YouMath.it

Grazie alle proprietà dei radicali la somma algebrica dei due integrali diventa

Le funzioni integrande non sono altro che potenze, dunque utilizzando la relativa regola di integrazione otteniamo

Semplifichiamo le frazioni e riportiamo le potenze con esponente razionale fratto sotto forma di radici

Pij88u9k
Attenzione, pertanto la precedente espressione diventa

In definitiva concludiamo che

Finalmente l'esercizio è completamente svolto.

Esercizio 11. Risolvere

mediante il metodo di sostituzione.

Svolgimento: prima di procedere con il metodo di sostituzione, effettuiamo alcune manipolazioni algebriche per rendere i calcoli
successivi più semplici. Cominciamo con il raccogliere al numeratore dell'integranda, in questo modo l'integrale diventa

Esprimiamo sotto forma di potenza con esponente razionale

62
Navdeep Singh - navdeep99@outlook.it - 146.241.52.118 - 2020-01-04 22:31:14 r1t2e3m4u5k cikelpo
YouMath.it

e grazie alle proprietà delle potenze aventi la stessa base otteniamo

Per agevolare la sostituzione scriviamo come

Ora possiamo procedere per sostituzione ponendo da cui segue che e di conseguenza il differenziale

diventa . Rimpiazziamo i vari termini nell'integrale così da ottenere

Sfruttiamo la linearità dell'integrale scrivendo l'integrale della somma come somma di integrali

Scriviamo le radici sotto forma di potenze con esponente razionale e facciamo intervenire le proprietà delle radici

Pij88u9k

Quelli ottenuti sono integrali immediati, di cui conosciamo già il risultato

Manca poco per giungere al risultato: dobbiamo semplicemente ripristinare la variabile tenendo a mente che
pertanto

In definitiva, concludiamo l'esercizio scrivendo per bene il risultato:

con costante reale.

63
eBook edito da YouMath.it acquistato da Navdeep Singh
YouMath.it
Esercizio 12. Risolvere il seguente integrale mediante il metodo di sostituzione

Suggerimento: porre .

Svolgimento: calcoliamo

ponendo da cui ricaviamo il differenziale e da cui determiniamo la trasformazione diretta

(Osservazione: l'uguaglianza è consistente se e solo se perché l'immagine della funzione arcoseno è appunto

l'intervallo appena scritto).

in questo modo l'integrale diventa

Pij88u9k

Grazie alla relazione fondamentale della trigonometria possiamo scrivere . Notiamo che il

valore assoluto sparisce perché varia in , intervallo in cui il coseno è non negativo. In definitiva il precedente integrale

diventa

La risoluzione dell'ultimo integrale richiede un po' di Trigonometria, e più precisamente intervengono le formule di duplicazione del
coseno

Sostituiamo nell'integrale che diventa

64
Navdeep Singh - navdeep99@outlook.it - 146.241.52.118 - 2020-01-04 22:31:14 r1t2e3m4u5k cikelpo
YouMath.it

Non ci resta che ritornare nella variabile tenendo a mente che pertanto

A tutti gli effetti questo è il risultato dell'integrale di partenza, ma può essere riscritto anche in modo differente se si conoscono
veramente bene le formule trigonometriche. Osserviamo, infatti, che per le formule di duplicazione del seno

Pij88u9k

Ora poiché l'arcoseno è la funzione inversa del seno si ha che

inoltre dalla relazione fondamentale della Trigonometria, segue che

Possiamo pertanto scrivere

Queste relazioni ci permettono di concludere l'esercizio e scrivere il risultato come segue

65
eBook edito da YouMath.it acquistato da Navdeep Singh
YouMath.it

dove è una costante reale.

Esercizio 13. Calcolare il seguente integrale indefinito con il metodo di sostituzione

Svolgimento: l'integrale non è immediato, ma nemmeno così difficile e per risolverlo è sufficiente azzeccare la giusta sostituzione
che permetta di eliminare tutte le radici. Nell'integrale:

compaiono sia radici quadrate che radici cubiche, che hanno indice rispettivamente e ponendo si ha che:

1. il differenziale diventa ;

2.

e sostituendo nell'integrale otteniamo

Pij88u9k

Ci siamo ricondotti ad un integrale di una funzione razionale in cui il grado del numeratore è maggiore del grado al denominatore. In
questi casi si può procedere tranquillamente con la divisione polinomiale, grazie alla quale possiamo esprimere l'integrale come

e per la linearità dell'integrale

Quelli ottenuti sono sostanzialmente integrali immediati, pertanto

66
Navdeep Singh - navdeep99@outlook.it - 146.241.52.118 - 2020-01-04 22:31:14 r1t2e3m4u5k cikelpo
YouMath.it

Dalla sostituzione segue che e rimpiazzando con nel risultato otteniamo

Semplifichiamo il più possibile ciascun termine mediante l'uso delle proprietà dei radicali

e dunque possiamo concludere che

dove è una costante reale. Finito.

Esercizio 14. Calcolare il seguente integrale con il metodo di sostituzione

Pij88u9k l'integrale
Svolgimento:

si risolve abbastanza agevolmente se utilizziamo la seconda relazione fondamentale della Trigonometria:

Sostituiamo nell'integrale così da ottenere

Ora possiamo procedere per sostituzione: poniamo di conseguenza il differenziale diventa

e dunque .

Risolviamo l'ultimo integrale con la regola di integrazione delle potenze

67
eBook edito da YouMath.it acquistato da Navdeep Singh
YouMath.it

Teniamo a mente che e rimpiazziamo

In definitiva possiamo concludere che

dove è una costante reale. Abbiamo terminato l'esercizio.

Esercizio 15. Determinare la famiglia delle primitive associata alla funzione

Svolgimento: in altri termini dobbiamo calcolare l'integrale indefinito

procedendo con un piccolo trucchetto algebrico che consiste nel riscrivere l'integrale nella forma

Pij88u9k

e sfruttare l'identità fondamentale della Trigonometria grazie alla quale l'integrale diventa

A questo punto integriamo per sostituzione e poniamo , per cui se calcoliamo il differenziale della trasformazione diretta
otteniamo e dunque l'integrale diventa

A questo punto ripristiniamo la variabile tenendo a mente la sostituzione fatta

dove è una costante reale, dunque

e l'esercizio è concluso.

Esercizio 16. Calcolare il seguente integrale tramite il metodo di sostituzione

68
Navdeep Singh - navdeep99@outlook.it - 146.241.52.118 - 2020-01-04 22:31:14 r1t2e3m4u5k cikelpo
YouMath.it
Svolgimento: per calcolare l'integrale

per prima cosa facciamo ricorso all'identità fondamentale della Trigonometria, grazie alla quale possiamo sostituire
.

L'integrale diventa

ora integriamo per sostituzione ponendo e ne calcoliamo il differenziale direttamente:

Otteniamo

e a questo punto l'integrale è pressoché immediato

ed effettuando la sostituzione al contrario:

Pij88u9k

e abbiamo concluso, ma prima di mettere un punto all'esercizio riscriviamo per bene il risultato

Finito.

Esercizio 17. Calcolare il seguente integrale mediante l'uso del metodo di sostituzione

Svolgimento: osserviamo che il metodo di sostituzione non è strettamente necessario, giacché l'integrale

può essere ricondotto ad un integrale immediato. Decidiamo, in ogni caso di integrare per sostituzione, così da soddisfare a pieno la
traccia dell'esercizio.

La sostituzione più utile consiste nel porre , inoltre, almeno in questo caso, non conviene ricavare la funzione inversa
per determinare il differenziale ; con un po' d'occhio conviene di più differenziare direttamente la legge che esprime il
cambiamento di variabile

da cui

69
eBook edito da YouMath.it acquistato da Navdeep Singh
YouMath.it

Ecco perché non conveniva determinare la trasformazione inversa: perché l'integranda contiene già l'espressione del differenziale
nella nuova variabile.

Riscriviamo l'integrale

e dunque

Torniamo nella variabile , tenendo a mente che

Non ci rimane altro che scrivere per bene il risultato

e concludere l'esercizio.

Esercizio 18. Calcolare l'integrale indefinito

Pij88u9k

mediante il metodo di integrazione per sostituzione.

Svolgimento: risolviamo l'integrale

ponendo , dalla quale esprimiamo in funzione di : . Il nuovo differenziale è e l'integrale diventa

Semplifichiamo e trasportiamo fuori dal simbolo di integrale il segno meno

Risolviamo questo integrale con il metodo di integrazione per parti scegliendo come fattore finito, facile da derivare, e
come fattore differenziale, da integrare, la funzione . La formula di integrazione per parti permette di esprimere l'integrale
precedente come:

A questo punto torniamo nella variabile ricordando che :

70
Navdeep Singh - navdeep99@outlook.it - 146.241.52.118 - 2020-01-04 22:31:14 r1t2e3m4u5k cikelpo
YouMath.it

Perfetto, non ci rimane altro che scrivere per bene il risultato

dove è una costante reale additiva.

Esercizio 19. Risolvere il seguente integrale

Svolgimento: l'integrale

si risolve agevolmente con il metodo di sostituzione: ponendo infatti segue che di conseguenza il differenziale

diventa . Rimpiazziamo i termini e scriviamo:

A questo punto raccogliamo al numeratore


Pij88u9k

e semplifichiamolo con il denominatore

Utilizziamo un espediente algebrico: scriviamo il numeratore come

Spezziamo la frazione all'interno dell'integrale in modo da poter semplificarne l'espressione

Sfruttiamo la linearità dell'integrale e scriviamo l'integrale della somma come somma di integrali

Siamo di fronte a due integrali fondamentali, pertanto

71
eBook edito da YouMath.it acquistato da Navdeep Singh
YouMath.it

Ricordando ora che avevamo posto si ha che

Osserviamo che il valore assoluto è superfluo perché è certamente una quantità positiva, dunque

Ora l'esercizio è concluso.

Esercizio 20. Calcolare l'integrale indefinito

Svolgimento: l'integrale indefinito

può essere risolto facilmente con il metodo di sostituzione ponendo da cui segue che e di conseguenza

. Grazie alla sostituzione, l'integrale di partenza diventa quindi

Pij88u9k
Ci siamo ricondotti all'integrale di una funzione razionale in cui il denominatore è già espresso come prodotto di polinomi irriducibili
entrambi di primo grado. A ciascuno di essi associamo un fratto semplice, più precisamente:

Dobbiamo quindi determinare due costanti reali in modo che sia

A secondo membro, dopo qualche semplice conto, abbiamo che

Dovendo essere

per il principio di identità dei polinomi deve valere

72
Navdeep Singh - navdeep99@outlook.it - 146.241.52.118 - 2020-01-04 22:31:14 r1t2e3m4u5k cikelpo
YouMath.it

da cui . Il nostro integrale diventa quindi

e sfruttando la linearità dell'integrale

Ci siamo così ricondotti a due integrali immediati, di cui conosciamo già il risultato:

Ora ripristiniamo la variabile , tenendo a mente che :

Osserviamo che per definizione di logaritmo , inoltre il valore assoluto è superfluo perché è positivo giacché è
somma di funzioni positive.

Pij88u9k
Abbiamo finalmente portato a termine l'esercizio, scriviamo per bene il risultato

Finito.

Esercizio 21. Calcolare l'integrale indefinito

con una opportuna sostituzione trigonometrica.

Svolgimento: per prima cosa portiamo fuori dal simbolo di integrazione la costante , dopodiché utilizziamo la definizione di potenza
per esprimere diversamente l'integranda

e procediamo con la sostituzione trigonometrica da cui si ottiene facilmente che . Il nuovo

differenziale è . Grazie alla sostituzione, l'integrale diventa

73
eBook edito da YouMath.it acquistato da Navdeep Singh
YouMath.it

Adesso un po' di formule trigonometriche. Per definizione di tangente si ha che

e grazie alla relazione fondamentale della goniometria possiamo scrivere che


pertanto l'integrale precedente diventa

Pij88u9k
Ora dobbiamo ripristinare la variabile , ricordando che:

(formule di duplicazione per il seno)

Con le informazioni in nostro possesso possiamo concludere che

con costante reale. L'integrale è finalmente risolto.

ESERCIZI SUGLI INTEGRALI DI FUNZIONI TRIGONOMETRICHE


Esercizio 1. Calcolare il seguente integrale indefinito

Svolgimento: osserviamo che l'integrale in questione può essere considerato un integrale notevole del tipo

74
Navdeep Singh - navdeep99@outlook.it - 146.241.52.118 - 2020-01-04 22:31:14 r1t2e3m4u5k cikelpo
YouMath.it

dove . Usiamo questa formula per concludere immediatamente che

L'integrale è risolto.

Esercizio 2. Risolvere il seguente integrale mediante le sostituzioni parametriche del coseno

Svolgimento: per risolvere l'integrale

utilizziamo due volte il metodo di sostituzione, sostituendo , da cui e quindi

Usiamo le formule parametriche: si pone e si ricava


Pij88u9k

Sostituiamo tutto:

Eseguiamo qualche conto algebrico così da ottenere

Effettuiamo la sostituzione al contrario

ossia

75
eBook edito da YouMath.it acquistato da Navdeep Singh
YouMath.it

Riscriviamo per bene il risultato

dove è una costante reale... e anche questo integrale è andato.

Esercizio 3. Calcolare il seguente integrale indefinito

Svolgimento: l'integrale

si risolve abbastanza agevolmente a patto di utilizzare astutamente le formule trigonometriche. Esprimiamo


e dalla relazione fondamentale della Trigonometria segue che per ogni
: più precisamente

Pij88u9k
A questo punto integriamo per sostituzione ponendo , di conseguenza il nuovo differenziale è
pertanto si ha:

Ripristiniamo la variabile , tenendo a mente che

Non ci resta che scrivere per bene il risultato

e mettere un punto all'esercizio.

Esercizio 4. Risolvere il seguente integrale mediante l'uso delle sostituzioni trigonometriche

76
Navdeep Singh - navdeep99@outlook.it - 146.241.52.118 - 2020-01-04 22:31:14 r1t2e3m4u5k cikelpo
YouMath.it
Svolgimento: la difficoltà di questo esercizio non risiede tanto nei calcoli, quanto nell'usare astutamente le formule di Trigonometria.
La relazione fondamentale della Goniometria assicura che e grazie ad essa l'integrale diventa

Spezziamo la frazione e semplifichiamo in modo opportuno

Utilizziamo la linearità dell'integrale, così da esprimere l'integrale della somma come somma di integrali

Risolviamo separatamente i due integrali, cominciando dal primo ossia

Esso è un integrale immediato che si presenta nella forma

Pij88u9k
dove , pertanto

Dedichiamoci ora al secondo integrale

in cui ancora una volta esprimiamo come somma tra

Spezziamo la frazione e semplifichiamo

77
eBook edito da YouMath.it acquistato da Navdeep Singh
YouMath.it

Osserviamo che entrambi gli integrali si presentano nella forma

di conseguenza

Perfetto, abbiamo gli elementi per scrivere il risultato dell'integrale di partenza

dove è una costante reale. L'esercizio è concluso.

Esercizio 5. Calcolare il seguente integrale indefinito

Pij88u9k

Svolgimento: per risolvere l'integrale

passiamo, per linearità, a calcolare

Risolviamo separatamente i due integrali, cominciando dal primo che può essere affrontato dividendo termine a termine il quadrato
del coseno:

da cui, ancora una volta per linearità

Passiamo un istante a vedere il secondo integrale dei due iniziali ossia

78
Navdeep Singh - navdeep99@outlook.it - 146.241.52.118 - 2020-01-04 22:31:14 r1t2e3m4u5k cikelpo
YouMath.it
Procediamo con l'integrazione per sostituzione ponendo , da cui calcoliamo direttamente il differenziale della
trasformazione, ossia , e sostituiamo

Spezziamo ancora una volta la frazione come somma di frazioni

così come spezziamo l'integrale come somma di integrali

Ripristiniamo la variabile , notando che :

Finalmente abbiamo a disposizione gli elementi per scrivere il risultato dell'integrale di partenza:
Pij88u9k

L'esercizio è finalmente concluso.

ESERCIZI SUGLI INTEGRALI DI FUNZIONI FRATTE


Esercizio 1. Calcolare il seguente integrale indefinito

Svolgimento: l'integrale proposto è a tutti gli effetti un integrale immediato, che con dei semplici pasaggi algebrici si può ricondurre
nella tipologia

Esprimiamo il radicale sotto forma di potenza con esponente razionale

e osserviamo che la derivata della base della potenza è esattamente , dunque possiamo subito concludere che

79
eBook edito da YouMath.it acquistato da Navdeep Singh
YouMath.it

In definitiva

e ciò conclude l'esercizio.

Esercizio 2. Determinare la famiglia delle primitive associata alla funzione

Svolgimento: in altri termini, l'esercizio chiede di calcolare l'integrale indefinito

che può essere espresso come

Osserviamo che è esattamente la derivata dell'arcotangente pertanto si può applicare la formula di integrazione notevole:

Pij88u9k

Nel nostro caso la soluzione è

Chiudiamo l'esercizio scrivendo per bene il risultato

dove è una costante reale. Ecco fatto!

Esercizio 3. Calcolare l'integrale indefinito

Svolgimento: dobbiamo risolvere l'integrale

che possiamo riscrivere come

80
Navdeep Singh - navdeep99@outlook.it - 146.241.52.118 - 2020-01-04 22:31:14 r1t2e3m4u5k cikelpo
YouMath.it
Il metodo che porta più velocemente alla soluzione consiste nell'integrare mediante la sostituzione da cui si evince

che il nuovo differenziale è .

Sostituiamo nell'integrale di partenza

A questo punto abbiamo praticamente il risultato in pugno, è sufficiente ripristinare la variabile , ricordando che
pertanto

L'esercizio è concluso.

Esercizio 4. Risolvere il seguente integrale indefinito

Svolgimento: per una nota proprietà dei logaritmi, l'integrale

può essere riscritto nella forma

Pij88u9k

e quindi, per le proprietà lineari di cui gode l'operatore integrale, vale:

Se integriamo per sostituzione, poniamo e deriviamo direttamente la funzione che definisce il cambiamento di
variabile troviamo che il nuovo differenziale diventa

ossia

e possiamo riscrivere l'integrale nella forma

81
eBook edito da YouMath.it acquistato da Navdeep Singh
YouMath.it

Perfetto, come al solito scriviamo per bene il risultato

dove è una costante reale.

Esercizio 5. Risolvere il seguente integrale indefinito

Svolgimento: grazie alle formule di Trigonometria, ed in particolare alle formule di duplicazione del coseno, possiamo scrivere

Il numeratore può essere ricondotto all'arco doppio sempre per le formule di duplicazione del coseno:

Pij88u9k

Grazie a queste due relazioni, possiamo essere certi della validità della seguente uguaglianza

Spezziamo la frazione e utilizziamo la linearità dell'operatore integrale

Evidentemente il primo integrale è immediato, mentre il secondo merita di essere svolto a parte

82
Navdeep Singh - navdeep99@outlook.it - 146.241.52.118 - 2020-01-04 22:31:14 r1t2e3m4u5k cikelpo
YouMath.it

Moltiplichiamo e dividiamo l'integranda per

e facciamo intervenire la relazione fondamentale della Trigonometria così da esprimere come

Procediamo per sostituzione, ponendo da cui il nuovo differenziale è e dunque

, pertanto l'integrale diventa

Quello che abbiamo ottenuto è un integrale di una funzione razionale fratta che può essere risolta mediante il metodo dei fratti
semplici, osservando preliminarmente che è una differenza di quadrati e in quanto tale si scompone come
.

Al fattore associamo il fratto semplice , e al fattore associamo . Ora il nostro obiettivo è determinare le
Pij88u9k
costanti così che sussista la seguente uguaglianza

Portiamo tutti i termini allo stesso denominatore dopodiché lo semplifichiamo

Raccogliamo secondo le potenze di

e utilizziamo il principio di identità dei polinomi, che ci permette di costruire il seguente sistema lineare

Il sistema fornisce come unica soluzione pertanto

83
eBook edito da YouMath.it acquistato da Navdeep Singh
YouMath.it

e facendo intervenire le proprietà dei logaritmi

Perfetto, ora dobbiamo ripristinare la variabile tenendo a mente la sostituzione effettuata, ossia

Ora, con tutte le attenzioni del caso, ricostruiamo il risultato dell'integrale di partenza

con costante reale.

Osservazione: in realtà è possibile utilizzare le formule di Trigonometria che permettono di scrivere diversamente il risultato, ma non
Pij88u9k
ci sono motivazioni valide che ci spingono a modificarlo, pertanto possiamo considerare l'esercizio concluso.

Esercizio 6. Calcolare il seguente integrale

Svolgimento: l'integrale proposto si risolve mediante il metodo di sostituzione, ponendo da cui pertanto il

nuovo differenziale si scrive come .

Scriviamo in forma normale le frazioni di frazioni, stando attenti ai calcoli

e a questo punto spezziamo la frazione e utilizziamo la linearità dell'operatore integrale

84
Navdeep Singh - navdeep99@outlook.it - 146.241.52.118 - 2020-01-04 22:31:14 r1t2e3m4u5k cikelpo
YouMath.it

Esprimiamo le radici sotto forma di potenza con esponente razionale e utilizziamo le proprietà delle potenze

Bene, ora possiamo fare intervenire la regola di integrazione delle potenze e scrivere

Abbelliamo un po' l'espressione raccogliendo

Pij88u9k
È giunto il momento di ripristinare la variabile , tenendo a mente la sostituzione

Adesso è solo una questione puramente algebrica

Abbiamo terminato, non ci resta che scrivere per bene il risultato

dove è una costante reale.

Esercizio 7. Calcolare il seguente integrale indefinito

Svolgimento: il metodo di sostituzione si presta bene alla risoluzione dell'integrale, ma prima utilizziamo la definizione di cotangente
per esprimere la funzione integranda in soli seni e coseni

85
eBook edito da YouMath.it acquistato da Navdeep Singh
YouMath.it

La sostituzione che fa al caso nostro è da cui e grazie a tali relazioni, riscriviamo l'integrale come
segue:

Procediamo con una nuova sostituzione e poniamo

L'integrale assumerà la seguente forma

Ci siamo ricondotti ad un integrale di una funzione razionale in cui il numeratore ha grado minore rispetto a quello del denominatore
e dunque il metodo dei fratti semplici è quello che si candida per risolvere l'integrale.

Per prima cosa fattorizziamo il denominatore, ossia lo scriviamo come prodotto di polinomi irriducibili
Pij88u9k

A ciascun fattore associamo il fratto semplice:

L'obiettivo ora è determinare le costanti tali che

Portiamo il secondo membro a denominatore comune

I denominatori non servono più perché uguali membro a membro, inoltre è utile sviluppare i prodotti e in seguito raccogliere
secondo le potenze di . Otteniamo dunque:

e grazie al principio di identità dei polinomi possiamo costruire il sistema lineare


86
Navdeep Singh - navdeep99@outlook.it - 146.241.52.118 - 2020-01-04 22:31:14 r1t2e3m4u5k cikelpo
YouMath.it

Risolvendolo scopriamo che le costanti sono

pertanto l'integrale si riscrive come

Ricordiamo che , di conseguenza

infine ripristiniamo la variabile tenendo a mente la sostituzione quindi

Pij88u9k

dove è una costante reale. Non ci resta che scrivere per bene il risultato dell'integrale di partenza

e mettere un punto all'esercizio.

ESERCIZI SUGLI INTEGRALI DI FUNZIONI PARTICOLARI


Gli esercizi svolti che vi proponiamo qui sono molto interessanti, perché riguardano il calcolo di integrali particolari. Cosa significa?
Sono sostanzialmente integrali di funzioni non notevoli, ma che ricorrono spesso negli esercizi delle verifiche scolastiche e degli
esami universitari.

Data l'eterogeneità delle tipologie di integrali proposti, questa raccolta di esercizi è da considerarsi come una scheda di riepilogo,
nel senso che richiede la conoscenza delle varie tecniche di integrazione già incontrate nella risoluzione degli esercizi precedenti.

Esercizio 1. Calcolare l'integrale indefinito

Svolgimento: sebbene sia possibile sviluppare la settima potenza del binomio mediante l'uso del triangolo di Tartaglia, è opportuno
evitare questo passaggio perché molto scomodo dal punto di vista del calcolo. L'approccio giusto consiste invece nel ricondursi

87
eBook edito da YouMath.it acquistato da Navdeep Singh
YouMath.it
all'integrale notevole della potenza in forma generale.

Moltiplicando e dividendo la funzione integranda per

è possibile applicare la formula di integrazione

dove . Osserviamo che abbiamo moltiplicato e diviso per così che apparisse all'interno dell'integranda la
derivata della base . Grazie alla formula di integrazione

L'esercizio è praticamente concluso.

Esercizio 2. Determinare la famiglia delle primitive della funzione

Svolgimento: dobbiamo semplicemente integrare la funzione arcotangente di , ossia risolvere

Pij88u9k

Per integrare un'arcotangente, bisogna procedere con la formula di integrazione per parti prendendo come fattore finito, facile da
derivare e come fattore differenziale . Come conseguenza di ciò, si ha che

e grazie alla formula di integrazione per parti possiamo scrivere

A questo punto interessandoci solamente all'integrale che resta, lo possiamo calcolare scrivendolo nella forma

è sufficiente moltiplicare e dividere per l'integranda

88
Navdeep Singh - navdeep99@outlook.it - 146.241.52.118 - 2020-01-04 22:31:14 r1t2e3m4u5k cikelpo
YouMath.it

Osserviamo che il valore assoluto è superfluo giacché il termine è positivo. Scriviamo per bene il risultato

e mettiamo un punto all'esercizio.

Esercizio 3. Calcolare l'integrale indefinito

Svolgimento: è un integrale che si può ricondurre nella forma

ma c'è un piccolo problema: la funzione integranda è sprovvista della derivata dell'argomento del seno. A questo però c'è un
rimedio, è sufficiente moltiplicare e dividere per , così da ottenere

Pij88u9k

Ecco fatto.

Esercizio 4. Calcolare il seguente integrale indefinito

Svolgimento: attenzione, questo è un integrale immediato, ma purtroppo si tende a dimenticarlo spesso

Osserviamo che l'integranda è proprio la derivata della tangente.

Esercizio 5. Risolvere

Svolgimento: l'integrale

89
eBook edito da YouMath.it acquistato da Navdeep Singh
YouMath.it

ricorre molto spesso nella risoluzione di altri integrali, ecco perché è opportuno imprimere bene nella mente il metodo più veloce per
risolverlo. L'idea è quella di utilizzare le formule di duplicazione del coseno che ci consentono di passare da

all'espressione . Vediamo in dettaglio come fare:

Per le formule di duplicazione del coseno si ha che

e dunque, esprimendo in funzione di otteniamo

Questa relazione è davvero importante perché ci permette di scrivere l'integrale come

Pij88u9k

A questo punto entrambi gli integrali sono immediati da calcolare: per il secondo basta ricordare qual è l'integrale del coseno e
aggiustare in modo opportuno l'integranda moltiplicando e dividendo per

In realtà, si può integrare mediante il metodo di integrazione per parti, ma porta via molto tempo.

Esercizio 6. Calcolare il seguente integrale

Svolgimento: per risolvere l'integrale

procediamo per parti prendendo come fattore finito, da derivare, la funzione e come fattore differenziale, da
integrare, la funzione . Grazie alla formula di integrazione per parti si ha che:

e integrando nuovamente per parti scegliendo come fattore finito e come fattore differenziale si
ha:

90
Navdeep Singh - navdeep99@outlook.it - 146.241.52.118 - 2020-01-04 22:31:14 r1t2e3m4u5k cikelpo
YouMath.it

Se ora chiamiamo l'integrale di partenza , ossia poniamo , allora possiamo scrivere

Trattiamo come un'incognita e l'uguaglianza come un'equazione

Dividendo membro a membro per

In definitiva

dove è una costante reale. L'esercizio è concluso.


Pij88u9k

ESERCIZI DI RIEPILOGO SUGLI INTEGRALI INDEFINITI


Gli esercizi che seguono sono di riepilogo, dunque richiedono la conoscenza di tutte le strategie di calcolo degli integrali.

Esercizio 1. Risolvere

Svolgimento: è sufficiente scrivere l'integranda come una potenza con esponente negativo e utilizzare la regola di integrazione delle
potenze

pertanto

91
eBook edito da YouMath.it acquistato da Navdeep Singh
YouMath.it

Ricordando infine che , si ottiene

Finito!

Esercizio 2. Risolvere il seguente integrale utilizzando le regole di integrazione opportune

Svolgimento: per risolvere questo integrale utilizziamo la linearità di cui gode tale operatore: tale proprietà permette di scrivere
l'integrale della somma come somma di integrali. Osserviamo, inoltre, che ogni addendo dell'integranda si può tranquillamente
esprimere sotto forma di potenza, pertanto interviene anche la regola di integrazione per le potenze
Pij88u9k

Ora che conosciamo le strategie di soluzione procediamo con la soluzione dell'integrale

Per la linearità si ha

Esprimiamo ogni termine in forma di potenza

e facciamo intervenire la regola di integrazione delle potenze

L'integrale è risolto, bisogna portare a termine i semplici calcoli

92
Navdeep Singh - navdeep99@outlook.it - 146.241.52.118 - 2020-01-04 22:31:14 r1t2e3m4u5k cikelpo
YouMath.it

Esercizio 3. Risolvere il seguente integrale

Svolgimento: l'integrale diventa molto semplice se scriviamo i termini sotto forma di potenze, in particolare

Possiamo scrivere quindi l'integrale come segue

Bene, ora spezziamo l'integrale della somma come somma di integrali


Pij88u9k

Osserviamo che è intervenuta la regola di integrazione della potenza per risolvere i tre integrali:

L'esercizio è terminato.

Esercizio 4. Determinare la famiglia delle primitive della funzione

Svolgimento: determinare l'insieme di tutte le primitive di equivale a risolvere l'integrale indefinito in cui la funzione
integranda è proprio , ossia

Grazie alle proprietà delle potenze possiamo esprimere come , in questo modo l'integrale diventa:

93
eBook edito da YouMath.it acquistato da Navdeep Singh
YouMath.it

Attenzione: la costante moltiplicativa può essere trasportata fuori dal simbolo di integrazione

e a questo punto possiamo ricondurci all'integrale notevole in forma generale dell'esponenziale, ossia:

per farlo, moltiplichiamo e dividiamo l'integranda per :

Eseguiamo gli ultimi conti così da concludere che

dove è una costante reale. Finito.

Pij88u9k
Esercizio 5. Risolvere il seguente integrale

Svolgimento: la strategia risolutiva consiste nello sviluppare il quadrato presente nell'integranda

dopodiché interviene la proprietà di linearità dell'integrale che ci permette di scrivere l'integrale della somma come somma di
integrali, oltre a trasportare fuori le costanti moltiplicative

Scriviamo tutti ii termini sotto forma di potenza, tenendo presente che una potenza al denominatore può essere portata a numeratore
a patto di cambiare il segno all'esponente

A questo punto è d'obbligo l'uso della regola di integrazione di una potenza, grazie alla quale otteniamo

94
Navdeep Singh - navdeep99@outlook.it - 146.241.52.118 - 2020-01-04 22:31:14 r1t2e3m4u5k cikelpo
YouMath.it

dove è una costante reale. Non ci resta che scrivere per bene il risultato

e mettere un punto all'esercizio.

Esercizio 6. Risolvere il seguente integrale

Svolgimento: l'idea per calcolare questo integrale è in soldoni: spezzare la frazione e sfruttare la linearità dell'integrale cosicché ci si
riduce a calcolare

Pij88u9k il semplificabile e scriviamo le frazioni nella forma


Semplifichiamo : è cosa buona e giusta osservare che (deriva dalla
definizione stessa di radicale)

Ora facciamo riferimento ad una bellissima proprietà degli integrali, vale a dire l'omogeneità che ci permette di trasportare fuori dal
simbolo di integrazione una costante moltiplicativa

e grazie alla regola di integrazione delle potenze scriviamo

dove , pertanto possiamo concludere che

Finito.

Esercizio 7. Determinare la famiglia delle primitive della funzione

95
eBook edito da YouMath.it acquistato da Navdeep Singh
YouMath.it

Svolgimento: in breve l'esercizio chiede di risolvere l'integrale indefinito

che può essere affrontato o per mezzo di una sostituzione, oppure mediante l'uso dell'integrale di una potenza in forma generale,
ossia

Osserviamo che la derivata di è esattamente , pertanto l'integrale in questione si presenta nella forma notevole

appena vista nel caso particolare , quindi:

L'esercizio è concluso.

Esercizio 8. Svolgere il seguente integrale indefinito

Pij88u9k per calcolare l'integrale


Svolgimento:

dobbiamo solo osservare che siamo di fronte a un prodotto di funzioni, delle quali il primo fattore è la derivata dell'argomento del
secondo fattore. In accordo con il teorema di derivazione della funzione composta, applicato al contrario, avremo dunque che

In alternativa possiamo integrare per sostituzione ponendo , per cui e l'integrale diventa

vale a dire un banalissimo integrale fondamentale. Non ci resta che ripristinare la variabile ricordando la sostituzione così
da giungere allo stesso risultato scritto in precedenza.

Esercizio 9. Risolvere il seguente integrale

Svolgimento: la strada migliore per risolvere

consiste nello spezzare la frazione e conseguentemente spezzare l'integrale come somma di integrali

96
Navdeep Singh - navdeep99@outlook.it - 146.241.52.118 - 2020-01-04 22:31:14 r1t2e3m4u5k cikelpo
YouMath.it

Semplifichiamo con così da ricondurci alla somma di due integrali immediati

In definitiva

dove è una costante reale. Finito.

Esercizio 10. Determinare la famiglia delle primitive della funzione

Svolgimento: dobbiamo semplicemente risolvere l'integrale

Pij88u9k

possibilmente riconducendoci all'integrale notevole . Per fare ciò, è sufficiente mettere

in evidenza al denominatore

Con l'aiuto delle proprietà delle potenze possiamo scrivere come

Abbiamo bisogno della derivata di al numeratore, ossia abbiamo bisogno di . Moltiplichiamo e dividiamo per così da ottenere

quanto segue:

Possiamo quindi concludere che

dove è una costante additiva reale.

97
eBook edito da YouMath.it acquistato da Navdeep Singh
YouMath.it

Esercizio 11. Calcolare l'integrale

utilizzando il metodo di integrazione per parti.

Svolgimento: come suggerito dalla traccia procediamo con il metodo di integrazione per parti scegliendo come fattore finito (da
derivare)

e come fattore differenziale (da integrare)

Utilizziamo la formula di integrazione per parti, grazie alla quale otteniamo:

Utilizziamo il tipico trucco algebrico: quando siamo di fronte ad integrali di questo tipo sommiamo e sottraiamo al numeratore della
funzione integranda al secondo membro. Questo barbatrucco permette di esprimere l'integrale come somma di integrali elementari,
in dettaglio:

Pij88u9k
spezziamo la frazione in modo che si possa semplificare

Osserviamo che, poiché (altrimenti il logaritmo non esiste) allora pertanto possiamo riscrivere il
risultato come segue:

Perfetto, l'esercizio è concluso.

Esercizio 12. Risolvere il seguente integrale

98
Navdeep Singh - navdeep99@outlook.it - 146.241.52.118 - 2020-01-04 22:31:14 r1t2e3m4u5k cikelpo
YouMath.it
Svolgimento: quello proposto è un integrale di una funzione razionale fratta, in cui il grado del numeratore coincide con il grado del
polinomio al denominatore. Sappiamo che in questi casi converrebbe eseguire la divisione polinomiale tra numeratore e
denominatore, ma in questo ci facciamo furbi e utilizziamo un trucchetto algebrico: sommiamo e sottraiamo al numeratore:

Spezziamo la frazione così da semplificare

e a questo punto sfruttiamo la linearità così da scrivere l'integrale della somma come somma di integrali

Entrambi sono integrali immediati, pertanto possiamo scrivere subito il risultato


Pij88u9k

dove è una costante reale. Scriviamo per bene il risultato

e mettiamo un punto a questo esercizio.

Esercizio 13. Svolgere il seguente integrale utilizzando le identità trigonometriche

Svolgimento: per risolvere l'integrale proposto è necessario usare prima di tutto la definizione di potenza così da esprimere
come il prodotto tra , inoltre per la relazione fondamentale della Trigonometria possiamo scrivere come
. Grazie a queste identità, l'integrale di partenza diventa veramente semplice da svolgere, ma entriamo nel vivo
dell'azione

Sviluppiamo il prodotto

99
eBook edito da YouMath.it acquistato da Navdeep Singh
YouMath.it

e utilizziamo la linearità così da esprimere l'integrale come somma (algebrica) di integrali

Sia il primo che il secondo integrale sono immediati, in particolare il secondo è della forma

dove ovviamente . Attenzione! La derivata del coseno di è , di

conseguenza ci manca un segno per avere l'integrale nella forma richiesta. Poco male, è sufficiente trasportare il segno meno
all'interno dell'integrale e siamo a posto.

Possiamo quindi concludere che

Pij88u9k
dove è una costante reale.

Esercizio 14. Tra le primitive della funzione

determinare la primitiva passante per il punto .

Svolgimento: risolviamo l'esercizio per passi, calcolando prima di tutto l'insieme di tutte le primitive associato alla funzione :
detto in termini più espliciti dobbiamo risolvere l'integrale

Osserviamo che esso è praticamente nella forma con . Grazie alla regola di

integrazione appena scritta possiamo asserire che

e rappresenta la famiglia delle primitive associata alla funzione . Il secondo passaggio consiste nel determinare la primitiva di
che passa per il punto , o detto in altri termini, dobbiamo trovare la costante di modo che il grafico della funzione

passi per il punto . Imponiamo la condizione di passaggio:

100
Navdeep Singh - navdeep99@outlook.it - 146.241.52.118 - 2020-01-04 22:31:14 r1t2e3m4u5k cikelpo
YouMath.it

Possiamo quindi concludere che la primitiva di che passa per il punto è

L'esercizio è così concluso.

Esercizio 15. Calcolare il seguente integrale senza utilizzare il metodo di sostituzione

Svolgimento: l'integrale può essere risolto con la sostituzione , ma la traccia ci impedisce di utilizzare tale strategia
risolutiva. L'alternativa consiste nello spezzare la frazione e in un secondo momento spezzare l'integrale come somma di integrali

Pij88u9k
A questo punto osserviamo che così che possiamo scrivere

Ora si può integrare per direttissima: basta notare che sia il primo che il secondo integrale si presentano nella forma

Grazie alla formula di integrazione per le potenze in forma generale possiamo asserire che il primo integrale è

mentre

In definitiva

101
eBook edito da YouMath.it acquistato da Navdeep Singh
YouMath.it

Concludiamo l'esercizio scrivendo per bene la soluzione

dove è una costante reale.

Esercizio 16. Risolvere il seguente integrale utilizzando il metodo di sostituzione

Svolgimento: per risolvere l'integrale, applichiamo il metodo di integrazione per sostituzione, ponendo per la quale non
calcoliamo il differenziale della trasformazione inversa, bensì il differenziale della trasformazione diretta. Tramite una semplice

derivazione otteniamo che . Sostituiamo tutto nell'integrale così da ottenere

A questo punto l'integrale è semplice, infatti per le proprietà delle potenze possiamo riscriverlo nella forma

e moltiplicando e dividendo per , ossia

Pij88u9k

abbiamo come integranda una derivata esatta, quindi possiamo applicare la formula di integrazione delle potenze in forma generale
avendo come risultato:

Non resta che effettuare la sostituzione al contrario, così da ripristinare la variabile

e portare a termine il nostro compito.

Esercizio 17. Calcolare il seguente integrale parametrico riconducendosi ad un integrale notevole

Svolgimento: quello proposto è un integrale parametrico nella variabile e con parametro . L'idea è quella di ricondursi
all'integrale fondamentale

Affinché compaia al denominatore dell'integrale

102
Navdeep Singh - navdeep99@outlook.it - 146.241.52.118 - 2020-01-04 22:31:14 r1t2e3m4u5k cikelpo
YouMath.it

mettiamo in evidenza

e grazie alle proprietà delle potenze che hanno lo stesso esponente si ha inoltre

Per ricondurci all'integrale notevole dobbiamo avere al numeratore dell'integranda un fattore correttivo che coincide con la derivata

di ossia . Per farlo apparire è sufficiente moltiplicare e dividere per

Portiamo fuori dal simbolo di integrazione

e semplifichiamo il semplificabile

Pij88u9k

A questo punto interviene l'integrale notevole scritto in precedenza:

con costante reale. Concludiamo l'esercizio scrivendo per bene il risultato

Osservazione: l'integrale proposto ricorre in moltissimi esercizi, suggeriamo di tenere a mente le idee e le strategie usate per
giungere al risultato.

Esercizio 18. Risolvere il seguente integrale irrazionale fratto

utilizzando la sostituzione .

Svolgimento: l'esercizio propone come metodo risolutivo l'integrazione per sostituzione, proponendo la sostituzione , da cui

e pertanto il differenziale diventa

Grazie alla sostituzione proposta

103
eBook edito da YouMath.it acquistato da Navdeep Singh
YouMath.it

diventa

Eseguiamo alcune operazioni algebriche così da esprimere in forma normale le frazioni di frazioni

Semplificando il semplificabile, otteniamo

Pij88u9k

Ora procediamo ancora una volta per sostituzione, ponendo da cui ricaviamo il nuovo differenziale

L'integrale a questo punto è immediato

dove è una costante reale. Ora ripercorreremo le sostituzioni fatte a ritroso, in questo modo esprimeremo il risultato nella variabile
: partiamo da

e concludiamo con :

In definitiva possiamo concludere che

104
Navdeep Singh - navdeep99@outlook.it - 146.241.52.118 - 2020-01-04 22:31:14 r1t2e3m4u5k cikelpo
YouMath.it

dove è una costante reale additiva.

Esercizio 19. Calcolare il seguente integrale

Svolgimento: il metodo di integrazione per parti è quello che conduce più velocemente alla soluzione. Prendiamo come fattore finito,
da derivare, la funzione logaritmo

mentre prendiamo come fattore differenziale la funzione polinomiale, di cui ci serva una primitiva qualsiasi (e noi prenderemo quella
con costante additiva nulla per comodità)

Grazie alla formula di integrazione per parti otteniamo

PerPij88u9k
linearità, possiamo spezzare l'integrale come somma di integrali (attenzione al segno ­ che si distribuisce ad entrambi gli
integrali) e in più possiamo trasportare fuori dal simbolo di integrazione le costanti moltiplicative

Siamo di fronte a due integrali di potenze e applicando la relativa regola otteniamo

con costante reale. Per concludere scriviamo per bene il risultato

dove è l'ormai solita costante reale. Abbiamo finito!

Esercizio 20. Calcolare il seguente integrale utilizzando il metodo di sostituzione

Suggerimento:

Svolgimento: abbiamo bisogno di alcune manipolazioni algebriche per esprimere l'integranda in modo diverso, in particolare
inizieremo con il raccogliere totalmente :

105
eBook edito da YouMath.it acquistato da Navdeep Singh
YouMath.it

Abbiamo espresso la funzione integranda come prodotto tra la potenza terza della funzione tangente e la sua derivata. Procediamo
per sostituzione ponendo da cui il differenziale è . Effettuiamo la sostituzione così che
l'integrale diventi

Ripristiniamo la variabile sostituendo a la funzione tangente di . Il risultato dell'integrale di partenza è quindi

dove è una costante reale. L'esercizio è concluso.

Esercizio 21. Risolvere il seguente integrale

Svolgimento: l'integranda è una funzione razionale fratta in cui il polinomio al numeratore ha lo stesso grado del polinomio al
denominatore e in questi casi una mossa furba consiste nell'effettuare la divisione polinomiale tra il numeratore e il denominatore,
ma di seguito procederemo in modo diverso, facendo intervenire i tanto temuti trucchetti algebrici.
Pij88u9k
Il nostro intento è quello di ricreare il denominatore al numeratore e per fare ciò mettiamo in evidenza al numeratore

Grazie all'omogeneità dell'integrale possiamo trasportare fuori dal simbolo di integrazione la costante moltiplicativa

Ora moltiplichiamo e dividiamo per così che il coefficiente di al numeratore sia uguale a quello di a denominatore

Facciamo apparire al numeratore sommando e sottraendo

e spezziamo la frazione così da poter effettuare le dovute semplificazioni

106
Navdeep Singh - navdeep99@outlook.it - 146.241.52.118 - 2020-01-04 22:31:14 r1t2e3m4u5k cikelpo
YouMath.it

Possiamo spezzare l'integrale come somma di integrali e portare fuori le costanti moltiplicative, in forza della linearità di cui gode
l'operatore

Il primo integrale è immediato, il secondo può essere risolto velocemente a patto di moltiplicare e dividere per , così da avere al
numeratore esattamente la derivata del denominatore

Non ci resta che espandere i prodotti

e rappurare le costanti in un unica costante additiva


Pij88u9k

dove . Prima di chiudere definitivamente la faccenda, riscriviamo per bene il risultato dell'integrale

con . Fatto!

Esercizio 22. Calcolare il seguente integrale con il metodo di sostituzione

Svolgimento: la strategia risolutiva dell'integrale

consiste nel porre da cui, esprimendo la variabile in funzione di , e inoltre il nuovo differenziale è
. Sostituendo nell'integrale otteniamo

107
eBook edito da YouMath.it acquistato da Navdeep Singh
YouMath.it

e, a questo punto, possiamo tranquillamente utilizzare la linearità dell'integrale che ci permette di:

1) scrivere l'integrale della somma come somma di integrali;

2) trasportare fuori dal simbolo di integrazione le costanti moltiplicative.

Grazie alle due proprietà l'integrale diventa

Gli integrali ottenuti sono immediati e si possono calcolare con la regola di integrazione delle potenze

Manca poco per concludere l'esercizio, dobbiamo solamente riportare il risultato nella variabile , tenendo a mente la sostituzione
fatta, ossia

e con le proprietà dei radicali esprimiamo il risultato come segue:

Pij88u9k

Gli ultimi passaggi sono esclusivamente estetici, effettuati con l'intento di scrivere il risultato in maniera compatta, possiamo
comunque considerare concluso l'esercizio al passaggio indicato con .

Esercizio 23. Determinare la famiglia delle primitive della funzione logaritmica

Svolgimento: in altri termini, l'esercizio chiede di calcolare l'integrale indefinito

e la strategia risolutiva passa attraverso il metodo di integrazione per parti scegliendo come fattore finito, facile da derivare, la
funzione

e come fattore differenziale, facile da integrare, la funzione

108
Navdeep Singh - navdeep99@outlook.it - 146.241.52.118 - 2020-01-04 22:31:14 r1t2e3m4u5k cikelpo
YouMath.it

Per la formula di integrazione per parti, otteniamo:

Concentriamo la nostra attenzione sull'integrale

che può essere risolto con il metodo dei fratti semplici. Il primo passo consiste nella decomposizione del denominatore come
prodotto di fattori irriducibili

Fatto ciò, il secondo passo consiste nel determinare due costanti reali tali che

Portiamo le frazioni algebriche a denominatore comune

I denominatori non servono più perché uguali membro a membro

Pij88u9k
e raccogliendo secondo le potenze di otteniamo l'identità polinomiale

A questo punto interviene il principio di identità dei polinomi, con cui possiamo costruire il sistema lineare

La coppia che soddisfa il sistema è , pertanto l'integranda si riscrive come

e dunque

Ricomponiamo il risultato dell'integrale di partenza e scriviamo:

109
eBook edito da YouMath.it acquistato da Navdeep Singh
YouMath.it

Ora l'esercizio è concluso.

Esercizio 24. Determinare la primitiva di che nel punto sia tangente alla retta di equazione
.

Svolgimento: per determinare una primitiva della funzione

è sufficiente calcolarne l'integrale indefinito mediante la formula di integrazione per parti, prendendo come fattore finito, da derivare,
e come fattore differenziale , la cui primitiva (a meno di costanti additive) è data da , dunque

La retta tangente al grafico di una funzione in un punto ha il coefficiente angolare dato dalla valutazione della derivata prima della
funzione nel punto.

Il teorema fondamentale del calcolo integrale ci dice che la derivata prima della funzione integrale è l'integranda stessa. La
valutazione dell'integranda nel punto coincide con il coefficiente angolare della retta: retta tangente e grafico devono inoltre
intersecarsi nel punto di tangenza, quindi possiamo determinare la costante individuando l'ordinata della retta in corrispondenza
dell'ascissa :

Pij88u9k

e imponendo che appartenga al grafico della primitiva otteniamo l'equazione

da cui . La primitiva richiesta è data da

Abbiamo così determinato la primitiva di che è tangente alla retta .

Gli integrali che seguono hanno un livello di difficoltà maggiore rispetto a quelli proposti.

Esercizio 25. Calcolare il seguente integrale

Svolgimento: l'integrale

si può risolvere mediante l'uso del metodo di integrazione per parti, scegliendo come fattore finito da cui

. Il fattore differenziale è da cui

Utilizziamo la formula di integrazione per parti

110
Navdeep Singh - navdeep99@outlook.it - 146.241.52.118 - 2020-01-04 22:31:14 r1t2e3m4u5k cikelpo
YouMath.it

e svolgiamo i semplici calcoli

Grazie alla prima relazione fondamentale della Trigonometria, possiamo scrivere e rimpiazzando
nell'integranda otteniamo:

Spezziamo la frazione e semplifichiamo il semplificabile

La linearità dell'integrale permette di scrivere l'integrale della somma come somma di integrali

Pij88u9k
L'ultimo integrale è immediato, mentre

è un po' più delicato da trattare, perciò lo risolviamo a parte. Iniziamo subito con un barbatrucco, moltiplicando e dividendo per

Ancora una volta interviene in nostro soccorso la relazione fondamentale della Trigonometria che ci assicura l'uguaglianza
. Rimpiazzando con l'integrale diventa

L'integrale si presta bene per una sostituzione, ponendo e calcolando il nuovo differenziale segue
che:

Grazie alla sostituzione abbiamo ottenuto l'integrale di una funzione razionale fratta che può essere risolto mediante il metodo dei
fratti semplici

111
eBook edito da YouMath.it acquistato da Navdeep Singh
YouMath.it

Ora ripristiniamo la variabile , tenendo a mente la sostituzione fatta, ossia

Bene, abbiamo tutto quello che ci serve per ricomporre l'integrale di partenza

dove è una costante reale. L'esercizio è concluso.

Esercizio 26. Calcolare il seguente integrale

tenendo a mente che .


Pij88u9k
Svolgimento: per definizione di potenza possiamo scrivere l'integrale come segue

e integriamo per parti prendendo come derivata, una primitiva della quale è . Otteniamo:

Grazie all'identità proposta dall'esercizio stesso, otteniamo

ossia

Abbiamo raggiunto quindi l'uguaglianza

112
Navdeep Singh - navdeep99@outlook.it - 146.241.52.118 - 2020-01-04 22:31:14 r1t2e3m4u5k cikelpo
YouMath.it

Poniamo e scriviamo

Consideriamo come se fosse un'incognita e risolviamo l' "equazione"

Dividiamo membro a membro per e otteniamo:

Possiamo quindi asserire che

dove è una costante reale. Finalmente abbiamo concluso l'esercizio.

ESERCIZI DI RIEPILOGO SUGLI INTEGRALI DEFINITI


Pij88u9k
Esercizio 1. Calcolare il seguente integrale definito

Svolgimento: per risolvere un integrale definito, è necessario determinare una primitiva qualsiasi della funzione integranda, che
chiamiamo , dopodiché calcoleremo la differenza tra la primitiva valutata nell'estremo superiore e la primitiva valutata
all'estremo inferiore.

Risolviamo quindi l'integrale definito

osservando che se moltiplichiamo e dividiamo per l'integranda otteniamo un integrale di una potenza in forma generale di cui già
conosciamo il risultato

Valutiamo la funzione tra parentesi quadre agli estremi di integrazione

113
eBook edito da YouMath.it acquistato da Navdeep Singh
YouMath.it

L'esercizio è terminato, possiamo infatti concludere che l'integrale di partenza vale .

Esercizio 2. Calcolare l'integrale definito

Svolgimento: grazie alle proprietà degli integrali definiti, possiamo esprimere l'integrale della somma come somma di integrali:

Il primo è un integrale immediato, mentre il secondo può essere ricondotto all'integrale fondamentale in forma generale

, l'unica cosa che manca nell'integranda è un segno meno. Sappiamo già come affrontare queste

situazioni, è sufficiente moltiplicare e dividere per

Pij88u9k

Non ci resta che valutare le funzioni tra le parentesi quadre e portare a termine i conti

Abbiamo concluso l'esercizio, l'integrale definito vale esattamente .

Esercizio 3. Determinare il valore del seguente integrale definito

Svolgimento: in questo caso il calcolo esplicito della primitiva è davvero molto scomodo, dunque dobbiamo cambiare strada e fare
uso di una proprietà notevole degli integrali definiti: se una funzione è continua in un intervallo con allora
l'integrale definito di sull'intervallo è uguale a .

Grazie a questo semplice quanto importante risultato possiamo concludere che

infatti:

l'intervallo di integrazione è simmetrico rispetto all'origine;

la funzione è continua e dispari, ossia

114
Navdeep Singh - navdeep99@outlook.it - 146.241.52.118 - 2020-01-04 22:31:14 r1t2e3m4u5k cikelpo
YouMath.it
L'esercizio è terminato.

Esercizio 4. Calcolare il seguente integrale definito

Svolgimento: risolviamo l'integrale

utilizzando le proprietà delle funzioni esponenziali, così da esprimere l'integranda in modo differente. La relazione che ci viene in
soccorso è

mediante la quale scriviamo

L'integrale di partenza diventa quindi

e può essere risolto riconducendolo all'integrale immediato in forma generale . Ci manca però la
Pij88u9k

derivata di , ossia . Non ci resta quindi che utilizzare il solito stratagemma moltiplicando e dividendo per tale

costante

A questo punto dobbiamo valutare la differenza delle valutazioni

Per le proprietà dei logaritmi in combinazione con le proprietà delle potenze, inoltre, possiamo esprimere il risultato come segue:

Ora possiamo considerare concluso l'esercizio.

Esercizio 5. Risolvere il seguente integrale definito

115
eBook edito da YouMath.it acquistato da Navdeep Singh
YouMath.it

Svolgimento: per calcolare l'integrale definito

spezziamo la frazione e sfruttiamo la linearità dell'integrale

Il primo integrale è un logaritmo, il secondo invece lo riscriviamo come

e dunque

Pij88u9k

L'esercizio è completato!

Esercizio 6. Determinare il valore del seguente integrale definito

Svolgimento: per calcolare l'integrale

è sufficiente osservare che la derivata del coseno è il seno con segno negativo, quindi se diamo un'aggiustatina all'integranda
moltiplicando e dividendo per

ci riconduciamo alla formula di integrazione del seno in forma generale, ossia

Abbiamo così che l'integrale precedente è uguale a


116
Navdeep Singh - navdeep99@outlook.it - 146.241.52.118 - 2020-01-04 22:31:14 r1t2e3m4u5k cikelpo
YouMath.it

Per ottenere il risultato abbiamo semplicemente calcolato la differenza delle valutazioni della primitiva agli estremi dell'intervallo di
integrazione.

Osservazione: l'integrale proposto può essere risolto mediante l'uso degli archi associati con cui possiamo esprimere
. Lasciamo allo studente volenteroso il completamento dei calcoli.

Esercizio 7. Dimostrare che

Svolgimento: dobbiamo semplicemente risolvere l'integrale

e mostrare che il risultato è effettivamente . La strategia risolutiva passa attraverso le proprietà delle potenze grazie alle quali
Pij88u9k
l'integrale diventa

Ora osserviamo che l'integranda è della forma per cui possiamo far intervenire la formula di integrazione delle
potenze in forma generale

Non resta che valutare la primitiva agli estremi

Ecco fatto!

Esercizio 8. Calcolare il seguente integrale definito

Svolgimento: procediamo per sostituzione ponendo

117
eBook edito da YouMath.it acquistato da Navdeep Singh
YouMath.it
Attenzione: quando effettuiamo una sostituzione, cambiano anche gli estremi di integrazione: per ottenere i nuovi estremi è
sufficiente infatti valutare la funzione che sostituiamo negli estremi dati.

All'estremo inferiore associamo , mentre all'estremo superiore associamo

L'integrale

diventa

e l'esercizio è concluso.

Esercizio 9. Calcolare il seguente integrale definito con il metodo di integrazione per parti

Svolgimento: vogliamo calcolare

Pij88u9k

e per farlo, ci basta ricorrere alla formula di integrazione per parti

Nel nostro caso prendiamo come derivata , che ammette come primitiva e come funzione da derivare
. Grazie alla formula di integrazione per parti scriviamo

L'esercizio è completato.

Esercizio 10. Risolvere l'integrale definito

Svolgimento: per risolvere l'integrale

118
Navdeep Singh - navdeep99@outlook.it - 146.241.52.118 - 2020-01-04 22:31:14 r1t2e3m4u5k cikelpo
YouMath.it

procediamo con l'integrazione per sostituzione:

Grazie a questa sostituzione gli estremi di integrazione si trasformano di conseguenza, in particolare:

­ all'estremo associamo l'estremo ;

­ all'estremo associamo l'estremo .

L'integrale pertanto diventa

Abbiamo finito.

Esercizio 11. Calcolare l'integrale

Pij88u9k
dove è la seguente funzione definita per casi

Svolgimento: dobbiamo calcolare l'integrale

dove la funzione

è definita per casi (o a tratti). Proprio perché la funzione è così fatta, è necessario spezzare l'integrale in base a come è definita sui
singoli intervalli:

Per semplicità di esposizione, risolviamo separatamente i due integrali

119
eBook edito da YouMath.it acquistato da Navdeep Singh
YouMath.it

mentre il secondo integrale può essere risolto mediante la formula di integrazione per parti

prendendo come fattore finito e come fattore differenziale e per la


formula di integrazione per parti, si ha che

L'integrale di partenza è dato dalla somma dei contributi dei singoli integrali, dunque possiamo concludere che:

L'esercizio è concluso.

Pij88u9k
Esercizio 12. Calcolare l'integrale definito

Svolgimento: l'integrale definito

può essere risolto per sostituzione ponendo

Vediamo come si trasformano gli estremi di integrazione:

­ all'estremo associamo ;

­ all'estremo associamo .

A causa della sostituzione scelta, l'integrale diventa

120
Navdeep Singh - navdeep99@outlook.it - 146.241.52.118 - 2020-01-04 22:31:14 r1t2e3m4u5k cikelpo
YouMath.it

Esprimiamo la radice quinta sotto forma di potenza e utilizziamo la relativa regola di integrazione

e infine eseguendo le ultime operazioni otteniamo il risultato dell'integrale

L'esercizio è terminato.

Esercizio 13. Determinare il valore del seguente integrale

Pij88u9k

Svolgimento: l'integrale può essere affrontato per parti scegliendo come fattore finito e come fattore differenziale

Calcoliamo a parte la derivata del fattore finito

e una primitiva del fattore differenziale

e infine procediamo con il calcolo facendo uso della formula di integrazione per parti

L'esercizio è concluso.

121
eBook edito da YouMath.it acquistato da Navdeep Singh
YouMath.it

Esercizio 14. Calcolare l'integrale

Svolgimento: l'integrale si presenta nella forma

anche se effettivamente non è così evidente: proviamo a fare alcune manipolazioni algebriche così da far risaltare ciò

La base del quadrato è e al numeratore troviamo esattamente la sua derivata: non ci sono dubbi, l'integrale è nella forma
richiesta, pertanto

Fatto!

Esercizio 15. Calcolare il seguente integrale definito

Pij88u9k

dove è la funzione segno.

Svolgimento: un modo furbo per risolvere l'integrale è asserire che poiché l'integranda è una funzione dispari e l'intervallo di
integrazione è simmetrico allora l'integrale è nullo. L'esercizio si potrebbe concludere così, ma pensiamo che possa essere
didatticamente interessante vedere anche il calcolo esplicito dell'integrale.

Vogliamo quindi calcolare l'integrale utilizzando una delle proprietà degli integrali definiti

Concentriamo la nostra attenzione sul primo integrale e osserviamo che nell'intervallo di integrazione abbiamo che:

conseguentemente:

122
Navdeep Singh - navdeep99@outlook.it - 146.241.52.118 - 2020-01-04 22:31:14 r1t2e3m4u5k cikelpo
YouMath.it

Risolviamo il secondo integrale, ossia

che nell'intervallo di integrazione si scrive come

A questo punto possiamo ricomporre il risultato dell'integrale di partenza

e abbiamo finito.

(Diffi cile) Esercizio 16. Calcolare il seguente integrale

Suggerimento:

Svolgimento: mediante il suggerimento possiamo esprimere così che valga l'uguaglianza

Pij88u9k

Procediamo per sostituzione ponendo

Gli estremi di integrazione si trasformano come segue:

­a associamo ;

­a associamo .

L'integrale diventa

Procediamo per fratti semplici, fattorizzando il denominatore come prodotto di polinomi irriducibili:

Il nostro obiettivo ora diventa quello di determinare due costanti reali tali che

123
eBook edito da YouMath.it acquistato da Navdeep Singh
YouMath.it
Portiamo a denominatore comune e semplifichiamolo, dopodiché con semplici calcoli si arriva a

Raccogliamo secondo le potenze di

e mediante il principio di identità dei polinomi costruiamo il sistema lineare

da cui si ottengono le due costanti e grazie alle quali l'integrale da risolvere diventa

Spezziamo l'integrale come somma di integrali e portiamo fuori dal simbolo di integrazione le costanti moltiplicative

Pij88u9k
Entrambi gli integrali sono uguali a dei logaritmi:

e dopo un po' di calcoli algebrici otteniamo

L'esercizio è concluso.

ESERCIZI SULLA MEDIA INTEGRALE


Esercizio 1. Dopo aver provato che la funzione

soddisfa le ipotesi del teorema della media integrale sull'intervallo , verificare ciò che il teorema stesso afferma.

Svolgimento: dobbiamo semplicemente verificare che la funzione data soddisfi le ipotesi del teorema della media integrale
nell'intervallo , ossia dobbiamo verificare che sia continua nell'intervallo .

124
Navdeep Singh - navdeep99@outlook.it - 146.241.52.118 - 2020-01-04 22:31:14 r1t2e3m4u5k cikelpo
YouMath.it
Nell'intervallo , la funzione è costante di conseguenza è certamente continua. Nell'intervallo , la funzione
è certamente continua perché di tipo polinomiale. Dobbiamo controllare la continuità nel punto di raccordo
e lo facciamo con la definizione stessa di continuità.

Calcoliamo il limite destro e il limite sinistro per e controlliamo che siano uguali al valore che la funzione assume nel punto
. Cominciamo con il limite destro che risulta essere

mentre il limite sinistro è

ed entrambi sono uguali al valore che la funzione assume nel punto , ossia . Possiamo stare tranquilli, la
continuità della funzione nell'intervallo è assicurata.

La tesi del teorema della media integrale ci informa dell'esistenza di un numero per cui si ha che:

Continuiamo nella risoluzione dell'esercizio calcolando l'integrale definito

che può essere risolto grazie alla proprietà additiva dell'integrale rispetto agli estremi dell'intervallo:

Pij88u9k

Sostituiamo in e risolviamo l'equazione

Attenzione perché l'espressione di varia in base all'intervallo su cui lavoriamo, in particolare se allora e
l'equazione scritta in precedenza è impossibile. Necessariamente si troverà in , nel qual caso l'equazione diventa

che è la soluzione richiesta: realizza il teorema della media integrale.

Esercizio 2. Dopo aver calcolato la media integrale della funzione sull'intervallo , determinare un
punto tale che .

Svolgimento: la media integrale di una funzione nell'intervallo è dato da:

Nel caso proposto , pertanto

125
eBook edito da YouMath.it acquistato da Navdeep Singh
YouMath.it

Per calcolare l'integrale del logaritmo procediamo per parti scegliendo come fattore finito il logaritmo e come fattore differenziale il
fattore "nascosto" . Con semplici passaggi algebrici giungeremo a:

Determiniamo ora il punto nel quale:

ossia

Applichiamo membro a membro l'esponenziale e mediante l'uso delle proprietà delle funzioni esponenziali avremo

La costante che realizza la tesi del teorema della media integrale è .

Esercizio 3. Determinare gli eventuali punti in cui la funzione definita da

Pij88u9k

Svolgimento: calcoliamo la media integrale della funzione sull'intervallo impostando l'integrale

Poiché la funzione è definita per casi utilizziamo la proprietà additiva degli integrali sugli estremi di integrazione che ci permette di
scrivere quanto segue:

Ora possiamo determinare per quali valori di si ha che

La funzione è definita per casi e dobbiamo ovviamente tenere in considerazione la sua natura. I primi due rami della funzione sono
costanti e non sono uguali a , dobbiamo quindi lavorare necessariamente nel terzo ramo, ossia quando . Otteniamo

pertanto l'equazione

126
Navdeep Singh - navdeep99@outlook.it - 146.241.52.118 - 2020-01-04 22:31:14 r1t2e3m4u5k cikelpo
YouMath.it

Il valore ottenuto appartiene all'intervallo quindi è soluzione accettabile. L'esercizio è concluso.

Esercizio 4. La media integrale della funzione calcolata nell'intervallo è

Svolgimento: l'esercizio chiede in soldoni di calcolare la media integrale definita come

dove . Nel caso in questione dovremo quindi calcolare l'integrale

che per linearità diventa

Pij88u9k

dunque la risposta esatta è la c). Abbiamo finito.

Esercizio 5. Determinare il valor medio di sull'intervallo

Svolgimento: dobbiamo semplicemente determinare il valor medio integrale associato alla funzione riferito

all'intervallo ossia

Dalla formula di duplicazione del coseno segue che

e sostituendo l'espressione equivalente di nell'integrale otteniamo

127
eBook edito da YouMath.it acquistato da Navdeep Singh
YouMath.it
Grazie alle proprietà lineari dell'integrale possiamo inoltre scrivere quanto segue:

Entrambi gli integrali sono di facile risoluzione:

Osserviamo inoltre che è una funzione continua in e per il teorema della media integrale esiste

tale che

Risolviamo l'equazione trigonometrica così da calcolare :

Attenzione! non è accettabile perché non appartiene all'intervallo considerato, mentre è il valore che soddisfa la

tesi del teorema della media integrale. L'esercizio è concluso.

Esercizio 6. Consideriamo la funzione definita da


Pij88u9k

determinare i valori di per i quali è applicabile il teorema della media integrale a nell'intervallo .

Svolgimento: le ipotesi del teorema della media integrale sono relativamente poche. Abbiamo bisogno semplicemente di una
funzione continua in un intervallo chiuso e limitato. La funzione

è continua sia nell'intervallo perché di tipo polinomiale, sia nell'intervallo perché quoziente di funzioni continue.
L'unico punto su cui c'è incertezza è il cosiddetto punto di raccordo . Dobbiamo quindi determinare di modo che la funzione
risulti continua in .

Ricordiamo che una funzione è continua in un punto di accumulazione interno al dominio se e solo se il limite destro e il limite
sinistro per che tendono al punto coincidono con il valore che la funzione assume in tale punto.

Calcoliamo il limite destro e il limite sinistro:

Osserviamo che la funzione per valori più piccoli di assume la forma , ecco perché nel limite appare tale espressione,
mentre per valori più grandi di la funzione assume la forma .

128
Navdeep Singh - navdeep99@outlook.it - 146.241.52.118 - 2020-01-04 22:31:14 r1t2e3m4u5k cikelpo
YouMath.it
I due limiti coincidono con il valore che la funzione assume in , ossia se e solo se . Possiamo quindi
concludere che la funzione rispetta le ipotesi del teorema della media integrale se e solo se .

Esercizio 7. Determinare il numero reale positivo tale che la funzione

abbia media integrale uguale a nell'intervallo . Dire poi esiste, senza calcolarlo, un punto appartenente a in cui la

funzione assume il valore della sua media integrale.

Svolgimento: il primo passaggio consiste nel determinare la media integrale della funzione

nell'intervallo con ossia

L'integrale è di facile risoluzione: è sufficiente spezzare l'integrale come somma di integrali e utilizzare la formula di integrazione
delle potenze così da ottenere

Pij88u9k

Possiamo mettere in evidenza e semplificarlo in un secondo momento

Abbiamo quindi determinato la media integrale, al variare di . L'esercizio ora chiede di determinare il/i valore/i di tale/i per cui:

Portiamo tutto a denominatore comune e semplifichiamolo

Abbiamo ottenuto un'equazione di secondo grado completa il cui discriminante associato è:

e le soluzioni dell'equazione sono:

129
eBook edito da YouMath.it acquistato da Navdeep Singh
YouMath.it

La soluzione non è accettabile perché è negativa. Il valore di che realizza la richiesta dell'esercizio è .

Rispondiamo alla seconda parte dell'esercizio, facendo intervenire il teorema della media integrale la cui ipotesi è soddisfatta. La
funzione è infatti continua nell'intervallo (è di tipo polinomiale), di conseguenza esiste un valore per il quale
coincide con la media integrale. Finalmente l'esercizio è concluso.

ESERCIZI SULL'AREA CON INTEGRALI


Esercizio 1. Data la funzione di variabile reale

calcolare l'area della regione del piano delimitata dal grafico della funzione e dalle rette .

Svolgimento: volendo calcolare l'area della regione di piano sottesa dal grafico di una funzione rispetto all'asse delle
ascisse e limitata dalle rette verticali sarà sufficiente calcolare l'integrale definito

Nel nostro caso la funzione è non negativa sull'intervallo dunque possiamo tralasciare il modulo:

Pij88u9k
Per calcolare l'integrale definito è sufficiente moltiplicare e dividere l'integranda per

In definitiva l'area della regione di piano misura esattamente

e l'esercizio è concluso.

Esercizio 2. Data la funzione

calcolare l'area sottesa dal grafico della funzione, il quale interseca le rette e .

Svolgimento: per risolvere l'esercizio basta ricordare il significato geometrico degli integrali. Tenendo conto che l'integrale equivale
all'area sottesa dal grafico della funzione rispetto all'asse , intesa come area con segno, l'esercizio è estremamente semplice.

Le rette individuano l'intervallo di integrazione. Su tale intervallo la funzione assume valori negativi. Per
vederlo basta risolvere la disequazione di grado superiore al secondo

da cui, con un raccoglimento parziale

ossia
130
Navdeep Singh - navdeep99@outlook.it - 146.241.52.118 - 2020-01-04 22:31:14 r1t2e3m4u5k cikelpo
YouMath.it

La funzione è quindi positiva per , negativa per .

Poiché la funzione è negativa su , possiamo calcolare l'integrale

ossia

Il valore dell'integrale è negativo perché il grafico della funzione si trova, per , nel semipiano delle ordinate negative.

Per conoscere il valore d'area a prescindere dal segno se ne prende il valore assoluto, e si trova

Esercizio 3. Calcola l'area della parte di piano delimitata dalle rette di equazione e e dalla parabola
.

Pij88u9k partiamo dal presupposto che l'integrale secondo Riemann ha il significato geometrico di area sottesa dal grafico della
Svolgimento:
funzione integranda.

La regione di piano di cui vogliamo calcolare l'area è sottesa dal ramo di parabola riferito all'intervallo di ascisse , che si
estende fino al segmento orizzontale situato all'ordinata .

L'area calcolata con un integrale di Riemann si riferisce all'area racchiusa tra il grafico della funzione e l'asse delle ascisse
, quindi nel caso considerato dobbiamo calcolare l'area richiesta come somma di due aree: una è data dall'integrale della funzione
sull'intervallo

L'altra è l'area del rettangolo compreso tra le due rette orizzontali e le due rette verticali la cui
base coincide con l'ampiezza dell'intervallo e l'altezza coincide invece con la distanza tra rette .

In definitiva l'area della regione di piano con cui stiamo lavorando è

e l'esercizio finisce qui.

Esercizio 4. Calcolare l'area della porzione di piano compresa tra l'asse delle e il grafico di nell'intervallo .

Svolgimento: per determinare l'area della porzione di piano data dall'esercizio dobbiamo calcolare l'integrale

131
eBook edito da YouMath.it acquistato da Navdeep Singh
YouMath.it

Studiamo il segno dell'argomento del valore assoluto, impostando la disequazione , da studiare nell'intervallo .

Naturalmente , mentre . Mediante la proprietà sull'additività degli


estremi possiamo esprimere

Calcoliamo l'integrale indefinito

che può essere risolto mediante il metodo di integrazione per parti scegliendo come fattore finito e come fattore
differenziale . Grazie alla formula di integrazione per parti si ha che

In forza del teorema fondamentale del calcolo integrale si ha

Pij88u9k

mentre

Ora non ci resta che ricostruire l'integrale che definisce l'area stando particolarmente attenti ai segni

In definitiva possiamo concludere che la regione piana considerata ha .

Esercizio 5. Si determinino le coordinate dei punti comuni alle due curve aventi le seguenti equazioni e si calcoli l'area della parte
di piano limitata dagli archi delle due curve considerate, aventi per estremi i punti prima determinati:

Svolgimento: cominciamo determinando i punti di intersezione tra le due funzioni

(è l'equazione di una parabola convessa)

(è l'equazione di una retta)

impostando il sistema di equazioni

132
Navdeep Singh - navdeep99@outlook.it - 146.241.52.118 - 2020-01-04 22:31:14 r1t2e3m4u5k cikelpo
YouMath.it

da cui possiamo estrapolare la risolvente associata:

Abbiamo ottenuto un'equazione di secondo grado completa le cui soluzioni sono le ascisse dei punti di intersezione. Per ottenere le
ordinate è sufficiente sostituire i valori ottenuti nell'equazione .

I punti di intersezione sono . La prima parte dell'esercizio è terminata, non ci resta che determinare
l'area della parte di piano limitata dalla retta e dalla parabola, ma prima dobbiamo comprendere chi tra le due è ad una quota
maggiore. Impostiamo quindi la seguente disequazione

e risolviamola non prima di averla scritta in forma normale

Deduciamo quindi che nell'insieme la parabola di equazione è ad una quota maggiore


rispetto alla retta di equazione mentre nell'intervallo la funzione è ad una quota
maggiore rispetto alla funzione .

Grazie a queste considerazioni possiamo calcolare l'area della porzione di piano che giace tra la retta e la parabola, ossia
calcoleremo l'area colorata in blu del seguente grafico
Pij88u9k

Grazie all'interpretazione geometrica dell'integrale definito si ha che l'area è data da

133
eBook edito da YouMath.it acquistato da Navdeep Singh
YouMath.it

Facciamo presente che per calcolare l'area della porzione di piano tra due grafici di funzioni è sufficiente integrare la funzione che
"sta più in alto" meno la funzione che "sta più in basso". Nel nostro caso, nell'intervallo la retta stava più in alto rispetto alla
parabola.

Esercizio 6. Calcolare l'area della regione di piano delimitata dal grafico della funzione e dall'asse delle , in corrispondenza
dell'intervallo e con

Svolgimento: dobbiamo semplicemente calcolare l'area della parte di piano limitata dal grafico di funzione, dall'asse delle e dalle
rette mediante l'interpretazione geometrica di integrale definito.

In altri termini dobbiamo risolvere l'integrale


Pij88u9k

Attenzione, la funzione è definita per casi, e l'intervallo di integrazione è insieme in cui pertanto l'integrale
da calcolare è

che tramite le proprietà della funzione valore assoluto si può scrivere come

Nell'intervallo il prodotto è positivo, conseguentemente il valore assoluto è superfluo e può essere tranquillamente
eliminato.

Risolviamo l'integrale con il metodo di integrazione per parti scegliendo come fattore finito e come fattore differenziale
:

Procediamo nuovamente per parti scegliendo come fattore finito e come fattore differenziale . In accordo
con la formula di integrazione per parti

134
Navdeep Singh - navdeep99@outlook.it - 146.241.52.118 - 2020-01-04 22:31:14 r1t2e3m4u5k cikelpo
YouMath.it

Possiamo quindi concludere che l'area della parte di piano che ci interessa è .

Esercizio 7. Date le funzioni , l'area della regione finita di piano compresa tra le due
funzioni è

Svolgimento: l'area compresa tra il grafico di due funzioni si determina con un opportuno integrale definito. L'integrale secondo
Riemann di una funzione limitata e integrabile su ha infatti il significato geometrico di area sottesa tra il grafico della
funzione e l'asse delle ascisse sull'intervallo .

Nel caso considerato ci interessa l'area compresa tra i grafici delle funzioni

Possiamo calcolare tale area come differenza tra l'area sottesa dal grafico della parabola e l'area sottesa dal grafico di .
Pij88u9k
Ci serve però l'intervallo su cui calcolare i due integrali.

Per semplificarci la vita, osserviamo che quelle considerate sono funzioni pari, cioè simmetriche rispetto all'asse dell'ascisse. Infatti

petanto possiamo cercare l'ascissa dell'intersezione tra i due grafici limitandoci al semiasse delle ascisse positive, in cui
possiamo scrivere la funzione nella forma

e risolvendo l'equazione

otteniamo , ossia che ha soluzioni . Scartiamo la soluzione negativa e


consideriamo solamente quella positiva: .

Morale della favola: l'intervallo su cui dobbiamo calcolare i due integrali è . Dobbiamo calcolare

Sfruttiamo la già citata parità delle funzioni, così da poter calcolare l'integrale tra e

135
eBook edito da YouMath.it acquistato da Navdeep Singh
YouMath.it

Nell'intervallo la variabile è non negativa, e per definizione di valore assoluto si ha che , dunque l'integrale diventa

e l'esercizio è concluso: l'area della parte di piano a cui siamo interessati è .

Esercizio 8. Si considerino i grafici delle funzioni continue

sull'intervallo . Si calcoli l'area della regione piana costituita da tutti i punti del piano la cui ascissa cade nell'intervallo
e laPij88u9k
cui ordinata soddisfa la condizione , ossia della regione tra i grafici delle due funzioni .

Svolgimento: abbiamo le due funzioni polinomiali

che sono definite su tutto l'asse reale, ma l'esercizio chiede di concentrarci sull'intervallo . L'ordinata deve essere inoltre
compresa tra , quindi dobbiamo vedere per quali valori di si ha che:

Dobbiamo pertanto risolvere la disequazione

Applichiamo la regola di Ruffini per scomporre il polinomio al primo membro, così da ottenere

A questo punto inizia il panico! Come affrontiamo questa disequazione? Ci dobbiamo concentrare sull'insieme e in questo
intervallo il polinomio è sempre positivo perché somma di numeri positivi, dunque il prodotto è negativo o
nullo se e solo se il primo fattore è negativo o nullo ossia

Possiamo asserire che . Per determinare l'area di faremo riferimento al significato geometrico dell'integrale, con cui
avremo

136
Navdeep Singh - navdeep99@outlook.it - 146.241.52.118 - 2020-01-04 22:31:14 r1t2e3m4u5k cikelpo
YouMath.it

Pertanto l'area è e l'esercizio è concluso.

Esercizio 9. Calcolare la misura della porzione di piano compresa tra la curva di equazione e l'asse delle
ascisse, con .

Svolgimento: per risolvere questo tipo di esercizi dobbiamo usare l'interpretazione geometrica di integrale definito, e detto in altri
termini dobbiamo semplicemente calcolare l'integrale

Il valore assoluto è necessario perché il nostro scopo è quello di determinare l'area e affinché ciò sia possibile dobbiamo richiedere
che la funzione integranda sia non negativa.

Grazie alla definizione di valore assoluto inoltre si ha che

Osserviamo comunque che il primo ramo della funzione si riscrive come e poiché coincide con il secondo possiamo
Pij88u9k
scrivere che

Sostituiamo nell'integrale di partenza e risolviamolo con il metodo di integrazione per parti

Scegliamo come fattore finito e come fattore differenziale e mediante la formula di integrazione per
parti

L'esercizio è terminato.

Esercizio 10. Trovare in modo tale che sia pari a l'area sottesa tra il grafico della seguente funzione e compresa tra le rette
:

137
eBook edito da YouMath.it acquistato da Navdeep Singh
YouMath.it
Svolgimento: ricordiamo che data una funzione continua , per trovare l'area sottesa tra il grafico di e le rette di equazione
con , è sufficiente risolvere il seguente integrale

che fornisce proprio l'area cercata. Nel caso in esame, poiché la funzione è una funzione non
negativa per ogni allora l'area è data da

Il valore assoluto non serve proprio per la positività della funzione (è somma di funzioni positive). Grazie alla linearità dell'integrale
possiamo scrivere

che sono entrambi integrali immediati

Imponiamo che il valore ottenuto sia uguale a , in questo modo determineremo tale per cui

Pij88u9k

da cui . L'esercizio è completato.

ESERCIZI SUL VOLUME DI SOLIDI DI ROTAZIONE CON GLI


INTEGRALI
Esercizio 1. Calcolare il volume del solido generato dalla rotazione intorno all'asse della porzione del primo quadrante limitato
dalla parabola , dall'asse e dalla retta .

Svolgimento: prima di iniziare lo svolgimento dell'esercizio, forniamo un piccolo riassunto teorico. Consideriamo una funzione
continua nell'intervallo e sia la parte di piano limitata dal grafico della funzione, dall'asse e dalle rette
allora il volume del solido generato dalla rotazione di attorno all'asse è dato dall'integrale definito

Ora dedichiamoci all'esercizio: sappiamo che la parte di piano è limitata dalla parabola con asse parallelo all'asse e dalla retta
. Dall'equazione della parabola segue che . Perché ciò è vero? Semplice: l'equazione
ha senso nel momento in cui sia il primo che il secondo membro sono concordi (hanno lo stesso segno), e poiché è
positivo o nullo allora anche il secondo membro lo è.

Sempre dall'equazione estraiamo la radice quadrata membro a membro e otteniamo . Poiché siamo nel
primo quadrante pertanto l'equazione diventa e rappresenta la nostra funzione . La parte di piano che
ruotando genera il solido di rotazione è quella limitata dalla funzione e dalle rette .

138
Navdeep Singh - navdeep99@outlook.it - 146.241.52.118 - 2020-01-04 22:31:14 r1t2e3m4u5k cikelpo
YouMath.it
Per calcolare il volume del solido generato dalla rotazione utilizziamo la formula scritta prima

L'esercizio è terminato.

Esercizio 2. Il volume del solido di rotazione dell'area delimitata da , rispetto all'asse vale:

Svolgimento: per trovare il volume del solido generato dalla rotazione attorno all'asse dell'area delimitata da
, scriviamo in funzione di :

Estraendo membro a membro la radice quadrata otteniamo

Pij88u9k
e poiché la definizione di valore assoluto assicura che pertanto

Ora si tratta di calcolare l'integrale

ossia

che, molto semplicemente, vale

Il volume del solido è pertanto .

Esercizio 3. Calcolare il volume del solido ottenuto dalla rotazione della parte di piano delimitata dal grafico della funzione
e dalla retta intorno all'asse delle .

Svolgimento: nel caso in esame dobbiamo calcolare il volume del solido individuato dalla rotazione dell'area sottesa dal grafico
della funzione (il cui grafico è una parabola) sull'intervallo d'ascisse attorno all'asse delle .

139
eBook edito da YouMath.it acquistato da Navdeep Singh
YouMath.it
Per calcolarlo è sufficiente applicare la seguente formula:

dove è l'intervallo sul quale la funzione sottende l'area e è l'espressione analitica della funzione. Nel nostro caso:

Il volume del solido di rotazione attorno all'asse è .

Esercizio 4. Calcolare il volume del solido ottenuto ruotando attorno all'asse la regione di piano limitata dalla funzione
e dall'asse .

Svolgimento: la funzione ha per dominio e il suo grafico è una semicirconferenza di centro


e raggio che si trova nel semipiano dell'ordinate non negative. Poiché la rotazione avviene attorno all'asse delle
allora il volume è dato da

Pij88u9k

Osserviamo che il volume è in effetti quello della sfera di raggio .

Esercizio 5. Calcolare il volume del solido ottenuto dalla rotazione completa attorno all'asse della regione di piano delimitata
dal grafico della funzione per .

Svolgimento: è sufficiente utilizzare la formula per i volumi dei solidi di rotazione attorno all'asse , ossia

Nel caso in questione, mentre conseguentemente il volume è dato da

L'esercizio è concluso.

140
Navdeep Singh - navdeep99@outlook.it - 146.241.52.118 - 2020-01-04 22:31:14 r1t2e3m4u5k cikelpo
YouMath.it
Esercizio 6. Calcolare il volume del solido generato dalla rotazione completa attorno all'asse del trapezoide individuato dal
grafico della funzione nell'intervallo .

Svolgimento: il solido è generato dalla rotazione attorno all'asse delle pertanto interverrà la formula

dove nel nostro caso così che l'integrale diventi

L'integrale ottenuto può essere risolto per parti scegliendo come fattore finito e come fattore differenziale il
fattore nascosto

Pij88u9k

Non resta che effettuare le dovute sostituzioni:

Il risultato ottenuto è effettivamente il volume del solido di rotazione.

ESERCIZI SUL CALCOLO DEGLI INTEGRALI IMPROPRI


Esercizio 1. Calcolare il seguente integrale improprio di prima specie utilizzando la definizione

Svolgimento: siamo di fronte ad un integrale improprio di prima specie, infatti è definita su un intervallo di
integrazione illimitato quale è appunto .

Aggiungiamo inoltre che è continua in e di conseguenza è integrabile secondo Riemann in ogni intervallo chiuso e
limitato di . Per definizione di integrale improprio di prima specie abbiamo che

Mediante la definizione, ci siamo ricondotti al limite di un integrale definito che possiamo calcolare in modo standard

141
eBook edito da YouMath.it acquistato da Navdeep Singh
YouMath.it

Valutiamo la primitiva agli estremi di integrazione

e passiamo al limite per

In definitiva

di conseguenza possiamo concludere che l'integrale improprio converge e l'esercizio è terminato.

Esercizio 2. Calcolare l'integrale improprio

Svolgimento: quello proposto è un integrale improprio di prima specie perché l'intervallo di integrazione è illimitato e la
funzione integranda è addirittura continua in perché composizione di funzioni continue.

Pij88u9k
La continuità di in assicura l'integrabilità della stessa in ogni intervallo chiuso e limitato contenuto in , detto
in altri termini la funzione integranda è localmente integrabile in .

Per definizione di integrale improprio di prima specie

possiamo vedere l'integrale improprio come un limite di un integrale definito con un estremo variabile . Risolviamo l'integrale
definito osservando che il numeratore è quasi la derivata del denominatore (manca solo un ). Moltiplichiamo e dividiamo per così
otteniamo

Non ci resta che valutare la primitiva agli estremi e calcolare il limite

In conclusione, possiamo asserire che l'integrale improprio diverge positivamente.

Esercizio 3. Mostrare che l'integrale improprio

142
Navdeep Singh - navdeep99@outlook.it - 146.241.52.118 - 2020-01-04 22:31:14 r1t2e3m4u5k cikelpo
YouMath.it

mediante la definizione.

Svolgimento: quello proposto è un integrale improprio di prima specie perché presenta un estremo infinito (o detto in altri termini
l'intervallo di integrazione è illimitato). Per definizione di integrale improprio di prima specie abbiamo che:

Siamo quindi passati dall'integrale improprio ad un limite di un integrale definito con variabile. Per il momento tralasciamo il
limite e calcoliamo l'integrale

che possiamo risolvere mediante la regola di integrazione per parti, scegliendo come fattore finito
e come fattore differenziale

Pij88u9k
Rimpiazziamo il risultato ottenuto in e facciamo tendere

Il risultato di tale limite è perché:

In particolare il limite è zero perché la funzione esponenziale tende a molto più velocemente rispetto alla potenza.

Concludiamo l'esercizio asserendo che l'integrale improprio converge a .

Esercizio 4. Calcolare il seguente integrale improprio e dire se converge o diverge

Svolgimento: ci troviamo di fronte ad un integrale improprio di seconda specie, infatti l'intervallo di integrazione è limitato

mentre la funzione è illimitata nell'intervallo dato: lo si può evincere dal fatto che

143
eBook edito da YouMath.it acquistato da Navdeep Singh
YouMath.it

Calcoliamo l'integrale procedendo con la definizione

Ci siamo quindi ricondotti ad un limite di un integrale definito con estremo inferiore variabile. Risolviamo l'integrale con il metodo di

sostituzione, ponendo da cui il nuovo differenziale è . Attenzione agli estremi di integrazione:

­ se allora ;

­ se allora .

L'integrale diventa

e dunque possiamo concludere che l'integrale di partenza è divergente.

Esercizio 5. Calcolare l'integrale improprio


Pij88u9k

Svolgimento: l'integrale

è improprio di prima specie perché l'intervallo di integrazione è illimitato e la funzione integranda è continua in .
Utilizziamo la definizione di integrale improprio di prima specie per risolvere l'esercizio.

Risolviamo l'integrale indefinito il quale ci fornirà almeno una primitiva che verrà in seguito valutata agli estremi. Possiamo ricorrere
al metodo dei fratti semplici per calcolare

Ricerchiamo dunque quattro costanti reali tali che l'integranda si possa esprimere come somma di frazioni più
semplici da integrare

Portiamo tutto a denominatore comune

144
Navdeep Singh - navdeep99@outlook.it - 146.241.52.118 - 2020-01-04 22:31:14 r1t2e3m4u5k cikelpo
YouMath.it

Semplifichiamo i denominatori membro a membro ed eseguiamo i prodotti così da ottenere

Ora non ci resta che raccogliere secondo le potenze di

Affinché l'uguaglianza sia identicamente verificata deve essere:

Il sistema lineare si può risolvere agevolmente per sostituzione ed ha come soluzione

di conseguenza

e quindi l'integrale indefinito diventa

Pij88u9k

Grazie alle proprietà delle potenze possiamo scrivere

e svolgendo i calcoli

È ora la volta dell'integrale definito

Sostituiamo gli estremi

Perfetto, ora non ci resta che far tendere e determinare il valore dell'integrale improprio di partenza

145
eBook edito da YouMath.it acquistato da Navdeep Singh
YouMath.it

Osserviamo che l'addendo tende a per perché il denominatore è un infinito di ordine

superiore a quello del numeratore.

Possiamo concludere conseguentemente che l'integrale improprio converge a .

Esercizio 6. Calcolare l'integrale improprio

Svolgimento: abbiamo a che fare con un integrale improprio di prima specie, infatti il dominio di integrazione e la funzione

è continua ivi continua.

Calcoliamo esplicitamente il valore dell'integrale mediante la definizione di integrale improprio

Per il momento tralasciamo il limite e concentriamoci sull'integrale

e risolviamolo tramite il metodo di sostituzione per gli integrali ponendo

Pij88u9k

Non dimentichiamoci di modificare anche gli estremi di integrazione che in base alla sostituzione fatta diventano:

­ se allora

­ se allora

L'integrale diventa

A questo punto procediamo per fratti semplici, fattorizzando il denominatore della funzione integranda ossia
e a ciascun fattore che compare nella fattorizzazione associamo un fratto semplice. In particolare:

mentre

Dobbiamo ora determinare le costanti di modo che:

146
Navdeep Singh - navdeep99@outlook.it - 146.241.52.118 - 2020-01-04 22:31:14 r1t2e3m4u5k cikelpo
YouMath.it

Portiamo a denominatore comune, sviluppiamo i conti e raccogliamo secondo le potenze di :

Semplifichiamo membro a membro i denominatori, i quali hanno concluso il loro compito:

Il principio di identità dei polinomi permette di costruire il sistema lineare

da cui otteniamo e l'integrale da risolvere diventa

Pij88u9k
Il primo integrale è immediato, il secondo un po' meno ma raccogliendo al denominatore e usando la proprietà delle potenze
possiamo ricondurci all'integrale notevole che dà come risultato un arcotangente

Moltiplichiamo e dividiamo per così da avere al numeratore dell'integranda rimasta la derivata di

L'integrale rimasto è in realtà un'arcotangente pertanto

Per concludere l'esercizio dobbiamo far tendere , pertanto

147
eBook edito da YouMath.it acquistato da Navdeep Singh
YouMath.it

L'ultima cosa da fare è effettuare qualche passaggio algebrico per "abbellire il risultato"

Poiché il limite è finito allora possiamo concludere che l'integrale di partenza converge.

Esercizio 7. Calcolare il seguente integrale improprio

Svolgimento: quello proposto è un integrale improprio di prima specie, infatti l'intervallo di integrazione è illimitato mentre la funzione
integranda è continua in esso

Risolviamo l'integrale definito al variare di , utilizzando la regola di integrazione per parti prendendo come fattore finito

e come fattore differenziale . Grazie alla

formula di integrazione per parti otteniamo che


Pij88u9k

L'integrale rimasto può essere risolto mediante il metodo dei fratti semplici, che possiamo innescare subito giacché il denominatore
dell'integranda è già scomposto.

Al fattore associamo la frazione , mentre al fattore associamo la frazione . Il nostro obiettivo diventa quello di

determinare le costanti reali di modo che

Portiamo tutto a denominatore comune

Eseguiamo i conti e raccogliamo secondo le potenze di

148
Navdeep Singh - navdeep99@outlook.it - 146.241.52.118 - 2020-01-04 22:31:14 r1t2e3m4u5k cikelpo
YouMath.it

Semplifichiamo il denominatore che tanto è uguale membro a membro

Interviene in nostro soccorso il principio di identità dei polinomi con cui possiamo costruire il sistema lineare

da cui si ottiene facilmente che e pertanto l'integrale da risolvere diventa

Osserviamo che il valore assoluto non serve quando ha per argomento , giacché questa quantità è certamente positiva.
Continuiamo con il calcolo di questo integrale sostituendo in modo ordinato gli estremi

Pij88u9k

Grazie alle proprietà di cui godono i logaritmi, tale espressione diventa

Finalmente abbiamo gli strumenti per ricomporre l'integrale di partenza

e passando al limite avremo che

149
eBook edito da YouMath.it acquistato da Navdeep Singh
YouMath.it

Osserviamo che il calcolo del limite non è così terribile come si potrebbe pensare infatti:

e questo è facile. Il secondo invece potrebbe creare più di qualche grattacapo

Per le proprietà delle radici possiamo scrivere la radice del prodotto come prodotto delle radici e tenendo a mente la relazione
possiamo scrivere

Finalmente abbiamo concluso questo esercizio piuttosto elaborato: l'integrale improprio fornito dalla traccia converge.

Pij88u9k

è il risultato dell'integrale.

ESERCIZI SULLA CONVERGENZA DEGLI INTEGRALI


IMPROPRI DI PRIMA SPECIE
Attenzione: gli esercizi svolti sugli integrali impropri di prima specie sono esercizi in cui bisogna effettuare lo studio della
convergenza/divergenza senza ricorrere alla definizione, semplicemente perché l'uso della stessa potrebbe essere un metodo non
conveniente o nei peggiori dei casi non applicabile.

Alla luce di ciò conviene di gran lunga procedere con l'applicazione dei criteri di convergenza per gli integrali impropri di prima
specie, i quali sono oggetto di studio approfondito nei corsi universitari di Analisi 1.

Esercizio 1. Studiare la convergenza dell'integrale improprio

Svolgimento: per studiare correttamente la convergenza di un integrale improprio, bisogna prima di tutto riconoscere quali sono i
punti problematici. L'intervallo di integrazione è ed è illimitato, inoltre la funzione integranda

è ivi continua pertanto è integrabile in ogni sottointervallo chiuso e limitato contenuto in . Queste informazioni ci
permettono di concludere che l'integrale proposto è di prima specie.

150
Navdeep Singh - navdeep99@outlook.it - 146.241.52.118 - 2020-01-04 22:31:14 r1t2e3m4u5k cikelpo
YouMath.it

Studiare la convergenza di questo integrale con la definizione di integrale improprio è sconveniente in questo caso perché non è
possibile determinare esplicitamente una primitiva della funzione integranda, o detta in altri termini, le primitive dell'integranda non
sono esprimibili per mezzo di funzioni elementari. In questi casi è utile far affidamento ai criteri di convergenza degli integrali
impropri, quali: il criterio del confronto, il criterio del confronto asintotico o ancora il criterio di convergenza assoluta.

Fatta questa premessa, possiamo cominciare con l'analisi: utilizziamo il criterio del confronto asintotico per mostrare la convergenza
dell'integrale. Per poterlo innescare senza problemi, il criterio del confronto asintotico richiede che la funzione integranda sia
continua e positiva nell'intervallo di integrazione.

Ora che sia continua, questo è evidente, infatti è composizione di funzioni continue, così come è evidente il fatto che sia positiva,
osserviamo infatti che è rapporto di quantità positive.

Il prossimo passo consiste nel determinare una funzione positiva e asintoticamente equivalente a per .
Osserviamo che per valgono le seguenti equivalenze asintotiche:

mentre

possiamo infatti trascurare perché è un infinito. In definitiva, per le proprietà delle equivalenze asintotiche si ha che:

Abbiamo così determinato una funzione che è asintoticamente equivalente alla funzione per . Se

l'integrale
Pij88u9k

converge allora convergerà anche l'integrale di partenza. Osserviamo a questo punto che per la funzione esponenziale vale la
seguente disuguaglianza

Passando ai reciproci e invertendo il verso della disequazione otteniamo

e dunque per il teorema del confronto (semplice) si ha che

L'ultimo è un integrale improprio di prima specie notevole che sappiamo essere convergente pertanto possiamo concludere che

converge per il criterio del confronto così come converge

per il criterio del confronto asintotico.

151
eBook edito da YouMath.it acquistato da Navdeep Singh
YouMath.it

Esercizio 2. Dimostrare che l'integrale improprio

diverge.

Svolgimento: quello proposto è un integrale improprio di prima specie, infatti la funzione integranda è continua nell'intervallo
illimitato . La continuità della funzione integranda assicura la sua integrabilità in ogni sottointervallo chiuso e limitato di
. Per studiare la convergenza/divergenza dell'integrale utilizzeremo il criterio del confronto asintotico, stando attenti al fatto
che la funzione integranda non rispetta ancora le sue pretese, essa infatti non è positiva in .

Facciamoci furbi ed utilizziamo una proprietà dell'integrale

Cosa abbiamo fatto? Semplice! Abbiamo cambiato segno all'integranda trasportando il segno negativo fuori dal simbolo di
integrazione, rende e naturalmente se l'integrale al secondo membro diverge, divergerà anche l'integrale al primo membro.

Studiamo il comportamento asintotico della funzione integranda per . In un intorno di abbiamo che

La funzione asintotica a quella data è di conseguenza ma

Pij88u9k

è un integrale improprio fondamentale di prima specie notoriamente divergente, pertanto l'integrale di partenza diverge per il criterio
del confronto asintotico.

Esercizio 3. Studiare la convergenza dell'integrale improprio di prima specie

senza utilizzare la definizione.

Svolgimento: dobbiamo studiare la convergenza dell'integrale improprio

senza risolverlo esplicitamente ma con l'ausilio dei criteri di convergenza degli integrali impropri di prima specie. Nel caso in
questione, il criterio che ci permette di giungere velocemente alla soluzione dell'esercizio è il criterio del confronto asintotico.

Osserviamo preliminarmente che:

­ l'intervallo di integrazione è illimitato;

­ la funzione integranda è sia continua, perché composizione di funzioni continue, sia positiva nell'intervallo considerato perché
quoziente di funzioni positive.

Il criterio del confronto asintotico può essere quindi applicato. Il nostro intento è ora quello di determinare una funzione positiva
che sia asintoticamente equivalente alla funzione

152
Navdeep Singh - navdeep99@outlook.it - 146.241.52.118 - 2020-01-04 22:31:14 r1t2e3m4u5k cikelpo
YouMath.it

per

Se è integrabile lo sarà anche la funzione . Le stime asintotiche non sono però facili da determinare, bisogna avere un
po' di malizia matematica e un occhio clinico. Nel caso in questione per valgono le seguenti stime:

infatti le costanti additive possono essere trascurate se accostate a degli infiniti;

infatti è un infinito di ordine superiore rispetto a ecco perché questo addendo viene trascurato.

Grazie a queste considerazioni possiamo asserire che:

dove nell'ultima uguaglianza è intervenuta la proprietà sul quoziente di due potenze aventi stessa base.

La funzione , asintoticamente equivalente alla funzione è . Il criterio del confronto

asintotico assicura che

ha lo stesso comportamento dell'integrale dato in partenza e lo studio del suo comportamento è molto più semplice da effettuare.
Qui si aprono due strade entrambe molto valide: le percorreremo entrambe perché presentano spunti interessanti e ragionamenti
chePij88u9k
si possono riciclare in altri esercizi.

La prima consiste nel calcolare esplicitamente l'integrale improprio mediante la definizione

Poiché l'integrale improprio converge allora convergerà anche l'integrale di partenza per il criterio del confronto asintotico.

La seconda strada consiste nell'utilizzare il criterio del confronto, osservando che è "definitivamente" maggiore o uguale a ,
ossia esiste tale che

Non importa quale sia il valore esatto di , l'importante è sapere che esiste. Passiamo ora ai reciproci membro a membro,
ricordandoci di cambiare verso della disuguaglianza

Per il criterio del confronto si ha che

153
eBook edito da YouMath.it acquistato da Navdeep Singh
YouMath.it

e poiché l'integrale a secondo membro è convergente (è un integrale improprio notevole con ) allora anche l'integrale al
primo membro converge, inoltre, ricordiamolo, l'integrale di partenza ha lo stesso comportamento di quest'ultimo, pertanto possiamo
concludere che l'integrale di partenza converge.

Esercizio 4. Studiare la convergenza dell'integrale improprio

Svolgimento: siamo in presenza di un integrale improprio di prima specie perché l'intervallo di integrazione è illimitato e la

funzione è continua in esso.

La funzione integranda tra le altre cose è a segno variabile pertanto il criterio del confronto e il criterio del confronto asintotico non
possono essere utilizzati. Fortunatamente abbiamo a disposizione il criterio di convergenza assoluta:

Se

converge allora converge anche l'integrale improprio

Studiamo dunque la convergenza dell'integrale


Pij88u9k

in cui questa volta la funzione integranda ha segno positivo per via del valore assoluto: possiamo conseguentemente utilizzare i
criteri di convergenza in cui è richiesta la positività della funzione. In questo caso è molto utile utilizzare il criterio del confronto.

Grazie alla disuguaglianza triangolare e alle disuguaglianze notevoli

possiamo scrivere la seguente doppia disuguaglianza

pertanto

Osserviamo che abbiamo potuto eliminare il valore assoluto al denominatore perché nell'intervallo di
integrazione.

Se riusciamo a dimostrare la convergenza dell'integrale

l'esercizio è praticamente concluso. Ci viene in soccorso il criterio del confronto asintotico e infatti per sussiste la stima
asintotica

154
Navdeep Singh - navdeep99@outlook.it - 146.241.52.118 - 2020-01-04 22:31:14 r1t2e3m4u5k cikelpo
YouMath.it

questo perché al denominatore è un infinito di ordine superiore rispetto ad che può essere pertanto trascurato. Ora
ragioniamo a cascata: poiché

converge (è un integrale improprio di prima specie notevole) allora converge anche per il criterio del

confronto asintotico e di conseguenza converge l'integrale di partenza per il criterio di convergenza assoluta.

Esercizio 5. Studiare il seguente integrale improprio

con .

Svolgimento: abbiamo a che fare con un integrale improprio di prima specie di cui possiamo studiare la convergenza nell'intervallo
in due modi differenti.

Il primo e il più veloce, prevede di ricorrere al teorema del confronto per integrali di prima specie e di determinare un'integranda che:

­ maggiori , o che comunque maggiori in un intorno di ;

­ dia luogo ad un integrale improprio convergente su .

Pij88u9k
Il secondo consiste nel calcolare l'integrale improprio mediante la definizione stessa. Non è sempre possibile, perché spesso capita
di avere a che fare con integrande che non ammettono una primitiva esprimibile in termini di funzioni elementari, ma nel nostro caso
è fattibilissimo, infatti:

Possiamo concludere che l'integrale improprio converge.

Esercizio 6. Utilizzare il criterio del confronto asintotico per stabilire il comportamento dell'integrale improprio

Svolgimento: l'integrale proposto è in effetti un integrale improprio di prima specie perché il dominio di integrazione è illimitato e la
funzione integranda è continua perché composizione di funzioni continue in .

Prima di applicare il criterio del confronto asintotico, dobbiamo semplificare l'espressione dell'integrale. Grazie a due note proprietà
dei logaritmi possiamo riscriverlo come

155
eBook edito da YouMath.it acquistato da Navdeep Singh
YouMath.it

e poi dividendo termine a termine come

Ora possiamo fare affidamento al criterio del confronto asintotico: per risulta che

Basta fare riferimento all'equivalenza asintotica associata al limite notevole del logaritmo in forma generale

Possiamo pertanto asserire che in un intorno di l'integranda soddisfa la seguente equivalenza asintotica

Pij88u9k

Da qui possiamo concludere direttamente, per confronto asintotico, che l'integrale improprio di partenza converge perché ha lo
stesso comportamento

che è un integrale improprio notevole convergente.

Esercizio 7. Studiare la convergenza dell'integrale improprio

Svolgimento: quello proposto è un integrale improprio di prima specie perché l'intervallo di integrazione è illimitato. Dopo aver
osservato che la funzione è continua su un qualsiasi intervallo della forma , possiamo procedere in due modi:

1. studiamo la convergenza dell'integrale improprio senza fare ricorso alla definzione, il che vuol dire "senza calcolarlo
esplicitamente". Questo metodo richiede un filino in più di esperienza, ma è molto molto veloce;

2. calcoliamo l'integrale improprio mediante la definizione. Ciò all'atto pratico consiste nel riscrivere l'integrale come il limite di un
integrale definito.

Se il valore assoluto crea qualche problema nel calcolo, possiamo ricorrere alla definizione di modulo e sfruttare una nota proprietà
degli integrali impropri: l'additività sugli estremi.

156
Navdeep Singh - navdeep99@outlook.it - 146.241.52.118 - 2020-01-04 22:31:14 r1t2e3m4u5k cikelpo
YouMath.it
Espandiamo il valore assoluto

ossia, molto semplicemente

Possiamo allora riscrivere l'integrale come segue

Il primo integrale non dipende da e possiamo "trasportarlo fuori" dal limite

A conti fatti, dobbiamo risolvere due integrali ma se osserviamo veramente bene le due integrande ci accorgiamo che in realtà sono
opposte, e dunque sarà sufficiente calcolare una primitiva di una e aggiustare un segno per ottenere poi una primitiva per l'altra.

Proponiamoci di calcolare l'integrale indefinito

Pij88u9kper parti scegliendo come fattore finito


procedendo da cui e come fattore differenziale

da cui si ottiene che . Mediante la formula di integrazione per parti otteniamo

ed eseguendo le dovute semplificazioni scriviamo

L'integrale rimasto può essere affrontato usando nuovamente il metodo di integrazione per parti scegliendo come fattore finito

da cui e come fattore differenziale da cui pertanto

L'ultimo integrale è immediato

e mediante delle manipolazioni algebriche scriviamo finalmente il risultato

157
eBook edito da YouMath.it acquistato da Navdeep Singh
YouMath.it

Osserviamo che grazie alle proprietà degli integrali indefiniti possiamo inoltre scrivere

Ora possiamo tornare all'integrale improprio

Il limite rimasto è zero perché al denominatore abbiamo un esponenziale che è un infinito di ordine superiore rispetto al polinomio al
numeratore.

Pij88u9k
Abbiamo ottenuto un valore finito e dunque possiamo concludere che l'integrale improprio di partenza converge.

Esercizio 8. Studiare il comportamento dell'integrale improprio

con .

Svolgimento: ci sono due modi per rispondere alla richiesta dell'esercizio:

­ il primo consiste nel dare una risposta diretta, previa conoscenza degli integrali impropri notevoli. Parlando a denti stretti questo è
da considerarli un integrale notevole convergente.

­ il secondo consiste nel calcolare esplicitamente una primitiva per la funzione integranda e calcolare l'integrale con la definizione di
integrale improprio di prima specie.

Al variare di risolviamo l'integrale definito mediante il metodo di sostituzione ponendo a cui associamo il

differenziale . Naturalmente dobbiamo modificare in modo appropriato anche gli estremi di integrazione: ad

associamo , mentre ad associamo

L'integrale diventa

158
Navdeep Singh - navdeep99@outlook.it - 146.241.52.118 - 2020-01-04 22:31:14 r1t2e3m4u5k cikelpo
YouMath.it

Poiché il limite è uguale ad un valore finito l'integrale improprio converge.

Esercizio 9. Stabilire se il seguente integrale converge

Svolgimento: iniziamo col dire che l'integrale proposto non si esprime con funzioni elementari quindi non potremo studiare il suo
comportamento con la definizione: dovremo ricorrere necessariamente ai criteri di convergenza per gli integrali impropri di prima
specie.

La funzione integranda è

che è chiaramente una funzione continua nell'intervallo considerato perché composizione di funzioni continue. Dobbiamo
analizzarne anche il segno così controlliamo se è possibile utilizzare il criterio del confronto asintotico.
Pij88u9k
Il denominatore della funzione è certamente positivo nell'intervallo di integrazione perché somma di quantità positive, ma il
numeratore? Facciamo alcune considerazioni: sappiamo che la funzione seno è limitata da , ossia

Per , moltiplichiamo membro a membro per così da ottenere

Il verso della disuguaglianza non si inverte perché se a maggior ragione sarà maggiore di .

Sommiamo membro a membro

Osserva che la parte centrale è in sostanza il numeratore della funzione integranda, inoltre

Il numeratore della funzione integranda è positivo o nullo perché maggiorato da una funzione non negativa, di conseguenza l'intera
funzione integranda è positiva o nulla perché quoziente di quantità non negative.

La continuità e la positività della funzione forniscono il lascia passare per criterio del confronto asintotico.

Per , il numeratore della funzione integranda ha come stima

perché è un infinito di ordine superiore rispetto all'altro addendo che può essere trascurato, mentre:

159
eBook edito da YouMath.it acquistato da Navdeep Singh
YouMath.it

Naturalmente è un infinito per , invece è una costante additiva e in quanto tale può essere trascurata.

In definitiva

L'integrale di partenza ha lo stesso carattere dell'integrale

ma quest'ultimo è un integrale improprio notevole che non converge, dunque nemmeno l'integrale di partenza converge.

Esercizio 10. Stabilire se una data funzione è integrabile e assolutamente integrabile nell'intervallo

Notiamo innanzitutto che abbiamo a che fare con una funzione continua e definita su un intervallo illimitato , quindi avremo
a che fare con un integrale improprio di prima specie.

Notiamo inoltre che la funzione data è non negativa in , infatti:

Pij88u9k

Queste relazioni ci assicurano la non negatività del numeratore perché possiamo vederlo come somma di quantità non negative. Il
denominatore è ovviamente positivo perché somma di due quadrati.

Ecco dunque un'ottima notizia: lo studio dell'integrabilità assoluta coincide con lo studio dell'integrabilità proprio perché la funzione
data è positiva.

Per studiare il comportamento dell'integrale improprio possiamo utilizzare inoltre il criterio del confronto asintotico.

Per valgono le seguenti stime:

pertanto

inoltre, poiché il logaritmo è un infinito di ordine inferiore rispetto a qualsiasi potenza con esponente positivo si ha che

Il logaritmo sparisce proprio perché la potenza lo batte all'infinito.

160
Navdeep Singh - navdeep99@outlook.it - 146.241.52.118 - 2020-01-04 22:31:14 r1t2e3m4u5k cikelpo
YouMath.it
Il denominatore è facile da analizzare

In definitiva la funzione integranda è asintotica a:

Per il criterio del confronto asintotico, l'integrale di converge se e solo se converge

ma questo è un integrale improprio notevole che diverge, quindi possiamo concludere che la funzione non è integrabile
sull'intervallo .

Esercizio 11. Determinare per quali valori del parametro reale l'integrale

converge.

Svolgimento: abbiamo a che fare con un integrale improprio di prima specie, infatti il dominio di integrazione è illimitato e

la funzione integranda è continua perché composizione di funzioni continue nell'intervallo dato. È evidente

che l'integranda non presenta alcun problema sull'intervallo e che dobbiamo fare riferimento solamente ad un intorno di
.
Pij88u9k
Nell'intorno di usiamo il criterio del confronto asintotico per integrali impropri di prima specie, e si vede che il numeratore è
asintotico ad una costante. In particolare, se

che, ribadiamolo, è una costante niente di più e niente di meno.

Per quanto concerne il denominatore, il valore del parametro è determinante per la convergenza/divergenza. Se abbiamo
infatti

perché è un infinito di ordine superiore ad per pertanto può essere trascurato. Quest'analisi ci conduce ad una stima
asintotica per :

e poiché l'integrale

converge per lo farà anche l'integrale improprio di partenza converge.

Se invece allora

infine se
161
eBook edito da YouMath.it acquistato da Navdeep Singh
YouMath.it

perché in questo caso è un infinito di ordine inferiore rispetto a .

In entrambi i casi, gli integrali impropri associati divergono e dunque divergerà anche l'integrale di partenza. In definitiva possiamo
concludere che

converge se e solo se mentre diverge se e solo se . Abbiamo concluso l'analisi.

ESERCIZI SULLA CONVERGENZA DEGLI INTEGRALI


IMPROPRI DI SECONDA SPECIE
Esercizio 1. Studiare la convergenza del seguente integrale improprio di seconda specie con integranda data da una funzione
razionale

Svolgimento: ci troviamo di fronte ad un integrale improprio di seconda specie perché l'intervallo di integrazione è limitato mentre la
funzione integranda è illimitata nell'intervallo dato.

Abbiamo due possibili strade da seguire per lo studio della convergenza:


Pij88u9k
­ procedere con la definizione;

­ usare qualche criterio di convergenza per gli integrali impropri di seconda specie e ricondurci ad un integrale improprio più
semplice, di cui studiare la convergenza con la definizione o sfruttando gli integrali impropri notevoli.

In questo caso è sconsigliabile usare la definizione perché i calcoli prendono molto tempo, è facile convincersene impostando
l'integrale; in ogni caso il secondo metodo è quello che permette sempre di risparmiare calcoli e fatica, dunque nettamente
preferibile. Iniziamo con l'analisi

Riscriviamo l'integrale scomponendo il denominatore, mediante la regola per la differenza di cubi

Ora cerchiamo di applicare il criterio del confronto asintotico per gli integrali impropri di seconda specie, dunque studiamo il
comportamento asintotico dell'integranda nell'intorno sinistro della singolarità .

Lasciamo perdere il coefficiente, dacché non incide in alcun modo sulla convergenza dell'integrale, e studiamo

Se questo integrale converge, allora convergerà anche l'integrale di partenza. Qui andiamo a razzo con la definizione

162
Navdeep Singh - navdeep99@outlook.it - 146.241.52.118 - 2020-01-04 22:31:14 r1t2e3m4u5k cikelpo
YouMath.it

Abbiamo a che fare con un integrale fondamentale

e con la valutazione agli estremi

dove il risultato viene dedotto dal comportamento della funzione logaritmica nell'intorno destro di zero. In definitiva, l'integrale
improprio diverge.

Esercizio 2. Determinare il valore del seguente integrale improprio

Pij88u9k

Svolgimento: la traccia chiede a tutti gli effetti di calcolare l'integrale improprio, ma prima di eseguire i calcoli che consentano di
giungere ad un risultato sarebbe opportuno analizzare il comportamento dell'integrale mediante i criteri di convergenza per gli
integrali impropri di seconda specie.

Perché è un integrale improprio di seconda specie? Perché l'intervallo di integrazione è limitato, in più la funzione integranda

è continua in ma non è limitata, infatti

Il punto che genera la singolarità, in questo caso, è l'estremo superiore di integrazione: . Possiamo, pertanto, lavorare
considerando la seguente equivalenza asintotica per

Osserviamo che possiamo razionalizzare la frazione moltiplicando e dividendo per così da ottenere

163
eBook edito da YouMath.it acquistato da Navdeep Singh
YouMath.it

e per vale l'equivalenza asintotica

Possiamo quindi asserire senza ombra di dubbio che per

Grazie al criterio del confronto asintotico tale osservazione ci riconduce allo studio della convergenza del seguente integrale
improprio che possiamo affrontare con la definizione

Pij88u9k

Questo integrale improprio diverge negativamente e per il criterio del confronto asintotico diverge negativamente anche l'integrale di
partenza.

Esercizio 3. Dire se il seguente integrale improprio converge o meno

Svolgimento: per capire se l'integrale improprio

converge o diverge, bisogna procedere fornendo una stima asintotica della funzione integranda nell'intorno dei punti singolari
contenuti nell'intervallo di integrazione.

I punti singolari di prima o terza specie non comportano problemi, quelli di seconda potrebbero e quindi vanno studiati in maniera
più approfondita.

La funzione initegranda è definita nell'estremo destro dell'integrazione , sicché ci limitiamo a considerare il punto .
Richiamiamo il limite notevole del coseno

164
Navdeep Singh - navdeep99@outlook.it - 146.241.52.118 - 2020-01-04 22:31:14 r1t2e3m4u5k cikelpo
YouMath.it

tale limite ci dice che, in un intorno di , sussiste la stima asintotica

e dunque possiamo equivalentemente studiare se converge o diverge nell'intorno di l'integrale improprio con funzione
integranda:

e poiché ci troviamo in un intorno destro di zero si ha che , e la precedente espressione diventa

D'altra parte, il termine tende a al tendere di e quindi, in riferimento al punto , la funzione integranda può essere

equivalentemente sostituita con .

L'integrale improprio di questa funzione diverge nell'intorno di per confronto con gli integrali impropri notevoli di seconda
specie.

Il criterio di convergenza asintotica ci assicura la divergenza dell'integrale fornito dalla traccia.

Pij88u9k
Esercizio 4. Studiare il comportamento del seguente integrale improprio

Svolgimento: quello proposto è un integrale improprio di seconda specie, infatti la funzione integranda

ha per dominio

ed è una funzione continua nel dominio perché composizione di funzioni continue. L'intervallo di integrazione proposto è , in
particolare:

pertanto possiamo asserire che la funzione data è continua e illimitata in .

Procediamo con lo studio dell'integrale improprio

utilizzando il metodo di sostituzione per gli integrali. Poniamo

165
eBook edito da YouMath.it acquistato da Navdeep Singh
YouMath.it
Gli estremi di integrazione diventano:

Dopo aver sostituito, l'integrale diventa:

e invece di risolvere l'integrale improprio con la definizione, possiamo pensare di utilizzare un criterio di convergenza che ci
permetta di concludere se appunto vi è convergenza oppure no. Il metodo del confronto asintotico per gli integrali impropri di
seconda specie fa al caso nostro.

Osserviamo che l'integranda è una funzione a segno costante nell'intervallo considerato, inoltre per da sinistra vale la stima
asintotica:

Pertanto l'integrale di partenza ha lo stesso carattere di

Per definizione di integrale improprio, si ha che l'integrale è

Pij88u9k

Questo integrale diverge negativamente e per il criterio del confronto asintotico divergerà negativamente anche l'integrale di
partenza.

Esercizio 5. Studiare il comportamento dell'integrale

senza utilizzare la definizione di integrale improprio.

Svolgimento: si tratta di un integrale improprio di seconda specie e per studiare la convergenza, osserviamo che la funzione
integranda si può scrivere nella forma

Abbiamo come estremi di integrazione proprio i due punti che rappresentano due singolarità di seconda specie della funzione.

Concentriamoci sull'estremo di integrazione , in cui il problema sorge dal fattore presente al denominatore.

È noto che, per gli integrali impropri notevoli, risulta che

166
Navdeep Singh - navdeep99@outlook.it - 146.241.52.118 - 2020-01-04 22:31:14 r1t2e3m4u5k cikelpo
YouMath.it

e questo ci permetterebbe di concludere, in un colpo solo, che l'integrale proposto diverge, grazie al confronto asintotico: in un
intorno sinistro di risulta infatti che

Questo sarebbe sufficiente per concludere che l'integrale proposto non converge. Per completezza, ragioniamo in modo del tutto
analogo in un intorno destro di , ottenendo che

e anche qui l'integranda è asintotica ad una funzione che ha integrale divergente. L'esercizio è concluso.

Esercizio 6. Stabilire il comportamento dell'integrale improprio

e in caso di convergenza calcolare il valore dell'integrale.

Svolgimento: ci troviamo di fronte ad un integrale improprio di seconda specie. Osserviamo infatti che l'intervallo di integrazione
è limitato, mentre per la funzione integranda tende a :

Pij88u9k

Questo assicura che l'integranda non è limitata nell'intervallo dato.

È facile inoltre determinare il segno della funzione integranda: essa è positiva perché quoziente di quantità positive, in più è
continua in perché composizione di funzioni continue. Utilizziamo il criterio del confronto asintotico per gli integrali impropri di
seconda specie:

Poiché è convergente giacché è un integrale improprio notevole con esponente allora anche l'integrale dato

dalla traccia converge.

A questo punto possiamo calcolare l'integrale improprio:

Calcoliamo l'integrale al variare di mediante il metodo di integrazione per sostituzione:

e gli estremi di integrazione diventano:

167
eBook edito da YouMath.it acquistato da Navdeep Singh
YouMath.it
Grazie alla sostituzione l'integrale diventa

L'integrale fornito dalla traccia vale dunque .

Esercizio 7. Studiare il comportamento dell'integrale improprio

Svolgimento: la funzione integranda è che è continua nell'intervallo oltre ad essere positiva.

Osserviamo inoltre che

Pij88u9k
è un integrale improprio di seconda specie, infatti l'intervallo di integrazione è limitato mentre la funzione integranda non è
limitata: sono infatti presenti due punti singolari che sono rispettivamente . Utilizziamo le proprietà degli integrali

e scriviamo l'integrale dato come somma di integrali aventi ognuno un solo punto problematico

A questo punto determiniamo una primitiva di risolvendo l'integrale indefinito

È di facile risoluzione perché è un integrale immediato. Ora possiamo risolvere i due integrali impropri mediante la definizione.
Iniziamo con il primo integrale improprio:

168
Navdeep Singh - navdeep99@outlook.it - 146.241.52.118 - 2020-01-04 22:31:14 r1t2e3m4u5k cikelpo
YouMath.it

Il secondo integrale improprio si risolve in maniera molto simile

Pij88u9k
In definitiva l'integrale di partenza è somma di integrali divergenti positivamente e dunque diverge positivamente.

Esempio 8. Dire se il seguente integrale improprio è convergente oppure no

Svolgimento: la funzione integranda dell'integrale proposto

è definita e continua nell'intervallo ma presenta delle singolarità nei punti e , inoltre è negativa nell'intervallo
perché quoziente di funzioni discordi. Osserviamo infatti che mentre per ogni .

Per determinare il comportamento dell'integrale improprio fornito dalla traccia, utilizziamo la proprietà additiva sugli estremi

in questo modo esprimiamo l'integrale improprio come somma di integrali impropri, ognuno dei quali ha una singolarità.

Sottolineiamo che è un valore arbitrariamente scelto: potevamo scegliere al suo posto un qualsiasi valore compreso tra ,

estremi esclusi.

Consideriamo il secondo integrale, ossia

169
eBook edito da YouMath.it acquistato da Navdeep Singh
YouMath.it

e analizziamo il comportamento della funzione integranda in un intorno sufficientemente piccolo di . Calcoliamo il limite

che può essere affrontato riconducendoci al limite notevole del logaritmo. Per fare ciò sommiamo e sottraiamo all'interno del
logaritmo

Poiché il limite esiste finito allora la funzione integranda è prolungabile con continuità nell'intorno di , di conseguenza

è convergente.

Dedichiamoci ora al primo integrale improprio, ossia

e studiamone il comportamento mediante il criterio di convergenza per gli integrali impropri di seconda specie. Per
sussistono le seguenti stime

Pij88u9k

di consegunza l'integranda è asintoticamente equivalente a per . Se converge convergerà anche

l'integrale .

Calcoliamo esplicitamente l'integrale, mediante la definizione di integrale improprio:

Procediamo con il metodo di integrazione per parti scegliendo come fattore finito e come fattore differenziale
, otteniamo

Abbiamo scoperto che il limite è finito pertanto l'integrale improprio converge e per il criterio del confronto asintotico converge anche

170
Navdeep Singh - navdeep99@outlook.it - 146.241.52.118 - 2020-01-04 22:31:14 r1t2e3m4u5k cikelpo
YouMath.it

ed è un'ottima notizia perché abbiamo espresso l'integrale di partenza come somma di integrali impropri convergenti e in quanto
tale convergente.

Esercizio 9. Studiare il comportamento del seguente integrale improprio

Svolgimento: consideriamo la funzione integranda

e osserviamo che rappresenta per essa un punto singolare, oltre al fatto che per ogni . Poiché sia il
seno che il coseno sono funzioni limitate si possono costruire le seguenti disuguaglianze

grazie alle quali possiamo utilizzare il criterio del confronto:

Pij88u9k

Se dimostriamo che l'ultimo integrale converge allora abbiamo la convergenza anche dell'integrale proposto.

Dedichiamoci dunque allo studio dell'integrale:

che possiamo affrontare mediante il criterio del confronto asintotico per integrali impropri di seconda specie. Il limite notevole del
coseno assicura la validità della seguente equivalenza asintotica per :

quindi sostituendo con la stima trovata otteniamo il seguente integrale

Ci siamo ricondotti ad un integrale improprio notevole convergente perché l'esponente .

171
eBook edito da YouMath.it acquistato da Navdeep Singh
YouMath.it

Possiamo concludere che convergerà anche:

e dunque anche l'integrale di partenza. L'esercizio è concluso.

Esercizio 10. Dire se il seguente integrale improprio converge

Svolgimento: osserviamo che la funzione integranda presenta due punti singolari che sono . Possiamo pensare di
utilizzare la proprietà additiva sugli intervalli così da esprimere l'integrale dato come somma di integrali impropri:

e se dimostriamo che entrambi gli integrali al secondo membro convergono allora convergerà anche l'integrale di partenza.

Mostriamo la convergenza dell'integrale improprio

mediante il criterio del confronto asintotico per gli integrali impropri di seconda specie, determinando una stima asintotica da
associare alla funzione integranda per . Sfruttiamo i limiti notevoli del logaritmo e del seno grazie ai quali possiamo costruire
Pij88u9kequivalenze asintotiche
le seguenti

mentre il termine . In definitiva possiamo concludere che

L'integrale converge perché è un integrale improprio notevole di seconda specie con esponente minore di e dunque

anche l'integrale converge.

Dedichiamo la nostra attenzione al secondo integrale

ed osserviamo che la singolarità si ha per questa volta. Utilizziamo ancora una volta il criterio del confronto asintotico
osservando che per valgono le seguenti stime

172
Navdeep Singh - navdeep99@outlook.it - 146.241.52.118 - 2020-01-04 22:31:14 r1t2e3m4u5k cikelpo
YouMath.it
e dunque possiamo asserire che:

Anche in questo caso il criterio del confronto asintotico ci assicura la convergenza.

In conclusione l'integrale improprio di partenza converge perché somma di integrali impropri convergenti.

Esercizio 11. Studiare il comportamento del seguente integrale improprio al variare del parametro

Svolgimento: quello proposto è un integrale improprio di seconda specie perché l'intervallo di integrazione è limitato e la

funzione integranda può non essere limitata agli estremi.

Osserviamo primariamente che la funzione integranda ha due punti singolari che sono : per facilitarci lo studio
sfruttiamo la proprietà addivita sull'intervallo di integrazione la quale ci permette di esprimere l'integrale dato come somma di due
integrali ognuno dei quali ha una sola singolarità:

Studiamo separatamente il comportamento dei due integrali partendo dal primo, ossia

Pij88u9k

Evidentemente, il punto problematico è e per vale la seguente stima asintotica

pertanto se converge allora convergerà anche .

Possiamo dimostrare la convergenza del seguente integrale con la definizione stessa di integrale improprio

Per definizione di valore assoluto abbiamo che pertanto l'integrale si scrive come

Procediamo per parti scegliendo come fattore finito il logaritmo e come fattore differenziale la funzione costante , otteniamo

173
eBook edito da YouMath.it acquistato da Navdeep Singh
YouMath.it

Orbene, il limite è finito e di conseguenza l'integrale improprio che stiamo studiando converge, indipendentemente dal valore di .

Occupiamoci ora del secondo integrale ossia

che scomponendo la differenza di quadrati presenti all'interno del valore assoluto si scrive come

Studieremo la convergenza di questo integrale con il criterio del confronto asintotico osservando che

dove per costruire l'equivalenza asintotica del logaritimo abbiamo fatto affidamento al suo limite notevole. Le relazioni appena scritte
ci dicono sostanzialmente che la funzione integranda per gode della seguente equivalenza asintotica
Pij88u9k

Nell'intorno sinistro di si ha che pertanto la funzione integranda è asintotica a .

Attenzione: è un integrale improprio notevole di seconda specie che converge se e solo se l'esponente

, mentre diverge se .

Possiamo concludere quindi che l'integrale dato dalla traccia converge se e solo se il parametro perché si esprime
come somma di integrali convergenti.

Esercizio 12. Stabilire il carattere del seguente integrale

Svolgimento: la funzione integranda dell'integrale proposto presenta un punto di singolarità per inoltre l'intervallo di
integrazione è limitato di conseguenza possiamo asserire che l'integrale

è improprio di seconda specie. Procediamo con il criterio del confronto asintotico per . Grazie al limite notevole
dell'esponenziale, abbiamo che

174
Navdeep Singh - navdeep99@outlook.it - 146.241.52.118 - 2020-01-04 22:31:14 r1t2e3m4u5k cikelpo
YouMath.it

In forza del teorema del confronto asintotico, e facendo riferimento agli integrali impropri notevoli di seconda specie, possiamo
concludere subito che l'integrale proposto converge.

ESERCIZI DI RIEPILOGO SULLA CONVERGENZA DEGLI


INTEGRALI IMPROPRI
Esercizio 1. Analizzare il comportamento dell'integrale improprio

Svolgimento: per determinare la convergenza/divergenza dell'integrale improprio proposto è sufficiente ricorrere al criterio del
confronto asintotico. Non serve calcolare una primitiva che tra l'altro richiederebbe una mole non indifferente di passaggi algebrici.

Facendo riferimento, agli integrali impropri notevoli, è sufficiente valutare il comportamento della funzione in un intorno di , che
è l'unico punto problematico.

Per sussistono le seguenti equivalenze asintotiche

La seconda stima asintotica si ottiene trascurando la costante perché è un infinito. Le due stime permetto di scrivere la
Pij88u9k
seguente equivalenza asintotica per l'integranda

Sottolineiamo che è un integrale improprio notevole di prima specie ed è convergente perché l'esponente è

maggiore di .

Il criterio del confronto asintotico ci permette di concludere che l'integrale dato dall'esercizio è convergente.

Esercizio 2. Studiare il carattere del seguente integrale improprio

Svolgimento: siamo di fronte ad un integrale improprio misto, nel senso che da un lato abbiamo un integrale improprio di prima
specie (l'intervallo di integrazione è illimitato), dall'altra un integrale improprio di seconda specie.

Studiamo la convergenza di

mediante il criterio del confronto asintotico, analizzando il comportamento asintotico dell'integranda negli intorni dei punti
problematici che sono e .

Per sussistono le seguenti equivalenze asintotiche

175
eBook edito da YouMath.it acquistato da Navdeep Singh
YouMath.it

pertanto per la funzione integranda vale l'equivalenza asintotica

Ciò assicura la convergenza nell'intorno di perché la funzione è asintotica a il cui integrale di prima specie associato è

appunto un integrale improprio notevole convergente.

Nell'intorno di sussiste invece la seguente equivalenza

ossia

Abbiamo convergenza perché l'integrale improprio di seconda specie associato alla stima
Pij88u9k è convergente.

Finalmente possiamo concludere che l'integrale di partenza converge.

Esempio 3. Studiare il comportamento dell'integrale improprio

Svolgimento: quello proposto è un integrale improprio di prima specie perché abbiamo a che fare con un integranda continua in un
intervallo illimitato.

Un modo per portare a casa l'esercizio consiste nello spezzare l'integrale in due parti mediante la proprietà additiva sugli intervalli:

Naturalmente se gli integrali al secondo membro convergono, allora convergerà anche l'integrale al primo membro: studiamo
separatamente dei due, cominciando da

Esso non è un integrale improprio perché abbiamo una funzione continua su intervallo chiuso e limitato, dunque converge
sicuramente.

L'analisi di avviene mediante il metodo del confronto. Per ogni si ha:

176
Navdeep Singh - navdeep99@outlook.it - 146.241.52.118 - 2020-01-04 22:31:14 r1t2e3m4u5k cikelpo
YouMath.it

e per la monotonia dell'integrale possiamo scrivere la seguente disuguaglianza doppia

Poiché

allora converge e grazie al criterio del confronto per gli integrali impropri di prima specie converge anche:

e di conseguenza anche l'integrale originale converge, perché somma di integrali convergenti.

Esercizio 4. Determinare il carattere del seguente integrale

Pij88u9k

Svolgimento: dobbiamo controllare la convergenza dell'integrale improprio

osservando che la funzione integranda è certamente positiva in , inoltre entrambi gli estremi di integrazione sono
problematici: uno è e l'altro è e annulla il denominatore della funzione.

Per determinare il comportamento, spezziamo l'integrale come somma di integrali

Se gli integrali al secondo membro convergono, allora converge anche l'integrale al primo.

Cominciamo con il secondo, ossia

Esso è un integrale improprio di seconda specie e per la funzione integranda in effetti vale

177
eBook edito da YouMath.it acquistato da Navdeep Singh
YouMath.it
In base ai criteri di convergenza per gli integrali impropri, poiché allora il secondo integrale improprio converge.

Se riusciamo a dimostrare che l'integrale improprio di seconda specie:

converge, l'esercizio è praticamente concluso. Il punto problematico è il primo estremo e per sussiste la seguente
equivalenza asintotica

Per i criteri di convergenza degli integrali impropri di seconda specie, l'integrale converge nell'intorno di .

Abbiamo espresso l'integrale di partenza come somma di integrali convergenti e di conseguenza converge.

Esercizio 5. Determinare il carattere del seguente integrale

Svolgimento: la prima cosa che salta all'occhio è che l'integrale proposto presenta due problematiche: una al primo estremo, valore
per il quale il denominatore dell'integranda si annulla, e una nel secondo estremo perché non è un numero finito.

In questo caso è conveniente spezzare l'integrale come somma di due integrali:

Pij88u9k

così facendo possiamo studiare i problemi dati dagli estremi singolarmente. Se entrambi gli integrali convergono, allora anche
l'integrale dato dalla traccia converge.

Cominciamo con il primo integrale che chiamiamo

di cui studiamo la convergenza mediante il criterio del confronto asintotico. Per sussiste l'equivalenza asintotica

e poiché converge possiamo concludere che converge anche l'integrale .

Consideriamo ora l'altro integrale:

La funzione integranda è tale che:

178
Navdeep Singh - navdeep99@outlook.it - 146.241.52.118 - 2020-01-04 22:31:14 r1t2e3m4u5k cikelpo
YouMath.it

e poiché converge allora convergerà anche .

L'integrale dato dalla traccia converge perché somma di integrali convergenti.

Esercizio 6. Dimostrare che il seguente integrale improprio converge

Svolgimento: questo è un integrale improprio molto particolare e richiede una strategia risolutiva diversa da quelle incontrate in
precedenza. Partiamo comunque dalla classificazione dell'integrale: esso è un integrale improprio di prima specie perché
l'integranda è una funzione continua in un intervallo illimitato.

Mediante la definizione di integrale improprio di prima specie possiamo scrivere

e grazie al metodo di integrazione per parti, scegliendo come fattore finito e come fattore differenziale

otteniamo:

Pij88u9k

Spezziamo ora il limite come somma di limiti

e per definizione di integrale improprio di prima specie si ha che

Osserviamo che

perché il termine per infatti il seno è una funzione limitata tra , e mentre il denominatore è un

infinito al tendere di .

Studiamo la convergenza dell'integrale

osservando preliminarmente che la funzione integranda è a segno variabile nell'intervallo dato: in questi casi è bene utilizzare il
criterio di convergenza assoluta, il quale ci assicura che se converge

179
eBook edito da YouMath.it acquistato da Navdeep Singh
YouMath.it

allora converge anche l'integrale . Poiché allora sussiste la seguente maggiorazione

ed inoltre sapendo converge (è un integrale improprio notevole di prima specie con esponente maggiore di )

scopriamo che l'integrale converge. Queste considerazioni ci permettono di concludere che l'integrale dato dalla traccia converge.

Esercizio 7. Determinare il caratttere del seguente integrale

Svolgimento: siamo di fronte ad un integrale improprio di seconda specie, infatti la funzione integranda è continua in ma

, ossia non è limitata nell'intervallo dato.

Procediamo con il criterio del confronto asintotico, osservando che per sussiste l'equivalenza asintotica

Pij88u9k
e poiché

è un integrale improprio notevole divergente possiamo concludere che anche l'integrale di partenza diverge.

Esercizio 8. Determinare il carattere dell'integrale

Svolgimento: siamo in presenza di un integrale improprio di seconda specie infatti la funzione integranda è continua nell'intervallo

ma e pertanto è illimitata nell'intervallo considerato.

Procediamo con il criterio del confronto asintotico per :

e la convergenza è garantita dal fatto che

è un integrale improprio notevole convergente di seconda specie.

180
Navdeep Singh - navdeep99@outlook.it - 146.241.52.118 - 2020-01-04 22:31:14 r1t2e3m4u5k cikelpo
YouMath.it
Esercizio 9. Dire se il seguente integrale converge oppure no

Svolgimento: l'integrale proposto è improprio e presenta problemi in , valore per cui si annulla il denominatore, e .
Procediamo spezzando l'integrale come somma di integrali, ognuno:

Studiamo la convergenza di entrambi, cominciando dal primo

Possiamo studiare il carattere di questo integrale con il criterio del confronto asintotico per gli integrali di seconda specie. Per
sussiste la seguente stima

e dunque

Pij88u9k
L'integrale converge perché ha lo stesso comportamento di

che è un integrale improprio notevole convergente perché del tipo

Consideriamo ora il secondo integrale, ossia

e osserviamo che per si ha che

Il criterio del confronto asintotico per gli integrali impropri di prima specie ci assicura che l'integrale converge perché converge

essendo quest'ultimo un integrale improprio notevole del tipo:

181
eBook edito da YouMath.it acquistato da Navdeep Singh
YouMath.it

L'integrale fornito dalla traccia converge perché somma di integrali convergenti.

Esercizio 10. Stabilire se il seguente integrale improprio è convergente

Svolgimento: dobbiamo controllare la convergenza dell'integrale improprio

osservando preliminarmente che presenta due problematiche:

­ in perché tale valore annulla il denominatore della funzione integranda;

­ in , perché è appunto un estremo non finito.

In questi casi, spezziamo l'integrale dato come somma di integrali così da poter affrontare un punto problematico alla volta.

Se dimostriamo la convergenza degli integrali al secondo membro allora anche l'integrale al primo membro converge.

Studiamo il primo integrale, ossia

Pij88u9k

che sembra essere un integrale improprio di seconda specie, avente problemi nell'estremo . Calcoliamo il limite per
per controllare se la funzione integranda sia effettivamente illimitata:

dove è intervenuto il limite notevole del seno per giungere al risultato. Il limite esiste ed è finito, di conseguenza l'integranda può
essere prolungata con continuità in ponendo , ciò assicura che l'integrale è convergente.

Dedichiamoci al secondo integrale ossia

Questo è un integrale improprio di prima specie, che può essere studiato mediante il metodo del confronto. Inneschiamo tale
procedimento facendo uso della disuguaglianza notevole

da cui segue immediatamente che

Poiché converge (si comporta come ) allora converge anche .

Possiamo concludere che l'integrale improprio considerato inizialmente converge perché somma di integrali convergenti.

182
Navdeep Singh - navdeep99@outlook.it - 146.241.52.118 - 2020-01-04 22:31:14 r1t2e3m4u5k cikelpo
YouMath.it

Esercizio 11. Verificare che il seguente integrale improprio sia convergente

Svolgimento: osserviamo preliminarmente che l'integrale improprio presenta due problematiche, infatti abbiamo un estremo infinito
e in più è un punto singolare per la funzione integranda.

Grazie all'additività sugli intervalli di integrazione possiamo spezzare l'integrale come somma di integrali ognuno dei quali presenta
una sola problematica:

Se i due integrali al secondo membro convergono lo farà anche l'integrale al primo e il gioco è fatto. Cominciamo con il primo
integrale, ossia:

Possiamo dimostrare che esso converge sfruttando il criterio del confronto asintotico per gli integrali impropri di seconda specie, è
sufficiente costruire la seguente equivalenza asintotica:

Poiché converge (è un integrale improprio notevole) allora convergerà anche l'integrale .


Pij88u9k
Consideriamo il secondo integrale

e analizziamo l'integranda nell'intorno di . In tal caso si può fare riferimento al teorema del confronto per integrali impropri di
prima specie e osservare che vale definitivamente la maggiorazione:

Osservazione: più in generale ricordiamo che la funzione esponenziale è definitivamente maggiore di qualsiasi potenza con
esponente positivo.

Dalla precedente disuguaglianza segue che

dove l'ultima funzione determina un integrale improprio convergente nell'intorno di . L'integrale di partenza converge perché
somma di integrali convergenti.

Esercizio 12. Determinare il carattere del seguente integrale improprio

Svolgimento: per studiare la convergenza dell'integrale improprio

183
eBook edito da YouMath.it acquistato da Navdeep Singh
YouMath.it

dobbiamo studiare il comportamento asintotico dell'integranda in un intorno destro del punto e nell'intorno di , infatti
osserviamo che a tutti gli effetti l'integrale dato presenta due criticità agli estremi: per il denominatore dell'integranda si
annulla mentre l'altro estremo non è finito

Cominciamo dall'intorno di : per la funzione integranda gode della seguente equivalenza asintotica:

per cui l'integrale è convergente in tale intorno perché ha lo stesso comportamento di che è un integrale improprio

notevole di prima specie convergente.

Per quanto riguarda l'intorno destro di prima di studiare cosa succede effettuiamo alcune manipolazioni algebriche
sull'integranda così da facilitarci il compito in seguito: è molto utile effettuare una razionalizzazione al contrario, moltiplicando e
dividendo per .

Pij88u9k

Calcoliamo il limite per

Poiché questo limite è finito allora la funzione integranda è prolungabile con continuità per ponendo e ciò

assicura la convergenza in almeno un intorno di .

In definitiva possiamo concludere che l'integrale improprio dato dalla traccia converge.

ESERCIZI SULLA CONVERGENZA DEGLI INTEGRALI


IMPROPRI PARAMETRICI
Gli esercizi sugli integrali impropri con parametro costituiscono il punto di arrivo di tutti gli esercizi che abbiamo visto fin qui, a partire
dagli esercizi sul calcolo degli integrali impropri. Affrontare questa sezione senza aver compreso a fondo i metodi per stabilire se un
integrale improprio converge è del tutto inutile.

Esercizio 1. Studiare il carattere del seguente integrale al variare del parametro reale

184
Navdeep Singh - navdeep99@outlook.it - 146.241.52.118 - 2020-01-04 22:31:14 r1t2e3m4u5k cikelpo
YouMath.it
Svolgimento: ci troviamo di fronte ad un integrale improprio di prima specie dipendente da un parametro reale . La funzione
integranda, infatti, è una funzione continua sull'intervallo di integrazione , dunque l'unico estremo che richiede uno studio
approfondito è .

Dopo questa analisi preliminare, dobbiamo appoggiarci ad un opportuno criterio di convergenza per gli integrali impropri di prima
specie. Ad esempio possiamo studiare il comportamento asintotico dell'integranda per e cercare di ricondurci ad un
integrale improprio notevole, in modo da stabilire quali sono i valori del parametro reale che garantiscono la convergenza
dell'integrale improprio.

Consideriamo l'integranda

e osserviamo che al tendere di possiamo tralasciare le costanti additive nei binomi , che sono
ininfluenti al cospetto degli infiniti generati dagli addendi .

Ok, ora per confronto asintotico con gli integrali impropri notevoli di prima specie possiamo concludere che:

­ l'integrale improprio converge se e solo se , ossia ;

­ l'integrale improprio diverge se ossia .

Esercizio 2. Studiare il comportamento del seguente integrale improprio, al variare del parametro reale .

Pij88u9k

Svolgimento: consideriamo l'integrale

e spezziamolo nella somma di due integrali, uno di prima specie e uno di seconda, più precisamente:

Se entrambi gli integrali al secondo membro convergono anche l'integrale al primo membro converge. Studiamo separatamente il
carattere dei due integrali cominciando dal primo ossia

che possiamo affrontare con il criterio del confronto asintotico, osservando che per vale la seguente equivalenza
asintotica:

il cui integrale converge in un intorno di se e solo se .

Occupiamoci del secondo integrale, ossia

185
eBook edito da YouMath.it acquistato da Navdeep Singh
YouMath.it
e determiniamo per quali valori di esso risulti convergente. In questo caso utilizziamo un metodo un po' particolare. Calcoliamo il
limite

Il limite è zero perché l'esponenziale al denominatore è un infinito di ordine superiore rispetto al numeratore indipendentemente dal
valore di . Dalla definizione di limite segue che esiste un intorno di tale che

Poiché l'integrale improprio di prima specie con integranda converge allora convergerà anche e ciò ce lo assicura il criterio

del confronto. Traiamo le dovute conclusioni:

converge per mentre converge indipendentemente dal valore di pertanto l'integrale dato dalla traccia converge
se e solo se .

Esercizio 3. Determinare per quali valori di il seguente integrale improprio converge

Pij88u9k

Svolgimento: per studiare la convergenza dell'integrale

dovremo studiare il comportamento asintotico dell'integranda nei punti problematici così da ricondurci in qualche modo a integrali
impropri notevoli.

Per vale la seguente equivalenza asintotica

Ci siamo ricondotti all'integrale notevole

e in base allo schema otteniamo che l'integrale converge se e solo se .

In un intorno destro di tralasciamo il termine quadratico a denominatore e osserviamo che


186
Navdeep Singh - navdeep99@outlook.it - 146.241.52.118 - 2020-01-04 22:31:14 r1t2e3m4u5k cikelpo
YouMath.it

Approfondimento: tale stima asintotica salta fuori dal limite notevole del logaritmo, e per determinarla abbiamo usato il solito
stratagemma di sommare e sottrarre .

Ci siamo ricondotti ad un integrale improprio notevole di seconda specie e converge se e solo se ossia .

Ricapitolando: nell'intorno di abbiamo convergenza per , nell'intorno di abbiamo convergenza per , pertanto
l'integrale di partenza converge se e solo se .

Esercizio 4. Studiare il comportamento del seguente integrale improprio al variare del parametro

Svolgimento: studiamo la convergenza dell'integrale improprio

che presenta problemi sia in che in . In tutti gli altri punti dell'intervallo di integrazione la funzione risulta essere continua e
positiva. Studiamo il comportamento dell'integranda in prossimità di con l'ausilio dello sviluppo di Taylor della funzione seno.

e sostituendo ad ogni occorrenza di il valore si ha che


Pij88u9k

pertanto possiamo scrivere lo sviluppo di Taylor per associato al numeratore

ed estrapolare la seguente stima asintotica:

Per il denominatore invece vale

e dunque sussiste la seguente equivalenza asintotica per l'integranda:

Dalla teoria, sappiamo che l'integrale converge se e solo se l'esponente è minore di , quindi

converge se e solo se .

187
eBook edito da YouMath.it acquistato da Navdeep Singh
YouMath.it

Il primo problema è stato risolto è la condizione di convergenza negli intorni di .

All'infinito vale invece:

questo perché il seno è una funzione limitata e in quanto tale può essere trascurata se messa a fianco all'infinito .

Per il denominatore invece sussiste la seguente relazione asintotica

perché è asintotico a se stesso mentre è asintotico a .

In conclusione possiamo asserire che:

Dalla teoria sappiamo che gli integrali impropri del tipo:

convergono se e solo se l'esponente è maggiore di , conseguentemente

Pij88u9k
converse se e solo se

Abbiamo ottenuto due condizioni, una per la convergenza in un intorno di , l'altra per la convergenza in un intorno di e
mettendole assieme otterremo i valori di in cui l'integrale di partenza è convergente: . L'esercizio è concluso.

Esercizio 5. Determinare per quali valori di la funzione

risulti essere Riemann Integrabile. Stabilire per quali valori di è integrabile in senso improprio; infine calcolare l'integrale
improprio per .

Svolgimento: ricordiamo che una funzione è Riemann integrabile su un intervallo chiuso e limitato se la funzione è limitata ed
esiste finito l'integrale associato.

La funzione proposta dall'esercizio è

ed è continua nell'intervallo di integrazione . Se il limite esiste ed è finito allora è certamente Riemann


integrabile, mentre se non è finito allora avremo a che fare con un integrale improprio di seconda specie.

Studiamo il limite al variare del parametro reale :

Per risolvere questo limite osserviamo che sussistono le seguenti relazioni asintotiche per

188
Navdeep Singh - navdeep99@outlook.it - 146.241.52.118 - 2020-01-04 22:31:14 r1t2e3m4u5k cikelpo
YouMath.it
(essa deriva dal limite notevole del seno)

Grazie a tali considerazioni, possiamo esprimere il limite come

Questo limite è finito se e solo se , e dunque possiamo asserire che la funzione data è Riemann -

integrabile se e solo se .

Per dobbiamo studiare l'integrabilità in senso improprio di nell'intervallo , ossia dobbiamo determinare per quali

valori di l'integrale improprio di seconda specie

converge. Il punto problematico è e in un intorno sinistro di tale valore sussiste la seguente equivalenza asintotica

Pij88u9k

Grazie al criterio del confronto asintotico per gli integrali di seconda specie, sappiamo che l'integrale dato converge se e solo se
converge

e ciò avviene se e solo se

Possiamo concludere l'esercizio scrivendo che:

­ per la funzione è Riemann-Integrabile su ;

­ per la funzione è integrabile in senso improprio.

Calcoliamo a questo punto l'integrale per mediante la definizione stessa di integrale improprio ossia

Osserviamo che al numeratore abbiamo quasi la derivata del radicando, abbiamo solo bisogno di una costante moltiplicativa che
aggiusti le cose. Moltiplichiamo e diviamo per così da ottenere

189
eBook edito da YouMath.it acquistato da Navdeep Singh
YouMath.it

Possiamo finalmente considerare completa la risoluzione dell'esercizio.

Esercizio 6. Studiare il seguente integrale improprio al variare del parametro

Svolgimento: la funzione integranda è continua nell'intervallo indipendentemente dal parametro . Abbiamo però una
singolarità per , infatti il denominatore si annulla per tale valore (se , abbiamo effettivamente dei problemi).

Utilizziamo il criterio del confronto asintotico per gli integrali impropri di seconda specie. Osserviamo che mediante i limiti notevoli
possiamo costruire le seguenti stime asintotiche

mentre

Pij88u9k
pertanto per la funzione integranda sussiste quanto segue

A questo punto interviene il criterio del confronto asintotico il quale ci assicura che l'integrale di partenza ha lo stesso
comportamento dell'integrale

che è un integrale improprio notevole di seconda specie che converge se e solo se l'esponente è minore di :

Possiamo concludere che l'integrale di partenza converge se e solo se .

Esercizio 7. Qual è l'insieme dei valori reali per cui l'integrale

è un integrale improprio e per quali valori tale integrale converge?

Svolgimento: cominciamo con l'analisi preliminare della funzione integranda

190
Navdeep Singh - navdeep99@outlook.it - 146.241.52.118 - 2020-01-04 22:31:14 r1t2e3m4u5k cikelpo
YouMath.it

Essa è continua nell'intervallo al variare di , mentre ha una singolarità per . Se è limitata nell'intervallo
dato sicuramente è Riemann integrabile (ossia l'integrale associato non è improprio) mentre se è illimitata siamo di fronte ad un
integrale impriprio di seconda specie.

Al fine di distinguere se siamo di fronte ad un integrale improprio o meno, studiamo il seguente limite al variare del parametro
.

Facciamo uso delle stime asintotiche che valgono per :

(deriva dal limite notevole dell'esponenziale);

grazie ad esse infatti possiamo riscrivere il limite come

Pij88u9k

che esiste ed è finito se e solo se . Se otteniamo certamente un integrale di Riemann

convergente perché la funzione è continua e limitata nell'intervallo dato.

Se però allora l'integrale è improprio di seconda specie di cui dobbiamo studiare la convergenza. Possiamo tranquillamente
riciclare la stima asintotica trovata in precedenza, così da asserire che per la funzione integranda gode della seguente
stima asintotica

dunque l'integrale parametrico dato dalla traccia si comporta come

ossia come un integrale improprio notevole di seconda specie che converge se e solo se .

In definitiva, possiamo concludere che l'integrale di partenza è improprio se e solo se mentre è un integrale improprio
convergente se e solo se .

Esercizio 8. Per quali valori del parametro il seguente integrale risulta essere convergente

191
eBook edito da YouMath.it acquistato da Navdeep Singh
YouMath.it

Svolgimento: nel caso in cui la funzione integranda

presenta una singolarità per , infatti il fattore non è definito per , mentre il denominatore si annulla.
Naturalmente dobbiamo tenere in considerazione di questa singolarità per studiare correttamente l'integrale improprio.

Possiamo pensare di spezzare l'integrale come somma di due integrali impropri in modo che:

­ il primo sia un integrale improprio di seconda specie;

­ il secondo sia un integrale improprio di prima specie.

La liceità di questo passaggio è assicurata dalla proprietà additiva sugli intervalli di integrazione di cui gode l'integrale:

dove è un qualsiasi numero reale positivo.

Ragioniamo separatamente partendo da

e applichiamo il criterio del confronto asintotico per gli integrali impropri. Il problema riguarda il comportamento dell'integranda
nell'intorno destro di . Troviamo un equivalente asintotico per , ragionando sui singoli fattori:

­ per non abbiamo molto da fare, tale potenza è asintotica a se stessa;


Pij88u9k

­ per è sufficiente osservare che

­ per basta applicare l'equivalenza asintotica determinata dal limite notevole del logaritmo:

In sintesi, possiamo asserire che

A questo punto è sufficiente osservare che l'integrale ha lo stesso comportamento di che è un integrale improprio

notevole di seconda specie e converge se e solo se .

Passiamo al secondo integrale ossia

anche qui ragioniamo per equivalenze asintotiche:

­ per siamo a posto, è equivalente a se stessa;

­ per siamo a posto, è equivalente a se stessa;

­ per è sufficiente trascurare la costante additiva (ininfluente sull'ordine di infinito) e applicare una nota proprietà dei
logaritmi

indi per cui

192
Navdeep Singh - navdeep99@outlook.it - 146.241.52.118 - 2020-01-04 22:31:14 r1t2e3m4u5k cikelpo
YouMath.it

Per il criterio del confronto asintotico per gli integrali impropri, l'integrale converge se e solo se converge

Come mostrare la convergenza? Semplicissimo, la presenza dell'esponenziale al denominatore è indice di probabile convergenza
indipendentemente dal valore che assume . Quando siamo in queste condizioni, è sufficiente mostrare che definitivamente la
funzione data è definitivamente minore di un'altra il cui integrale associato è convergente.

Proviamo a mostrare che in effetti la funzione data è definitivamente minore di per , per farlo è sufficiente mostrare

che il seguente limite è zero

Il limite è zero perché l'esponenziale è un infinito di ordine superiore a qualsiasi potenza. Questo assicura che definitivamente vale

Pij88u9k
Poiché converge allora converge anche l'integrale

per il criterio del confronto semplice, mentre il criterio del confronto asintotico assicura che converge indipendentemente dal
valore di . In definitiva possiamo concludere che l'integrale di partenza converge se e solo se perché somma di integrali
convergenti.

Esercizio 9. Determinare per quali valori del parametro il seguente integrale converge

Svolgimento: l'integrale improprio parametrico si deve esprimere come somma di due integrali ognuno dei quali presenta una sola
problematica

dove è un qualsiasi numero reale positivo. Studieremo separatamente i due integrali, il primo dei quali è un integrale improprio di
seconda specie

infatti la funzione integranda è continua, ma presenta una singolarità nel primo estremo di integrazione. Studiamo quindi il
comportamento asintotico per :

193
eBook edito da YouMath.it acquistato da Navdeep Singh
YouMath.it

Osservazione: la stima asintotica vale soltanto quando tende a zero. In parole: l'arcotangente è
asintotico al suo argomento tende a zero. Nel nostro caso l'argomento tende a più infinito quando , quindi la stima non è
valida.

Grazie a queste considerazioni possiamo asserire che

Occupiamoci del denominatore. Per vale la stima asintotica

e di conseguenza

pertanto per la funzione integranda sussiste la stima asintotica

Per il criterio del confronto asintotico, l'integrale converge se e solo se converge

Pij88u9k
che è riconducibile ad un integrale improprio notevole di seconda specie convergente se e solo se .

Consideriamo il secondo integrale

e studiamone la convergenza con il criterio del confrono asintotico. Per valgono le seguenti stime

questo perché il logaritmo è un infinito di ordine superiore a qualsiasi potenza. Grazie alle stime trovate possiamo asserire che per
la funzione integranda gode della seguente stima

L'integrale converge se e solo se converge

che è un integrale improprio notevole di prima specie e che sappiamo convergere se e solo se .

L'integrale dato dalla traccia converge se converge sia sia e ciò avviene se e solo se e l'esercizio è finalmente
concluso.

194
Navdeep Singh - navdeep99@outlook.it - 146.241.52.118 - 2020-01-04 22:31:14 r1t2e3m4u5k cikelpo
YouMath.it

Esercizio 10. Studiare il seguente integrale al variare del parametro

Svolgimento: analizziamo preliminarmente la funzione integranda , osservando che per la funzione è


continua nell'intervallo di conseguenza

è integrabile secondo Riemann per ogni , ricordiamo infatti che una funzione continua in un intervallo limitato è integrabile
secondo Riemann.

Per , la funzione ha una singolarità per , possiamo infatti vederla come

Facendo riferimento al limite notevole dell'esponenziale, possiamo scrivere la seguente relazione asintotica

e grazie al criterio del confronto asintotico per gli integrali impropri possiamo concludere che l'integrale di partenza converge se e
solo se converge

Pij88u9k

Quest'ultimo è un integrale improprio notevole di seconda specie e converge se e solo se . A questo


punto abbiamo a disposizione tutte le informazioni necessarie a mettere un punto all'esercizio. L'integrale

converge per .

Esercizio 11. Studiare il seguente integrale al variare del parametro

Svolgimento: consideriamo la funzione integranda

ed osserviamo che nell'intervallo di integrazione risulta essere continua, inoltre in tale intervallo il logaritmo è positivo,
pertanto il valore assoluto è superfluo.

L'integrale proposto ha problemi ad entrambi gli estremi, dobbiamo spezzarlo in due integrali e studiarne separatamente la
convergenza

195
eBook edito da YouMath.it acquistato da Navdeep Singh
YouMath.it

Cominciamo con il primo integrale al secondo membro, ossia

che è un integrale improprio di seconda specie in cui la singolarità è . Procediamo con il criterio del confronto asintotico. Per
valgono le seguenti stime

pertanto la funzione integranda gode della seguente equivalenza asintotica

L'integrale ha lo stesso comportamento di


Pij88u9k

che è un integrale improprio notevole di seconda specie che converge se e solo se .

Studiamo il secondo integrale, ossia

Esso converge indipendentemente dal valore di , questo perché l'esponenziale tende troppo velocemente a zero.
Formalmente possiamo asserire che

indipendentemente dal parametro . Definitivamente, per molto grande, sussiste la disuguaglianza

e poiché allora convergerà anche l'integrale .

In definitiva possiamo asserire che l'integrale di partenza converge se e solo se perché somma di integrali convergenti.

Esercizio 12. Studiare il carattere del seguente integrale al variare del parametro reale

196
Navdeep Singh - navdeep99@outlook.it - 146.241.52.118 - 2020-01-04 22:31:14 r1t2e3m4u5k cikelpo
YouMath.it

Svolgimento: per studiare la convergenza dell'integrale improprio bisogna studiare il comportamento dell'integranda nell'intorno
destro di e nell'intorno di , in modo da poter applicare il teorema del confronto asintotico e dedurne il carattere
mediante gli integrali impropri notevoli.

Nell'intorno di : i limiti notevoli ci suggeriscono le due equivalenze asintotiche relative a numeratore e denominatore

conseguentemente

e abbiamo convergenza se e solo se . Nell'intorno di conviene riscrivere il seno iperbolico


mediante la definizione

dunque la funzione integranda gode della stima asintotica

e inPij88u9k
questo caso l'integrale improprio associato a converge se .

Mettendo assieme le due condizioni otteniamo che l'integrale dato dalla traccia converge se e solo se .

Esercizio 13. Studiare al variare del parametro il seguente integrale

Svolgimento: dobbiamo determinare per quali valori di l'integrale

e per prima cosa dobbiamo studiare il segno dell'argomento del valore assoluto

Nell'intervallo di integrazione , il numeratore della funzione integranda si esprime come

quindi l'integrale si scompone come

Il primo integrale si può semplificare e si riscrive come

197
eBook edito da YouMath.it acquistato da Navdeep Singh
YouMath.it

e converge se e solo se , osserviamo infatti che quello ottenuto è un integrale improprio notevole di
seconda specie.

Il secondo integrale, ossia

ha lo stesso comportamento di

che converge se e solo se . Mettiamo assieme le due condizioni ottenute per così da concludere
che l'integrale di partenza converge se e solo se .

Esercizio 14. Discutere per quali valori di il seguente integrale improprio

Svolgimento: siamo di fronte ad un integrale improprio di seconda specie

Pij88u9k

ed infatti la funzione integranda presenta due punti singolari e .

Per studiare l'integrale improprio determiniamo le equivalenze asintotiche e facciamo intervenire il criterio del confronto asintotico
per gli integrali.

Quando sussistono le seguenti stime asintotiche

(deriva dal limite notevole dell'esponenziale)

conseguentemente si ha che la funzione integranda gode della seguente equivalenza asintotica

In base alla teoria degli integrali impropri notevoli di seconda specie abbiamo convergenza se e solo se
.

Lavoriamo per , nel qual caso conviene effetturare la seguente sostituzione: ed osservare
che quando si ha che .

Per il numeratore è asintotico a che è una costante. Il denominatore, grazie alla sostituzione
gode invece della seguente stima asintotica

198
Navdeep Singh - navdeep99@outlook.it - 146.241.52.118 - 2020-01-04 22:31:14 r1t2e3m4u5k cikelpo
YouMath.it
e dunque tornando in la precedente relazione diventa

Per la funzione integranda possiamo pertanto scrivere la seguente stima

la cui convergenza è assicurata se e solo se , questo perché ci siamo ricondotti ad un integrale improprio

notevole di seconda specie.

L'integrale di partenza converge se il parametro rispetta entrambe le condizioni contemporaneamente ossia quando
e ciò conclude l'esercizio.

Esercizio 15. Studiare al variare di la convergenza del seguente integrale

Svolgimento: l'integrale da studiare ossia

richiede un'attenzione particolare per via della potenza . In base al segno che assume la funzione integranda ha diverse stime
asintotiche.
Pij88u9k
Osserviamo comunque che la funzione integranda è continua nell'intervallo ed è a segno costante, inoltre l'integrale
presenta due problematiche, è naturale quindi spezzare l'integrale come somma di integrali:

Se gli integrali al secondo membro convergono contemporaneamente allora convergerà l'integrale al primo membro. Studiamo il
primo integrale ossia

Se allora per e di conseguenza , pertanto

e di conseguenza la funzione integranda è prolungabile con continuità in . Ricordiamo che le funzioni continue in un
intervallo chiuso e limitato sono integrabili secondo Riemann: possiamo concludere che converge per .

Possiamo dire lo stesso per , valore per il quale l'integrale diventa

Ancora una volta la funzione integranda è continua nell'intervallo chiuso e limitato e dunque è integrabile secondo Riemann.

199
eBook edito da YouMath.it acquistato da Navdeep Singh
YouMath.it

Per si ha che per e questa volta il punto rappresenta una singolarità di seconda specie, infatti la
funzione integranda smette di essere limitata nell'intervallo considerato.

Per studiare il comportamento dell'integrale facciamo affidamento al criterio del confronto asintotico per gli integrali impropri di
seconda specie. Per sussistono le seguenti stime asintotiche: la prima è quella dell'arcotangente derivante dal suo limite
notevole

e dunque la funzione integranda gode della seguente stima asintotica

L'integrale si comporta asintoticamente come

che converge se e solo se (è un integrale notevole di seconda specie). L'analisi condotta ci permette di
concludere che l'integrale converge se .

OraPij88u9k
concentriamo la nostra attenzione sul secondo integrale ossia

e affrontiamolo distinguendo i casi . Naturalmente abbiamo a che fare con un integrale improprio di prima
specie, giacché l'intervallo di integrazione è illimitato. Procediamo con il criterio del confronto asintotico per gli integrali di seconda
specie

Se allora valgono le seguenti stime asintotiche per :

e dunque per la funzione integranda vale la stima asintotica

Il criterio del confronto asintotico ci assicura che l'integrale converge se converge

e ciò avviene se e solo se

200
Navdeep Singh - navdeep99@outlook.it - 146.241.52.118 - 2020-01-04 22:31:14 r1t2e3m4u5k cikelpo
YouMath.it

Se l'integrale converge perché ha lo stesso comportamento di

Se l'integranda si comporta asintoticamente come , questo perché per sussistono le seguenti stime

Poiché l'integrale è un integrale improprio notevole convergente allora convergerà anche .

In definitiva possiamo asserire che converge se e solo se e di conseguenza l'integrale dato dalla traccia converge se e
solo se convergono contemporaneamente ossia se . L'esercizio è finalmente terminato.

Esercizio 16. Studiare la convergenza del seguente integrale al variare del parametro

Svolgimento: per studiare la convergenza/divergenza dell'integrale improprio di seconda specie

Pij88u9k

possiamo limitarci a studiare il comportamento della funzione integranda nell'intorno destro di , poiché in l'integranda
è ben definita e continua.

Per semplicità, ragioniamo distinguendo tra i vari casi.

1) Se il denominatore non crea problemi: è somma di infinito e di un infinitesimo, di conseguenza per la funzione integranda
vale

dunque la funzione integranda è prolungabile con continuità nel punto . Ricordando che la continuità in un intervallo chiuso e
limitato è condizione sufficiente per l'integrabilità possiamo asserire che l'integrale dato converge per .

2) Se il denominatore, ancora una volta, non crea problemi: è somma di un infinito e di un infinitesimo

di consuenza

e dunque la funzione integranda è prolungabile con continuità in . Poiché la funzione prolungata è continua in un chiuso e
limitato allora essa è integrabile secondo Riemann.

201
eBook edito da YouMath.it acquistato da Navdeep Singh
YouMath.it
3) Se il denominatore è finito e della forma , quindi la funzione è continua nell'intervallo di integrazione chiuso e
limitato ed è dunque integrabile secondo Riemann.

4) Mettiamoci nel caso . L'integranda è somma di un infinito e di un infinitesimo, di conseguenza possiamo scrivere la
stima asintotica

Per studiare la convergenza usiamo il criterio del confronto asintotico per gli integrali impropri di seconda specie e in particolare per
confronto asintotico con gli integrali impropri notevoli, vediamo che c'è convergenza se . Dato che ci
troviamo nel caso , l'integrale converge per .

5) Se l'integrale diventa

che è un integrale improprio notevole di seconda specie divergente.

6) Se , possiamo stimare asintoticamente l'integranda con , infatti

quindi abbiamo convergenza a patto che e dunque .

Mettiamo insieme i risultati, così da concludere che l'integrale converge se .

Pij88u9k
Esercizio 17. Studiare la convergenza del seguente integrale al variare del parametro

Svolgimento: la funzione integranda è continua nell'intervallo di integrazione perché

composizione di funzioni continue, non presenta punti singolari interni all'intervallo. Siamo di fronte ad un integrale improprio di
prima specie perché l'intervallo è illimitato.

Per studiare la convergenza dell'integrale improprio di prima specie osserviamo che nell'integranda al tendere di risulta

che , quindi possiamo ricorrere al limite notevole del coseno e passare all'equivalente asintotico

mentre per il denominatore osserviamo che è un infinito di ordine superiore a quello degli altri addendi, conseguentemente
sussiste la stima asintotica

In definitiva possiamo scrivere che vale la seguente stima per la funzione integranda:

Il criterio del confronto asintotico ci assicura quindi che l'integrale dato dalla traccia converge se e solo se converge il seguente:

202
Navdeep Singh - navdeep99@outlook.it - 146.241.52.118 - 2020-01-04 22:31:14 r1t2e3m4u5k cikelpo
YouMath.it

che è riconducibile ad un integrale improprio notevole di seconda specie che converge se e solo se

Chiudiamo l'esercizio scrivendo che l'integrale improprio converge se e solo se .

ESERCIZI SULLE FUNZIONI INTEGRALI


Esercizio 1. Determinare esplicitamente la seguente funzione integrale

dove

Svolgimento: l'esercizio fornisce una funzione definita mediante un integrale

Pij88u9k
in cui la funzione integranda è data da una funzione definita a tratti

La variabilità dell'estremo di integrazione superiore fa sì che il valore dell'integrale dipenda dal valore assunto da tale estremo (o
meglio, da tale variabile). In questo senso abbiamo a che fare con quella che a tutti gli effetti è una funzione.

Il nostro scopo prevede di determinare un'espressione esplicita della funzione , osserviamo comunque che non è sempre
possibile esprimere esplicitamente una funzione integrale.

Il fatto che l'integranda sia definita a tratti non deve spaventarci, perché sappiamo che vale l'utilissima proprietà degli integrali

Consideriamo l'integrale

se allora , in entrambi i casi e dunque ricadiamo sul secondo ramo della funzione
, di conseguenza

si tratta di un integrale banale in cui possiamo integrare separatamente i singoli addendi

203
eBook edito da YouMath.it acquistato da Navdeep Singh
YouMath.it

Men che meno dobbiamo farci spaventare dall'estremo in , perché abbiamo calcolato un integrale definito e dobbiamo effettuare
una pura e semplice valutazione letterale

Se , cosa succede? Usiamo la proprietà espressa precedentemente

in questo modo possiamo specificare i rami che definiscono l'integranda

Abbiamo finito. La definizione per tratti dell'integranda ci ha dettato la linea da seguire nel calcolo dell'integrale, e questo a sua volta
ci ha condotto ad un'espressione della funzione integrale come funzione definita a tratti:

Pij88u9k

Ora l'esercizio è completamente svolto.

Esercizio 2. La funzione integrale con integranda definita nell'intervallo a partire dal punto
calcolata in quanto vale?

Svolgimento: il primo passo per la risoluzione dell'esercizio prevede di scrivere la funzione integrale

dove l'integranda è da intendersi definita come . Vogliamo valutare in , e dunque calcolare

Il valore assoluto ci dà fastidio, dunque lo riscriviamo sfruttandone la definizione

possiamo allora ricorrere ad una nota proprietà dell'integrale secondo Riemann per riscrivere opportunamente il precedente
integrale come segue

e facendo intervenire la definizione di modulo

204
Navdeep Singh - navdeep99@outlook.it - 146.241.52.118 - 2020-01-04 22:31:14 r1t2e3m4u5k cikelpo
YouMath.it

Ora possiamo rispondere alla domanda posta dall'esercizio: .

Esercizio 3. Studiare il segno della seguente funzione integrale

Svolgimento: per prima cosa osserviamo che per una nota proprietà degli integrali definiti

inoltre la funzione integranda

è continua in tutto l'asse reale, pertanto la funzione integrale è definita per ogni e in più soddisfa le ipotesi del teorema
Pij88u9k
fondamentale del calcolo integrale, grazie al quale possiamo calcolare la derivata prima della funzione integrale, anzi ci dice che
essa coincide con l'integranda

La derivata prima di è positva o nulla per ogni (ed in particolare è nulla se e solo se )
perché prodotto di funzioni non negative, concludiamo abbastanza facilmente che è monotona crescente.

Si ha quindi che:

­ se allora la funzione soddisfa la relazione , cioè la funzione è non negativa per ;

­ se allora la funzione soddisfa la relazione , ossia la funzione è non positiva per .

L'esercizio è concluso.

Esercizio 4. Stabilire l'ordine di infinitesimo della seguente funzione integrale

rispetto all'infinitesimo campione per .

Svolgimento: applichiamo la definizione di ordine di infinitesimo rispetto all'infinitesimo campione per . In soldoni
dobbiamo determinare l'esponente tale che il seguente limite esiste finito e diverso da zero

205
eBook edito da YouMath.it acquistato da Navdeep Singh
YouMath.it

Naturalmente il limite appena scritto è una forma indeterminata , possiamo applicare il teorema di De l'Hopital in combinazione

con il teorema fondamentale del calcolo integrale, il quale ci aiuta nel calcolo della derivata della funzione integrale.

Deriviamo separatamente numeratore e denominatore cominciando dalla derivata di . Possiamo ottenere la sua derivata
applicando il teorema fondamentale del calcolo integrale: esso ci assicura che la derivata della funzione integrale coincide con
l'integranda

La derivata del denominatore è immediata, sappiamo infatti che

dunque il limite per determinare l'ordine di infinitesimo è questa

Possiamo affrontare questo limite affindandoci alle equivalenze asintotiche. Il limite notevole dell'arcotangente permette di scrivere
la seguente equivalenza

mentre per . In definitiva possiamo scrivere

Pij88u9k

L'ultimo limite esiste, finito e diverso da zero se e solo se . Possiamo quindi concludere che la funzione
integrale ha ordine di infinitesimo per rispetto all'ordine di infinitesimo .

Esercizio 5. Calcolare la derivata di ciascuna delle seguenti funzioni integrali

Svolgimento: prima di procedere con lo svolgimento dell'esercizio proponiamo una piccola e velocissima parentesi teorica. Il
teorema fondamentale del calcolo integrale ci dice che se abbiamo una funzione continua nell'intervallo allora la
funzione integrale

è una funzione derivabile in ed inoltre . Nel caso in cui avessimo un estremo di


integrazione divero da , ad esempio

dove è una funzione derivabile, possiamo vedere come una funzione composta. Per le regole di derivazione delle
funzioni composte abbiamo che

206
Navdeep Singh - navdeep99@outlook.it - 146.241.52.118 - 2020-01-04 22:31:14 r1t2e3m4u5k cikelpo
YouMath.it
quindi la derivata prima della funzione integrale è uguale alla funzione integranda composta con la funzione estremo per la derivata
della funzione estremo. Se anche il secondo estremo è una funzione di , ossia se la funzione integrale si presenta nella forma

in tal caso la sua derivata è

Ora possiamo calcolare la derivata di e . La derivata di si ottiene utilizzando la regola di derivazione del
prodotto

e per il teorema fondamentale del calcolo integrale otteniamo

Osserviamo che il calcolo esplicito dell'integrale è praticamente impossibile, perché non è esprimibile mediante composizione di
funzioni elementari.

Procediamo con la derivata di


Pij88u9k

La derivata del coseno è nota, mentre è un po' più delicato il calcolo della derivata in cui compare l'integrale. L'estremo superiore
non è la semplice ma qualcosa di più elaborato, dobbiamo pertanto integrare utilizzando la regola di derivazione delle funzioni
composte

Abbiamo terminato il nostro compito.

Esercizio 6. Dimostrare che la funzione integrale

il cui dominio è , è invertibile. Detta la funzione inversa, calcolare .

Svolgimento: iniziamo l'esercizio dimostrando che la funzione integrale è effettivamente invertibile e per farlo faremo vedere
che essa è monotona. Come fare? Faremo intervenire il teorema fondamentale del calcolo integrale, ma prima dobbiamo verificarne

207
eBook edito da YouMath.it acquistato da Navdeep Singh
YouMath.it
le ipotesi.

La funzione integranda

è ovviamente continua su tutto l'asse reale perché è quoziente di funzioni continue. Il teorema fondamentale del calcolo integrale, in
combinazione con la regola di derivazione delle funzioni composte, assicura che la derivata della funzione integrale è la seguente

Ovviamente la derivata prima di è sempre positiva perché prodotto di funzioni positive, e pertanto è una funzione
strettamente crescente.

La monotonia ci permette di concludere che la funzione è invertibile nel dominio. Dedichiamoci ora all'altra domanda
dell'esercizio. Per calcolare la derivata prima della funzione inversa, dobbiamo utilizzare la formula di derivazione per la funzione
inversa.

Per prima cosa dobbiamo determinare tale che , nel nostro caso , dunque dobbiamo risolvere
l'equazione

Poiché la funzione è monotona, l'equazione precedente ammette, se esiste, un'unica soluzione. Per determinare ci viene
in soccorso una proprietà degli integrali ossia

Pij88u9k

Se gli estremi di integrazione sono uguali allora l'integrale è zero. Alla luce di questa proprietà possiamo analizzare più a fondo
l'equazione (\bullet) che è soddisfatta se e solo se gli estremi sono uguali ossia quando :
abbiamo determinato il nostro .

Per la formula di derivazione per la funzione inversa sappiamo che

Ora

In conclusione .

Osservazione: è bene tenere a mente che l'arcotangente è la funzione inversa della funzione tangente ed infatti sussiste la relazione
notevole

E con questo è tutto.

208
Navdeep Singh - navdeep99@outlook.it - 146.241.52.118 - 2020-01-04 22:31:14 r1t2e3m4u5k cikelpo
YouMath.it
Esercizio 7. Provare che la seguente funzione è invertibile:

e calcolare

Svolgimento: consideriamo la funzione integrale

che è ovviamente definita nell'intervallo ed è facile capirlo: l'integrada è infatti definita e continua su tutto .

La funzione integrale è continua in ogni punto del proprio insieme di definizione perché l'integranda è limitata e integrabile.

Se riusciamo a dimostrare che la funzione è strettamente monotona abbiamo finito perché una funzione continua in un
intervallo è invertibile se e solo se è monotona. Per studiare la monotonia calcoliamo la derivata di

e ci serviamo del teorema fondamentale del calcolo integrale, il quale garantisce la derivabilità della funzione integrale (perché
l'integranda è continua) e ci permette di calcolare

Osserviamo che la derivata prima è maggiore di zero perché quoziente di quantità positive nel dominio:
.
Pij88u9k
Ne deduciamo subito che la funzione integrale è strettamente crescente sul proprio dominio. In sintesi, è definita su un
intervallo, è continua ed è strettamente monotona, dunque è invertibile.

Per la seconda parte del problema dobbiamo ricorrere al teorema per la derivata della funzione inversa, il quale ci permette di
calcolare la derivata della funzione inversa in un punto senza conoscere l'espressione esplicita dell'inversa.

A noi interessa . Il teorema ci dice che

Quali sono i valori di Ovviamente , mentre per determinare dobbiamo semplicemente ricavare la
preimmagine di mediante , ossia dobbiamo determinare il valore di tale che ossia

da cui

Dato che abbiamo a che fare con una funzione integrale invertibile su , ed in particolare iniettiva, ad un immagine può
corrispondere un'unica preimmagine. Le proprietà degli integrali non ci lasciano scampo: .

A questo punto non ci resta che applicare il teorema per la derivata della funzione inversa

209
eBook edito da YouMath.it acquistato da Navdeep Singh
YouMath.it
e abbiamo finito.

Esercizio 8. Dopo aver trovato esplicitamente l'espressione della seguente funzione integrale, determinare per quali valori del
parametro essa risulti essere monotona crescente

Svolgimento: per conoscere la monotonia della funzione integrale

non è necessario calcolarne una primitiva, ma nel caso della funzione considerata non è complicato, infatti abbiamo che

e quindi si può procedere allo studio della monotonia di procedendo nella maniera standard. Se vogliamo evitare il calcolo
della primitiva, viene in nostro soccorso il teorema fondamentale del calcolo integrale, per il quale risulta che

Per far sì che sia una funzione crescente dobbiamo richiedere che la derivata prima sia non negativa nel dominio e ciò
avviene se e solo se il polinomio di secondo grado risulti essere non negativo per ogni .
Pij88u9k
Tale condizione è soddisfatta nel momento in cui il discriminante associato è negativo o nullo e il coefficiente di è positivo,
proprio in questo caso.

da cui

Tali valori rendono quindi il discriminante del polinomio non positivo ed essendo il coefficiente del termine quadratico positivo,
abbiamo la certezza che il polinomio sia positivo sull'intero asse reale.

Possiamo quindi concludere che la funzione integrale è crescente nel dominio, nel momento in cui .

Esercizio 9. Studiare la monotonia della funzione integrale sull'intervallo dove

Svolgimento: abbiamo la funzione integrale

in cui la funzione integranda è continua nel dominio perché

composizione di funzioni continue.

Per determinare gli intervalli di monotonia di utilizziamo il teorema fondamentale del calcolo integrale che ci permette di
calcolare la derivata prima della funzione integrale

210
Navdeep Singh - navdeep99@outlook.it - 146.241.52.118 - 2020-01-04 22:31:14 r1t2e3m4u5k cikelpo
YouMath.it

e studiamo il segno della derivata prima nell'intervallo e per farlo studieremo separatamente il segno del numeratore e del
denominatore:

Risolviamo la disequazione di secondo grado così da ottenere che il denominatore è positivo se e solo se
.

Costruiamo la tabella dei segni concentrandoci nell'intervallo

Nell'intervallo la derivata prima della funzione è positiva e dunque la funzione è strettamente crescente in .

Esercizio 10. Detta

Pij88u9k
scrivere l'equazione della retta tangente al grafico della funzione integrale nel punto di ascissa .

Svolgimento: ricordiamo che l'equazione della retta tangente al grafico di una funzione derivabile nel punto di ascissa è
data dalla formula

da cui si evince che rappresenta il coefficiente angolare della retta tangente al grafico, in accordo con l'interpretazione
geometrica di derivata di una funzione.

Guardiamo un momento l'equazione della retta tangente: in essa compaiono , ossia la derivata prima di nel punto
e cioè il valore che la funzione assume nel punto .

Per applicare la formula abbiamo bisogno della derivata della funzione integrale che possiamo ottenere mediante il teorema
fondamentale del calcolo integrale congiuntamente al teorema per la derivata della funzione composta. Naturalmente tutte le ipotesi

sono soddisfatte giacché l'integranda è una funzione continua in tutto l'asse reale.

La derivata prima della funzione integrale è data da:

Grazie alla regola di derivazione delle potenze concludiamo il calcolo

211
eBook edito da YouMath.it acquistato da Navdeep Singh
YouMath.it
Valutiamo la derivata prima della funzione per

Ora non ci resta che determinare ossia

Rimarchiamo che il risultato è zero per una nota proprietà dell'integrale definito in cui gli estremi sono finiti e uguali.

Abbiamo tutti gli elementi per scrivere la retta tangente al grafico della funzione

L'esercizio è finito.

Esercizio 11. Trovare gli eventuali massimi e minimi e scrivere l'equazione della retta tangente al grafico della funzione integrale

nel punto di ordinata .

Svolgimento: siamo di fronte ad una funzione integrale

Pij88u9k

con funzione integranda continua in tutto il suo dominio, pertanto integrabile tra . Il dominio della funzione

integrale è quindi .

Calcoliamo al derivata prima della funzione integrale utilizzando il teorema fondamentale del calcolo integrale:

La derivata prima non si annulla per alcun valore di conseguentemente non ha punti estremanti, inoltre è sempre positiva perché
quoziente di quantità positive. Questo ci permette di asserire che la funzione integrale è monotona strettamente crescente (ed è
quindi iniettiva).

La seconda parte dell'esercizio richiede di determinare l'equazione della retta tangente al grafico della funzione integrale nel punto
di ordinata : viene in nostro soccorso la formula

purtroppo però non conosciamo il valore di , il testo dell'esercizio invece fornisce , però possiamo determinarlo
impostando l'equazione o detto in altri termini

Abbiamo già detto (e dimostrato) che la funzione integrale è iniettiva, dunque l'equazione appena scritta ha un'unica soluzione che
si ottiene abbastanza agevolmente se si tiene a mente che se gli estremi di un integrale sono uguali allora l'integrale è nullo: è
sufficiente quindi richiedere che .

Siamo a posto, l'esercizio è praticamente svolto, manca solo l'equazione della retta tangente che possiamo trovare sostituendo
semplicemente i valori che ci servono ossia:

212
Navdeep Singh - navdeep99@outlook.it - 146.241.52.118 - 2020-01-04 22:31:14 r1t2e3m4u5k cikelpo
YouMath.it

L'equazione della retta tangente è quindi

e l'esercizio è terminato.

Esercizio 12. Studiare il dominio della seguente funzione integrale

Svolgimento: l'esercizio chiede di determinare i valori di in cui l'integrale esiste finito. Come procedere in questi casi? La prima

cosa che bisogna fare è determinare il dominio della funzione integranda che si ottiene imponendo che il

denominatore sia diverso da zero, e che il radicando sia positivo (perché al denominatore). Con semplici calcoli si arriva a

Nel dominio è anche continua perché composizione di funzioni continue, inoltre rappresenta un punto singolare per
. Proprio per questo motivo

Pij88u9k

rappresenta un integrale di seconda specie al variare di . Ovviamente non dobbiamo calcolare esplicitamente l'integrale, non è
quello che chiede l'esercizio. Utilizzeremo invece i criteri di convergenza integrale di seconda specie.

Fissiamo e studiamo la convergenza dell'integrale. Osserviamo che la funzione integarnda è positiva nell'intervallo ed
utilizziamo il criterio del confronto asintotico per gli integrali nel punto singolare, ossia . Per sussiste la seguente stima
asintotica

e poiché

è un integrale improprio notevole convergente possiamo asserire che l'integrale

converge per ogni . Per la funzione integrale è perché gli estremi di integrazione coincidono. In definitiva la
funzione integrale ha per dominio

In realtà, il calcolo dell'integrale non è complicato ed è possibile determinare esplicitamente l'espressione della funzione integrale
che risulta essere

213
eBook edito da YouMath.it acquistato da Navdeep Singh
YouMath.it

Lasciamo i conti agli studenti più volenterosi (suggerimento, procedere per sostituzione ponendo )

L'esercizio è terminato.

Esercizio 13. Determinare il dominio della funzione

Svolgimento: la funzione integrale proposta dall'esercizio ha per funzione integranda

il cui dominio è dettato dalla condizione

Naturlamente possiamo risolvere l'equazione con la legge di annullamento del prodotto e ottenere che il dominio dell'integranda è

Inoltre la funzione è continua in tutto il suo dominio, pertanto è integrabile in ogni intervallo [-1, x] contenuto nel dominio,
naturalmente affinché si abbia un intervallo interamente contenuto nel dominio, dobbiamo richiedere che .

Possiamo trarre la prima importante informazione: se allora è ben definita e dunque .

Cosa succede se ? Otteniamo l'integrale improprio


Pij88u9k

di cui controlliamo il comportamento mediante il criterio del confronto asintotico per gli integrali di seconda specie. Per
valgono le seguenti relazioni asintotiche

e dunque per la funzione integranda soddisfa la seguente relazione asintotica

Ora attenzione:

è un integrale improprio notevole divergente, questo assicura che l'integrale improprio

diverge, e di conseguenza la funzione integrale non può essere definita per .

214
Navdeep Singh - navdeep99@outlook.it - 146.241.52.118 - 2020-01-04 22:31:14 r1t2e3m4u5k cikelpo
YouMath.it
Studiamo ora come si comporta la funzione integranda per . In tal caso valgono le seguenti stime asintotiche

di conseguenza la funzione integranda soddisfa la seguente stima asintotica

e l'integrale associato ossia

converge in quanto integrale improprio notevole di seconda specie e per il criterio del confronto asintotico per gli integrali di
seconda specie converge anche l'integrale

Questo ci assicura che è un punto del dominio della funzione integrale, inoltre per ogni possiamo scrivere la
funzione integrale come:

Pij88u9k

Il primo integrale è certamente convergente così come converge anche il secondo integrale al secondo membro per ogni . In
definitiva possiamo concludere che il dominio della funzione integrale è:

L'esercizio è concluso.

Esercizio 14. Dimostrare che la funzione definita da

è iniettiva in .

Svolgimento: per dire che la funzione integrale

è iniettiva o meno, è sufficiente determinare la derivata della funzione integrale mediante l'uso del teorema fondamentale del calcolo
integrale, che può essere applicato giacché la funzione integranda è continua su tutto l'asse reale.

Il teorema fondamentale del calcolo integrale assicura che la funzione è derivabile nel suo dominio ed in più fornisce la
formula che ci permette di determinare la derivata prima

La derivata prima è sempre positiva (è una funzione esponenziale) quindi la funzione integrale è strettamente crescente e poiché la
monotonia di una funzione continua in un intervallo ne assicura l'iniettività di quest'ultima, l'esercizio è praticamente finito: la

215
eBook edito da YouMath.it acquistato da Navdeep Singh
YouMath.it
funzione integrale è iniettiva.

Esercizio 15. Calcolare i punti estremanti della seguente funzione integrale sull'intervallo

Svolgimento: dobbiamo studiare la funzione integrale

nell'intervallo chiuso e limitato . Osserviamo che la funzione integranda è una funzione continua
perché prodotto di funzioni continue, pertanto non solo è continua ma è anche derivabile in .

Il teorema di Weierstrass ci assicura l'esistenza dei punti di massimo e di minimo in . Mediante il teorema fondamentale del
calcolo integrale, le cui ipotesi sono soddisfatte, possiamo calcolare la derivata prima di :

Utilizziamo il teorema di Fermat sui punti stazionari: se abbiamo un punto di massimo o di minimo interno all'intervallo
allora tale valore annulla la derivata prima. Imponiamo quindi che la derivata prima sia nulla, così da determinare gli eventuali
candidati punti di massimo o di minimo

Per la legge di annullamento del prodotto, l'equazione appena scritta è soddisfatta se e solo se almeno uno dei fattori è , ossia:

Pij88u9k

dove è giustificato dal fatto che stiamo lavorando in . Sia sono potenziali punti di massimo o di

minimo relativi. Per classificare i punti, studiamo il segno della derivata prima:

Per quindi non influisce sul segno del prodotto, che dipende invece dal segno del coseno:

A questo punto si costruisce la tabella dei segni per scoprire che è

­ positiva nell'intervallo e nell'intervallo ;

­ nulla per ;

­ negativa nell'intervallo

di conseguenza la funzione integrale

­ cresce strettamente nell'intervallo e nell'intervallo ;

­ ha per un punto di minimo locale (è di minimo assoluto);

216
Navdeep Singh - navdeep99@outlook.it - 146.241.52.118 - 2020-01-04 22:31:14 r1t2e3m4u5k cikelpo
YouMath.it

­ ha per è punto di flesso a tangente orizzontale;

­ decresce strettamente nell'intervallo .

Per completare l'analisi dobbiamo studiare cosa succede agli estremi dell'intervallo, ossia quando , in
particolare dobbiamo confrontare i valori che la funzione assume aglie estremi per calcolare il punto di massimo, il che equivale a
confrontare gli integrali

Entrambi gli integrali possono essere risolti per parti: risolviamo il primo

e procedendo per parti si ottiene facilmente quanto segue

mentre

Il primo integrale è maggiore del secondo, di conseguenza è un punto di massimo assoluto. Ora l'esercizio è completo.

Pij88u9k
Esercizio 16. Scrivere lo sviluppo di Taylor con resto di Peano, centrato in al secondo ordine della funzione

Svolgimento: la funzione integrale è

e per trovare lo sviluppo di Taylor utilizziamo la definizione stessa ossia:

Sostituendo a il valore la precedente espressione diventa

Per completare l'esercizio abbiamo bisogno dei valori che possiamo determinare mediante l'uso delle
proprietà degli integrali e il teorema fondamentale del calcolo integrale in combinazione con la regola di derivazione delle funzioni
composte. In dettaglio:

­ si vede facilmente che

perché gli estremi di integrazione sono uguali;

­ il calcolo di richede il calcolo della derivata prima della funzione integrale che è:

217
eBook edito da YouMath.it acquistato da Navdeep Singh
YouMath.it

Possiamo scrivere quindi che:

La derivata seconda della funzione integrale di si ottiene derivando la derivata prima

che tramite delle manipolazioni algebriche diventa

Pij88u9k
Valutiamo la funzione ottenuta per così da avere il termine necessario per costruire lo sviluppo di Taylor

Non ci resta che sostituire i valori ottenuti nello sviluppo di Taylor così da ottenere

dove è un o piccolo di . L'esercizio è concluso.

Esercizio 17. Scrivere la formula di Taylor arrestata al secondo ordine di nel punto di ascissa con il resto secondo
Peano, dove:

Svolgimento: ricordiamo che sotto le ipotesi del teorema di Taylor, la formula di Taylor centrata in un punto e del secondo ordine
associata ad una funzione è:

dove è un o piccolo di e rappresenta il resto nella forma di Peano.

Nel caso in questione la formula di Taylor diventa

218
Navdeep Singh - navdeep99@outlook.it - 146.241.52.118 - 2020-01-04 22:31:14 r1t2e3m4u5k cikelpo
YouMath.it

che per essere completata abbiamo bisogno di , ossia del valore che la funzione integrale assume per , di , ossia
la derivata di e della derivata seconda .

Calcoliamo che corrisponde alla seguente espressione

ed è uguale a zero perché gli estremi di integrazione sono uguali. Dedichiamoci al calcolo della derivata prima e in seguito la
valuteremo per . Tale calcolo avviene mediante il teorema fondamentale del calcolo integrale in combinazione con il teorema
di derivazione delle funzioni composte:

Il valore che la derivata prima di valutata per è

Ora tocca calcolare la derivata seconda di , derivando semplicemente la derivata prima con i soliti metodi di derivazione

Pij88u9k

A questo punto calcoliamo la derivata seconda di per

e sostituiamo i valori ottenuti nella formual di Taylor, così da raggiungere il risultato, ossia:

e l'esercizio è concluso.

Esercizio 18. Da la funzione integrale

­ calcolare e il polinomio di Taylor di ordine di in .

­ calcolare il polinomio di in .

Svolgimento: consideriamo la funzione integrale

219
eBook edito da YouMath.it acquistato da Navdeep Singh
YouMath.it

e osserviamo che, fissato , si ha che di conseguenza l'intervallo di integrazione è ed anche se la funzione


integranda presenta problemi in , siamo tranquilli, perché comunque si fissi non apparterrà all'intervallo.

Per determinare la derivata prima della funzione integrale dobbiamo necessariamente far ricorso al teorema fondamentale del
calcolo integrale, in combo con la regola di derivazione delle funzioni composte. Ovviamente tutte le ipotesi del teorema
fondamentale del calcolo sono soddisfatte, infatti l'integranda

è una funzione continua nell'intervallo , pertanto:

Il prossimo punto consiste nel determinare il polinomio di Taylor con centro associati alla derivata prima della funzione
integrale. Per farlo scriviamo come segue:

Pij88u9k

Utilizziamo gli sviluppi notevoli di Taylor, in particolare può tornare utile:

osserviamo infatti che

e dunque possiamo utilizzare lo sviluppo notevole scritto in precedenza: basta sostituire a la potenza , così da ottenere

che grazie alle proprietà delle potenze e degli o piccolo si riscrive come:

L'esercizio chiede il polinomio di Taylor di grado , pertanto possiamo tranquillamente trascurare tutti i termini che hanno grado
superiore a

Procediamo allo stesso modo per la frazione così da ottenere

220
Navdeep Singh - navdeep99@outlook.it - 146.241.52.118 - 2020-01-04 22:31:14 r1t2e3m4u5k cikelpo
YouMath.it

Sostituiamo nella funzione

ed eseguiamo i prodotti ricordandoci che tutti i termini di grado superiore a finiscono nell'o piccolo

Il polinomio di Taylor associato ad con centro è

Nota: non abbiamo scritto l'o piccolo perché un polinomio non ha o piccoli.

Calcoliamo a questo punto lo sviluppo di Taylor centrato in e di ordine associato alla funzione integrale :è
sufficiente integrare rispetto ad , termine a termine, lo sviluppo di Taylor associato alla derivata prima:

Il polinomio di Taylor richiesto dalla traccia e associato alla funzione integrale è


Pij88u9k

e finalmente possiamo considerare l'esercizio completato.

Esercizio 19. Calcolare lo sviluppo di Taylor fino all'ordine , della funzione

Trovare poi le costanti tali che

Svolgimento: cominciamo con il notare che è continua e derivabile infinite volte perché composizione di funzioni derivabili.
Per determinare lo sviluppo di Taylor centrato in , abbiamo bisogno delle derivate della funzione:

Possiamo procedere con il calcolo esplicito delle derivate utilizzando il teorema fondamentale del calcolo integrale in combo con la
regola di derivazione per le funzioni composte, ma richiede una mole non indifferente di conti. Facciamoci furbi e procediamo in
modo differente:

Poniamo

Lo sviluppo della funzione seno è notevole ed infatti sappiamo che


221
eBook edito da YouMath.it acquistato da Navdeep Singh
YouMath.it

Determiniamo lo sviluppo della funzione mediante la definizione stessa di sviluppo di Taylor al terzo ordine:

Abbiamo bisogno quindi della funzione, della derivata seconda e della derivata terza di valutate ne punto .

Partiamo da che può essere determinato utilizzando la ben nota proprietà degli integrali definiti secondo cui se gli estremi di
un integrale definito coincidono allora esso vale zero. Nel nostro caso abbiamo:

Calcoliamo le derivate utilizzando il teorema fondamentale del calcolo integrale in combinazione con il teorema di derivazione delle
funzioni composte:

Per calcolare questa derivata, dobbiamo utilizzare la formula di derivazione

Abbiamo a disposizione della derivata prima, pertanto possiamo calcolare il valore .


Pij88u9k
Calcoliamo la derivata seconda con le solite regole di derivazione

e infine la derivata terza

Valutiamo a questo punto :

Abbiamo a disposizione gli ingredienti che consentono di calcolare lo sviluppo di Taylor al terzo ordine della funzione integrale

222
Navdeep Singh - navdeep99@outlook.it - 146.241.52.118 - 2020-01-04 22:31:14 r1t2e3m4u5k cikelpo
YouMath.it

Siamo riusciti a calcolare lo sviluppo della funzione integrale che possiamo comporre con lo sviluppo notevole del seno così da
soddisfare la richiesta dell'esercizio

Abbiamo ottenuto lo sviluppo fino al terzo ordine della funzione :

Per soddisfare la seconda richiesta dobbiamo sostituire al posto di il suo sviluppo nel limite

Pij88u9k

Scriviamolo meglio raccogliendo parzialmente rispetto alle potenze di :

Affinché questo limite faccia esattamente dobbiamo richiedere che scompaiano le potenze aventi esponenti minori di : per fare in
modo che ciò accada le costanti devono soddisfare le seguenti condizioni

(in questo modo eliminiamo la potenza )

(in questo modo annulliamo la potenza )

Per questi valori il limite diventa

ed è esattamente uguale ad se e solo se . In definitiva, affinché il limite sia dobbiamo pretendere


che:

L'esercizio è finalmente concluso.

Esercizio 20. Calcolare i seguenti limiti di funzioni integrali


223
eBook edito da YouMath.it acquistato da Navdeep Singh
YouMath.it

Svolgimento: consideriamo il primo limite

per calcolarlo la prima considerazione da fare è che il limite e integrali sono operatori monotoni, quindi possiamo limitarci a trovare
un'opportuna funzione che minora l'integranda e tale per cui sia semplice calcolare l'integrale. Ad esempio

Per la monotonia del limite e integrale abbiamo quindi che

In questo modo ci siamo ricondotti ad un integrale semplice fa calcolare

Pij88u9k

e quindi si conclude che il limite è infinito.

Nel secondo caso, basta sfruttare la simmetria della funzione integranda, che è una funzione pari infatti detta la funzione
integranda si ha che:

e ricordare che l'integrale ha significato geometrico di area sottesa dal grafico della funzione e riscrivere il limite come

dove

224
Navdeep Singh - navdeep99@outlook.it - 146.241.52.118 - 2020-01-04 22:31:14 r1t2e3m4u5k cikelpo
YouMath.it
è finito perché l'integranda è continua nell'intervallo chiuso e limitato ed è pertanto Riemann integrabile, mentre

Vediamo come raggiungere questo risultato nel dettaglio. Poiché possiamo supporre senza perdita di generalità che
(intuitivamente, la deve andare a , e nel suo tragitto prima o poi dovrà essere minore di ).

Consideriamo l'intervallo , per ogni si ha che . In tale intervallo la funzione


potenza quarta è decrescente e applicandola membro a membro siamo costretti a cambiare il verso delle disuguaglianze

Sommiamo membro a membro :

e infine estraiamo la radice quadrata membro a membro che essendo una funzione crescente nel dominio non inverte le
disuguaglianze

Ribadiamo che l'operatore integrale è monotono (isotonia dell'integrale) e applicandolo membro a membro otteniamo

Risolviamo il primo integrale, quello in cui l'integranda è una funzione costante, ossia

Pij88u9k

e consideriamo il limite per

Grazie al teorema del confronto per gli integrali possiamo concludere che

e di conseguenza

Ora l'esercizio è completo.

Esercizio 21. Calcolare al variare del parametro reale il seguente limite

Svolgimento: per risolvere il limite utilizzeremo gli sviluppi notevoli del coseno e del logaritmo, ma prima osserviamo che
o scritto in altri termini di conseguenza, quando anche per il teorema dei carabinieri, pertanto possiamo
utilizzare lo sviluppo notevole del coseno giacché il suo argomento è infinitesimo.

Sappiamo che

225
eBook edito da YouMath.it acquistato da Navdeep Singh
YouMath.it

pertanto

conseguentemente, integrando membro a membro tra otteniamo l'identità

Possiamo asserire quindi che

Grazie allo sviluppo notevole del logaritmo potremo scrivere invece che

Pij88u9k

In definitiva

ossia il limite esiste finito se e solo se ed in particolare è se mentre è uguale a se e solo

se ossia quando . L'esercizio è completo!

Esercizio 22. Risolvere il seguente limite, dimostrando che effettivamente è presente una forma di indecisione

226
Navdeep Singh - navdeep99@outlook.it - 146.241.52.118 - 2020-01-04 22:31:14 r1t2e3m4u5k cikelpo
YouMath.it

Svolgimento: il primo passo consiste nel dimostrare che siamo di fronte ad una forma di indecisione. Naturalmente per
, inoltre la funzione integranda è continua nell'intervallo chiuso e limitato , pertanto

Abbiamo ottenuto come risultato perché gli estremi di integrazione sono uguali e dunque siamo di fronte ad una forma di

indecisione .

Accertato questo possiamo applicare il teorema di De l'Hopital: abbiamo bisogno della derivata della funzione integrale e della
derivata della potenza .

Cominciamo con la derivata della funzione integrale ossia:

che possiamo calcolare mediante il teorema fondamentale del calcolo integrale in composizione con il teorema di derivazione delle
funzioni composte

La derivata di è immediata

Pij88u9k
dunque il limite si riscrive come

semplificando con

L'esercizio è concluso.

Esercizio 23. Calcolare il seguente limite di funzione integrale

dove è la parte intera di .

Svolgimento: iniziamo con l'analisi della funzione integranda:

227
eBook edito da YouMath.it acquistato da Navdeep Singh
YouMath.it
ed in particolare osserviamo bene l'argomento della funzione logaritmo. Una delle proprietà fondamentali della funzione parte intera
che ci permetterà di portare a casa l'esercizio è:

e sommando membro a membro

Passiamo ai reciproci membro a membro ricordandoci di invertire le disuguaglianze:

Ora attenzione: passando al limite membro a membro scopriamo che è infinitesimo, infatti sussiste la seguente relazione

e il teorema dei carabinieri ci assicura che il limite centrate è zero perché i limiti esterni sono anch'essi nulli.

Mediante la stima asintotica del logaritmo

possiamo scrivere quanto segue

Pij88u9k
Approfondiamo un momento l'ultima stima asintotica, partendo dalla relazione trovata in precedenza, cioè:

dividendo membro a membro per per , il verso delle disuguaglianze si mantiene

Passando al limite i tre membri abbiamo

Il primo limite è certamente , così come lo è il terzo, di conseguenza anche il limite centrale è per via del teorema dei carabinieri.
Questi passaggi dimostrano la relazione asintotica , naturalmente dobbiamo utilizzare questa stima, altrimenti è stata tutta fatica
sprecata.

Grazie al criterio del confronto asintotico per gli integrali impropri e in forza alla stima asintotica trovata possiamo asserire che
l'integrale ha lo stesso comportamento dell'integrale

che è un integrale improprio notevole divergente. Possiamo concludere quindi che il limite di partenza diverge positivamente, ossia

228
Navdeep Singh - navdeep99@outlook.it - 146.241.52.118 - 2020-01-04 22:31:14 r1t2e3m4u5k cikelpo
YouMath.it

Abbiamo finito.

Esercizio 24. Determinare una funzione ed i valori del parametro per cui risulti

Svolgimento: l'esercizio richiede un po' di intuito ed infatti è come se ci stesse chiedendo di comprendere qual è la forma
dell'integranda conoscendo l'integrale. Come facciamo? Possiamo procedere in questo modo.

Al secondo membro dell'equazione

abbiamo un polinomio che è notoriamente una funzione derivabile e affinché vi sia uguaglianza, anche al primo membro dobbiamo
avere una funzione derivabile. Questa semplice osservazione ci permette di derivare membro a membro rispetto ad . La derivata
della funzione integrale al primo membro si calcola mediante il teorema fondamentale del calcolo integrale in combinazione con il
teorema di derivazione delle funzioni composte

Deriviamo il secondo membro

Pij88u9k

Imponiamo l'uguaglianza tra le due derivate

e poniamo da cui si evince che . Sostituiamo nell'uguaglianza appena scritta

da cui, dividendo membro a membro per due otteniamo l'espressione di , ossia della funzione integranda

Benissimo, ora siamo a conoscenza della funzione e possiamo calcolare esplicitamente l'integrale che definisce la funzione
integrale

Il nostro obiettivo è determinare le costanti tali che risulti

229
eBook edito da YouMath.it acquistato da Navdeep Singh
YouMath.it

e per calcolarle applichiamo il principio di identità dei polinomi che ci permetterà di trovare la condizione che le definisce:

Risolviamo l'equazione di secondo grado e otteniamo che . In definitiva la funzione integranda che soddisfa

l'equazione è mentre le costanti sono e l'esercizio è terminato.

Esercizio 25. Data la funzione integrale

dimostrare che per ogni vale

Svolgimento: per portare a casa l'esercizio è necessario utilizzare il teorema di Lagrange, dobbiamo però accertarci che tutte le sue
ipotesi siano soddisfatte. Fissiamo con e consideriamo l'intervallo .

Mostreremo che è continua nell'intervallo chiuso e limitato ed è anche derivabile nell'intervallo aperto , cominciando
dalla continuità.
Pij88u9k
La funzione integrale è continua nell'intervallo La risposta è ovviamente sì, perché per essa vale il teorema di continuità

delle funzioni integrali. Osserviamo infatti che l'integranda è continua in tutto l'asse reale e risulta di conseguenza

integrabile nell'intervallo .

La funzione è derivabile nell'intervallo Possiamo rispondere in scioltezza a questa domanda mediante il teorema
fondamentale del calcolo integrale.

Poiché le pretese del teorema di Lagrange sono soddisfatte allora esiste un punto tale che

Applichiamo membro a membro il valore assoluto così da ottenere:

Ricordiamo che il valore assoluto di un prodotto coincide con il prodotto dei valori assoluti dei singoli fattori. Tale proprietà ci
permette di scrivere quanto segue:

Se riusciamo a dimostrare che abbiamo praticamente finito.

A questo puntoo dobbiamo calcolare la derivata prima della funzione . Dal teorema fondamentale del calcolo integrale, in
combinazione con la regola di derivazione delle funzioni composte abbiamo:

e dunque

230
Navdeep Singh - navdeep99@outlook.it - 146.241.52.118 - 2020-01-04 22:31:14 r1t2e3m4u5k cikelpo
YouMath.it

Ora attenzione, sappiamo che la funzione seno è limitata da dunque

Aggiungiamo membro a membro

e passiamo ai reciproci, ricordandoci che questa procedura inverte il verso delle disuguaglianze in gioco

La disuguaglianza che ci interessa è la seconda, ossia

Osserviamo che stiamo giocando con quantità positive, e finora il valore assoluto non è servito.

Moltiplichiamo membro a membro per (qui il valore assoluto è necessario perché la funzione coseno è a segno variabile)

e ricordando che si ha che:


Pij88u9k

Questa relazione vale per ogni , varrà in particolare anche per ossia

Siamo felici perché abbiamo praticamente concluso:

ed ecco che abbiamo dimostrato quello che ci serviva

Finalmente l'esercizio può ritenersi concluso.

ESERCIZI SULLO STUDIO DELLA FUNZIONE INTEGRALE


Esercizio 1. Della seguente funzione integrale

231
eBook edito da YouMath.it acquistato da Navdeep Singh
YouMath.it
a) determinare il campo di esistenza, o dominio;

b) provare che è pari;

c) calcolare la derivata prima di ;

d) determinare massimi e/o minimi;

e) calcolare e studiare concavità e flessi.

Svolgimento: a) il dominio dell funzione integrale è , ma dobbiamo spiegare il perché. Consideriamo la funzione
integranda

essa è ovviamente continua in ed è inoltre prolungabile con continuità in , infatti

Attenzione, il limite è non perché siamo di fronte ad una forma di indecisione ma perché è prodotto di due infinitesimi.

Poiché l'integranda è prolungabile con continuità in allora è integrabile localmente in , indipendentemente dal valore che
assume .

b) Verifichiamo la parità o la disparità della funzione integrale prendendo in esame la seguente quantità:

Poniamo , grazie alla sostituzione gli estremi di integrazione diventano:


Pij88u9k

e dunque diventa

Osserviamo che la funzione integranda è dispari, infatti , e sostituendo in modo


opportuno all'interno dell'integrale avremo:

ossia la funzione integrale è pari nel dominio.

c ­ d) Calcoliamo la derivata prima di che possiamo determinare mediante il teorema fondamentale del calcolo integrale, il
quale ci assicura che la derivata di coincide con l'integranda (questo perché l'estremo variabile è la semplice )

Per il teorema sul limite della derivata si ha che

e dunque si candida come punto stazionario. Studiamo la natura di questo punto analizzando il segno della derivata prima
impostando la disequazione

232
Navdeep Singh - navdeep99@outlook.it - 146.241.52.118 - 2020-01-04 22:31:14 r1t2e3m4u5k cikelpo
YouMath.it
È chiaro che l'esponenziale non infierisce sul segno giacché è una funzione positiva, dunque il segno della derivata prima dipende
esclusivamente dal fattore

dunque la derivata prima di è:

­ positiva per ;

­ nulla per ;

­ negativa per .

Tali informazioni ci permettono di concludere :

­ è strettamente crescente per ;

­ ha un punto di minimo (assoluto) per ;

­ è strettamente decrescente per .

Il minimo assoluto vale

(è zero perché gli estremi di integrazione sono uguali.)

e dalla definizione stessa di minimo assoluto scopriamo che , si ha quindi che la funzione
integrale è sempre non negativa.

e) Calcoliamo la derivata seconda, in che modo? Semplicissimo, sarà sufficiente derivare la derivata prima utilizzando la regola di
derivazione del prodotto in combinazione con la regola di derivazione delle funzioni composte
Pij88u9k

che è sempre positiva sia nell'intervallo che in . In più, per la derivata seconda della funzione integrale
vale , infatti

Abbiamo visto che è un punto di minimo relativo e non può essere punto di flesso. In definitiva la funzione è convessa nel
dominio. Il grafico della funzione integrale è

233
eBook edito da YouMath.it acquistato da Navdeep Singh
YouMath.it

Lo studio di funzione è terminato.

Esercizio 2. Studiare la seguente funzione integrale

Svolgimento: il primo passo è quello di determinare il dominio della funzione integrale e per farlo dobbiamo considerare la funzione
integranda e determinare l'insieme in cui essa risulti continua.

Il suo dominio è ed è ivi continua, conseguentemente è integrabile su ogni intervallo


Pij88u9k
chiuso e limitato contenuto interamente in uno degli intervalli. Poiché si ha che è contenuto o
al più uguale al dominio della funzione integrale.

Verifichiamo se per l'integrale improprio

converge o meno e per studiarne l'andamento useremo il criterio del confronto asintotico per gli integrali impropri di seconda specie.

la funzione integranda si comporta asintoticamente come per , quindi l'integrale

converge se e solo se converge

Quest'ultimo è un integrale improprio notevole divergente, e questo ci permette di asserire che è un asintoto verticale per
la funzione. Il dominio della funzione integrale è quindi

Osserviamo ora che è certamente uno zero della funzione integrale infatti per una nota proprietà degli integrali si ha che

234
Navdeep Singh - navdeep99@outlook.it - 146.241.52.118 - 2020-01-04 22:31:14 r1t2e3m4u5k cikelpo
YouMath.it

Vediamo come si comporta la funzione all'estremo destro dell'intervallo, cioè per

Se convergerà avremo un asintoto orizzontale destro, ma purtroppo non potremo esplicitare l'equazione dell'asintoto a meno che
non venga richiesto di risolvere l'integrale. Nel caso non fosse finito, vi è la possibilità che ci sia un asintoto obliquo, ma per ora non
ci pensiamo.

Anche questa volta ci vengono in soccorso le stime asintotiche: per l'integranda soddisfa la seguente relazione

di conseguenza l'integrale improprio

converge se e solo se converge

che è un integrale notevole convergente. In tal caso, la funzione integrale presenta un asintoto orizzontale di equazione

dove

Pij88u9k
Passiamo allo studio della derivata prima della funzione integrale, ma prima dobbiamo calcolarla applicando il teorema
fondamentale del calcolo integrale

Studiamo gli zeri della derivata prima che si candidano come punti di massimo o di minimo relativi

da cui .

Per determinare la natura studieremo il segno della derivata prima

analizzando il segno di ciascun fattore presente nella frazione

Costruiamo la tabella dei segni

235
eBook edito da YouMath.it acquistato da Navdeep Singh
YouMath.it

Grazie allo studio del segno abbiamo scoperto che la derivata prima è

­ negativa per ;

­ nulla per ;

­ positiva per .

Il valore è un punto di minimo assoluto per la funzione integrale, ma non possiamo conoscere esplicitamente il massimo,

implicherebbe la risoluzione dell'integrale.

Osserviamo che il calcolo della derivata seconda può essere effettuato, ma non otterremo molte informazioni a causa
dell'espressione stessa di , non è possibile determinare in modo esatto gli zeri né il segno della derivata stessa.

Sta di fatto che l'analisi della derivata prima è sufficiente per disegnare un grafico qualitativo della funzione integrale a meno della
concavità: possiamo quindi considerare l'esercizio concluso.
Pij88u9k
Esercizio 3. Tracciare il grafico qualitativo della funzione

Svolgimento: consideriamo la funzione integranda ed osserivamo che essa è continua su tutto l'asse reale, e da

ciò possiamo trarre le seguenti informazioni:

1. la funzione integrale è continua in ed è derivabile in , lo possiamo asserire con certezza per il teorema
fondamentale del calcolo integrale.

2. poiché è continua in il teorema di Weierstrass assicura che ammette massimo e minimo assoluti in ,
ossia esitono tali che

Dopo le dovute premesse possiamo continuare con l'analisi, partendo dalle informazioni più elementari.

Se allora si ha che , infatti

questo perché gli estremi di integrazione sono finiti e coincidono. Osserviamo inoltre che poiché la funzione integranda è positiva
perché quoziente di quantità positive allora possiamo determinare il segno della funzione integrale, in dettaglio:

è positiva per e ciò deriva dalla monotonia dell'operatore integrale

236
Navdeep Singh - navdeep99@outlook.it - 146.241.52.118 - 2020-01-04 22:31:14 r1t2e3m4u5k cikelpo
YouMath.it

è negativa per , questo deriva dal fatto che il secondo estremo di integrazione è minore del primo, in tal caso si ha

(interviene la proprietà degli integrali )

e l'integrale a secondo membro è positivo, ma il segno negativo cambia le carte in tavola, rendendo il secondo membro negativo.

Dedichiamoci ora al calcolo della derivata prima che avviene mediante il teorema fondamentale del calcolo integrale le cui ipotesi
sono ovviamente soddisfatte

La derivata prima è sempre positiva perché quoziente di quantità positive, conseguentemente la funzione integrale è strettamente
crescente. Non annullandosi all'interno dell'intervallo considerato, possiamo asserire con tranquillità che non vi sono punto
estremali interni a . Essi si trovano necessariamente agli estremi dell'intervallo.

Grazie alla monotonia della funzione integrale, possiamo concludere che il punto di minimo è mentre il punto di
massimo è .

Determinare analiticamente il massimo e il minimo assoluto è impossibile, perché richiede la risoluzione degli integrali

Pij88u9k

ma purtroppo non è possibile esplicitare il calcolo perché non riusciamo a determinare una primitiva dell'integranda che sia
esprimibile mediante composizione di funzioni elementari.

Calcoliamo la derivata seconda, derivando con la regola di derivazione del quoziente

La derivata seconda della funzione integrale è sempre positiva quindi la funzione è convessa nell'intervallo . Abbiamo
abbastanza informazioni per costruire il grafico della funzione e l'esercizio è concluso.

Esercizio 4. Studiare la funzione integrale

dove è la seguente funzione definita a tratti

Svolgimento: studiamo preliminarmente la funzione integranda ossia

237
eBook edito da YouMath.it acquistato da Navdeep Singh
YouMath.it

Essa è certamente una funzione continua in perché:

­ se è continua perché prodotto di funzioni continue;

­ se che è notoriamente una funzione continua.

L'unico punto in cui abbiamo dubbi è il punto di raccordo, ossia il punto in cui avviene il cambio di espressione analitica, nel nostro
caso . Per mostrare la continuità in dobbiamo verificare che il limite destro e il limite sinistro della funzione integranda
coincidono con il valore che essa assume in :

Nel nostro caso , e il limite destro e il limite sinistro valgono rispettivamente

poiché il limite destro e il limite sinistro sono finiti e coincidono con il valore che la funzione integranda assume in possiamo
asserire, senza ombra di dubbio, che la funzione integranda è continua in tutto l'asse reale, conseguentemente, la funzione
integrale è continua e derivabile nel dominio , e tutto questo grazie al teorema fondamentale del calcolo
integrale.

Studiamo i limiti agli estremi del dominio di , partendo da

Pij88u9k

Poiché il limite è , certamente la funzione non ha asintoto orizzontale, ma può esserci asintoto obliquo. Procediamo nel modo
classico, verificando se esiste finito e diverso da zero il limite

che è ovviamente una forma di indecisione che può essere studiata mediante il teorema di de l'Hopital

Il limite che definisce il coefficiente angolare dell'asintoto obliquo destro non è finito, dunque la funzione non presenta l'asintoto
obliquo destro. Controlliamo se ha un asintoto obliquo sinistro.

Consideriamo il limite

che è ancora una volta una forma di indecisione che possiamo studiare mediante il teorema di De l'Hopital

Questa volta il limite è finito, può esserci asintoto obliquo sinistro. Dobbiamo controllare il limite
238
Navdeep Singh - navdeep99@outlook.it - 146.241.52.118 - 2020-01-04 22:31:14 r1t2e3m4u5k cikelpo
YouMath.it

esista finito. Osserviamo che possiamo vedere come e grazie alle proprietà degli integrali possiamo scrivere

quanto segue

Consideriamo il primo integrale

esso converge perché la funzione integranda è prolungabile con continuità in , dunque assume un valore finito.

Il secondo integrale ossia

Pij88u9k

può essere risolto mediante la regola di integrazione per parti

A questo punto possiam calcolare esplicitamente il limite

Poiché questo limite non è finito allora non sarà finito nemmeno il limite che definisce il termine noto dell'asintoto obliquo. Possiam
concludere che la funzione non ha asintoti obliqui.

Studiamo la monotonia della funzione integrale e per farlo studieremo il segno della derivata prima della funzione

239
eBook edito da YouMath.it acquistato da Navdeep Singh
YouMath.it
Grazie al teorema fondamentale del calcolo integrale si ha che

Il segno della derivata prima è dato dall'unione dei seguenti sistemi di disequazioni

Il secondo sistema non ammette soluzioni, mentre il primo sistema ha per soluzione . Possiamo asserire che è:

­ positiva per ;

­ nulla per ;

­ negativa per .

La funzione integrale, :

­ è strettamente crescente nell'intervallo ;

­ è strettamente decrescente nell'intervallo e nell'intervallo ;

­ ha un punto di minimo assoluto per ;

­ ha un punto di flesso a tangente orizzontale per .

Osserviamo che il minimo assoluto coincide con ossia con il seguente valore
Pij88u9k

ed è zero perché gli estremi di integrazione sono uguali. Dalla definizione stessa di minimo assoluto si ha inoltre che:

e dunque è non negativa (sta sopra l'asse delle ascisse).

Per studiare la concavità e la convessità della funzione integrale abbiamo bisogno della sua derivata seconda che possiamo
calcolare, derivando :

Attenzione perché la derivata seconda non è definita per , è sufficiente infatti osservare che il limite destro e il limite sinistro
della derivata seconda esistono finiti ma non coincidono

Occupiamoci del segno della derivata seconda

Occupiamoci del primo sistema ed in particolare della prima disequazione

240
Navdeep Singh - navdeep99@outlook.it - 146.241.52.118 - 2020-01-04 22:31:14 r1t2e3m4u5k cikelpo
YouMath.it

e intersecando tale condizione con otteniamo come soluzione .

Il secondo sistema è facilissimo da risolvere, la prima disequazione è sempre soddisfatta, infatti

e intersecando questa condizione con otteniamo che l'insieme soluzione del secondo sistema è dettato dalla condizione
.

In definitiva possiamo asserire che la derivata seconda di è

­ positiva per e per ;

­ negativa per ;

­ nulla per .

La funzione , di conseguenza:

­ è convessa nell'intervallo e nell'intervallo ;

­ è concava nell'intervallo ;

­ ha un punto di flesso per .

Con queste informazioni possiamo disegnare il grafico della funzione integrale.


Pij88u9k

L'esercizio è finalmente concluso.

Esercizio 5. Data la funzione

1. determinare il campo di esistenza;

2. studiare il segno;

3. calcolare ;

4. determinare eventuali punti di massimo e minimo relativi;

241
eBook edito da YouMath.it acquistato da Navdeep Singh
YouMath.it

5. dire se è limitata.

Svolgimento:

1. Cominciamo con il determinare il dominio della funzione integrale, o detto in altri termini dobbiamo determinare l'intervallo che

contiene l'estremo finito in modo che l'integrale che definisce la funzione converge. Come si procede? Per prima cosa

determiniamo il dominio della funzione integranda

Non è difficile mostrare che . Poiché si ha certamente che .

Osserviamo infatti che se l'estremo variabile appartiene all'insieme allora l'intervallo di integrazione che definisce la
funzione integrale si presenta in una delle due forme

In ogni caso, la funzione integranda è continua nell'intervallo chiuso e limitato, di conseguenza essa è integrabile.
Pij88u9k
Dobbiamo verificare se controllando se l'integrale

converge oppure no. Studiamo la convergenza utilizzando il criterio del confronto asintotico per gli integrali impropri di seconda
specie. Per si ha che

dunque l'integrale converge se e solo se converge

ma questo è un integrale improprio notevole convergente. Possiamo asserire quindi che .

Controlliamo ora cosa succede per , analizzando il comportamento dell'integrale improprio

mediante il criterio del confronto asintotico. Poiché per si comporta asintoticamente come , ma questa volta

242
Navdeep Singh - navdeep99@outlook.it - 146.241.52.118 - 2020-01-04 22:31:14 r1t2e3m4u5k cikelpo
YouMath.it

diverge, da ciò abbiamo tre conseguenze:la prima è che , la seconda è che la funzione integrale non può essere
definita per , la terza ma non meno importante è che rappresenta un asintoto verticale per . In definitiva, il
dominio di è:

2. Studiare l'integranda è abbastanza semplice se si osserva che l'integranda è sempre negativa nell'intervallo pertanto
la funzione integrale è strettamente decrescente e tenendo presente che

e per definizione di funzione strettamente decrescente si ha che se allora e dunque la

funzione integrale è positiva nell'intervallo .

Se allora ossia la funzione integrale è negativa nell'intervallo .

3. Calcoliamo la derivata prima di mediante il teorema fondamentale del calcolo integrale le cui ipotesi sono soddisfatte

4. Per determinare i punti stazionari imponiamo che la derivata prima sia nulla, ossia impostiamo l'equazione

Pij88u9k

L'equazione non ammette soluzioni, quindi possiamo concludere che non vi sono punti stazionari interni al dominio. Attenzione però
che è punto di massimo assoluto infatti è strettamente decrescente in .

5. Abbiamo visto nel punto 1. che la funzione integrale presenta un asintoto verticale di equazione , di conseguenza
non è limitata (inferiormente).

L'esercizio è finalmente concluso.

Esercizio 6. Studiare la seguente funzione integrale

individuando il dominio e gli intervalli di concavità e convessità. Trovare inoltre gli eventuali punti di flesso.

Svolgimento: vediamo come studiare la funzione integrale

Il teorema fondamentale del calcolo integrale ci permetterà di ricondurre lo studio della derivata prima e seconda ad un
normalissimo studio di funzione standard: la parte più interessante riguarda le considerazioni che precedono lo studio della derivata
prima.

Iniziamo dal dominio. La funzione integranda è definita solamente per i valori , dunque tale è l'insieme di
definizione della funzione integrale, dunque

243
eBook edito da YouMath.it acquistato da Navdeep Singh
YouMath.it

perché comunque si fissi , negli intervalli del tipo

l'integranda è continua, e questa rappresenta una condizione sufficiente per l'integrabilità.

Occupiamoci del segno e dell'intersezione con gli assi. La funzione integranda è positiva su tutto il proprio insieme di definizione,
quindi se avremo certamente che . Per una nota proprietà dell'integrale di Riemann, è facile vedere che

ed infinie, per un'ulteriore proprietà, sappiamo che

Se risulta che è una quantità positiva, dunque è negativa (per via del segno negativo).

L'unica intersezione con l'asse delle ascisse è data da .

L'intersezione con l'asse delle ordinate esiste, essendo definita in ed è data da

che è una quantità negativa e di cui purtroppo non possiamo conoscere esplicitamente il suo valore perché l'integrale non è
esprimibile
Pij88u9kin modo elementare.

Dedichiamoci ora allo studio dei limiti agli estremi del dominio. Il teorema fondamentale del calcolo integrale ci assicura che,
essendo continua su la funzione integrale non solo sarà ivi continua ma anche derivabile. L'unico limite da calcolare
è

dove è facile vedere che è un integrale improprio di prima specie divergente. Basta ad esempio, minorare

l'integranda con e procedere con il criterio del confronto.

Questa era la parte difficile dello studio di funzione integrale, d'ora in poi ci occuperemo della derivata prima e, in un secondo
momento, della derivata seconda.

Applicando il teorema fondamentale del calcolo integrale si ha facilmente che la derivata della funzione integrale coincide con
l'integranda ossia

Studiamo gli zeri e il segno della derivata prima imponendo la disequazione

che è ovviamente soddisfatta per ogni valore del dominio, possiamo concludere quindi che la funzione integrale è crescente nel
dominio, inoltre non si annulla mai nei punti interni all'intervallo quindi non vi sono punti stazionari. A causa della monotonia
di , la funzione presenta un punto di minimo assoluto per , il minimo associato vale

244
Navdeep Singh - navdeep99@outlook.it - 146.241.52.118 - 2020-01-04 22:31:14 r1t2e3m4u5k cikelpo
YouMath.it

di cui però non è possibile determinarne il valore numerico, perché implicherebbe la ricerca di una primitiva per che però non
si esprime mediante composizione di funzioni elementari.

Dedichiamoci ora alla derivata seconda, agli intervalli di concavità e convessità e gli eventuali punti di flesso.

A differenza di ciò che avviene per quello della derivata prima, il calcolo della derivata seconda può essere effettuato utilizzando le
usuali regole di derivazione su : procediamo in particolare con la regola di derivazione del prodotto

Studiamo gli eventuali zeri e il segno della derivata seconda, così da comprendere quali sono gli intervalli di concavità e quelli di
convessità e i possibili punti di flesso. Imponiamo quindi che la derivata seconda sia maggiore o uguale a zero

La disequazione è sempre soddisfatta infatti al primo membri i termini che compongono la funzione sono tutti positivi per .
Pij88u9k
Possiamo asserire senza problemi che la derivata seconda della funzione integrale è positiva per conseguentemente
è una funzione convessa nel dominio.

Abbiamo finito, abbiamo tutte le informazioni che ci servono per disegnare il grafico qualitativo della funzione integrale.

Esercizio 7. Data la funzione integrale

a) trovare l'insieme di definizione della funzione integrale e calcolare i limiti agli estremi del dominio;

b) calcolare la derivata , dove esiste, e studiare la monotonia della funzione integrale .

Svolgimento: per determinare il dominio della funzione integrale proposta, che indichiamo sinteticamente con

dobbiamo studiare l'esistenza dell'integrale improprio nell'intorno dei punti in cui l'integranda smette di esistere, più
precisamente dovremo considerare i punti singolari di .

Chiaramente la funzione integranda ha per dominio .

Cominciamo con lo studio dell'integrabilità in senso improprio della funzione integranda nell'intorno del punto , facendo
riferimento alla teoria degli integrali impropri.

245
eBook edito da YouMath.it acquistato da Navdeep Singh
YouMath.it

Dato che si vede che l'integrale improprio associato non esiste finito. In particolare possiamo asserire che

Passiamo a considerare il comportamento in ;

Per sussiste la seguente equivalenza asintotica

e poiché l'integrale

converge allora la funzione integranda è integrabile in senso improprio in un intorno di zero, e ciò si traduce in .

Consideriamo l'integrabilità in : in questo caso l'integrale improprio

converge in senso improprio perché la funzione integranda è continua e limitata in un intorno di


Pij88u9k . Osserviamo infatti che

poiché allora perché stiamo lavorando con numeri negativi, di conseguenza il limite si riscrive come

Poiché tale limite esiste finito allora la funzione integranda può essere prolungata con continuità anche nell'intorno destro di ,e
quindi è localmente integrabile in tale intorno.

Proprio perché l'integranda è integrabile localmente in allora possiamo asserire che il dominio della funzione integrale è

Per studiare la monotonia di , bisogna applicare il teorema fondamentale del calcolo integrale il quale fornisce i mezzi pratici
per calcolare la derivata prima

Studiamone il segno tenendo sempre a mente il dominio di : impostiamo la disequazione

246
Navdeep Singh - navdeep99@outlook.it - 146.241.52.118 - 2020-01-04 22:31:14 r1t2e3m4u5k cikelpo
YouMath.it
e studiamola, ragionando sul segno dei singoli fattori

Costruiamo la tabella dei segni così da ottenere che la derivata prima è

­ positiva se ;

­ nulla se ;

­ negativa se

di conseguenza possiamo asserire che la funzione integrale:

­ è crescente per , per e per ;

­ è decrescente per ;

­ ha un punto di massimo assoluto per , il massimo vale

Naturlamente il risultato è zero perché gli estremi di integrazione sono coincidenti. Osserviamo che poiché è il massimo assoluto
per la funzione allora per ogni valore del dominio: ciò assicura che la funzione è negativa o nulla nel dominio.

Pij88u9k
L'esercizio è terminato.

Esercizio 8. Della funzione integrale

1. determinare il campo d'esistenza;

2. calcolare ;

3. determinare eventuale massimi e minimi relativi;

4. dire se è limitata calcolando gli eventuali asintoti.

Svolgimento:

1. Cominciamo con il calcolo del dominio della funzione integrale che, detto alla buona, ha per elementi i valori di per i quali

l'integranda risulta integrabile sull'intervallo (oppure , a seconda che oppure no).

Per prima cosa determiniamo il dominio della funzione integranda tenendo presente che vi è un logaritmo che

pretende che il suo argomento sia maggiore di zero, inoltre il logaritmo è al denominatore quindi dobbiamo richiedere che esso sia
diverso da zero. Queste due condizioni permettono di concludere che il dominio della funzione integranda è

Osserviamo inoltre che essa è continua nel dominio.

247
eBook edito da YouMath.it acquistato da Navdeep Singh
YouMath.it

Poiché l'estremo finito di integrazione allora si ha che comunque si fissi allora è integrabile in

così come è integrabile in , conseguentemente possiamo asserire che , detto in

altri termini, dall'analisi effettuata possiamo asserire che l'intervallo è contenuto nel domino della funzione integrale.

Controlliamo l'integrabilità locale dell'integranda in . Se l'integrale

converge allora . Osserviamo che per si ha che pertanto l'integranda è prolungabile con

continuità in ponendo , così facendo abbiamo la certezza che l'integrale converge e dunque .

Controlliamo l'integrabilità locale di sull'intervallo così da verificare se appartiene o no al dominio della funzione

integrale.

In soldoni dobbiamo studiare la convergenza dell'integrale

Possiamo analizzarlo mediante il criterio del confronto asintotico osservando che per si ha che

e dunque l'integranda gode la relazione asintotica


Pij88u9k

Poiché l'integrale non converge allora non converge nemmeno e dunque non appartiene al

dominio dell'integrale. Possiamo asserire inoltre che è l'equazione dell'asintoto verticale.

Grazie a queste informazioni possiamo concludere che il dominio della funzione integrale è .

2. Calcoliamo la derivata prima di mediante il teorema fondamentale del calcolo integrale, è grazie ad esso infatti che
possiamo scrivere

3. Per determinare gli eventuali punti stazionari, imponiamo che la derivata prima sia uguale a zero

L'equazione non ha soluzioni, dunque non vi sono punti stazionari interni al dominio. Studiamo in ogni caso il segno della derivata
prima, grazie al quale potremo determinare gli intervalli di monotonia.

Il segno della derivata prima dipende esclusivamente da quello del denominatore, giacché il numeratore è positivo, dunque la
precedente disequazione è equivalente alla seguente

248
Navdeep Singh - navdeep99@outlook.it - 146.241.52.118 - 2020-01-04 22:31:14 r1t2e3m4u5k cikelpo
YouMath.it

Nell'intervallo la derivata prima è negativa e di conseguenza la funzione integrale è strettamente decrescente. Proprio per la
monotonia, possiamo dire che la funzione integrale ha massimo nel primo estremo del dominio ossia per . Il massimo è
quindi

ma non possiamo purtroppo conoscere esattamente il valore numerico perché non possiamo determinare una primitiva di

esprimibile come composizione di funzioni elementari.

4. è certamente non limitata perché come abbiamo visto nel punto 1. è presente un asintoto verticale di equazione .

Abbiamo svolto tutti i punti richiesti dalla traccia, possiamo considerare concluso l'esercizio.

Esercizio 9. Dire se la funzione

è limitata nel suo dominio, determinare inoltre gli eventuali punti di massimo o di minimo.

Svolgimento: il primo passo consiste nel determinare il dominio della funzione integrale. Naturalmente iniziamo con l'analisi della
funzione integranda

Pij88u9k

il cui dominio è dettato dalla condizione . Risolviamo la disequazione di secondo grado con la relativa tecnica
risolutiva così da ottenere

Possiamo quindi asserire che il dominio della funzione integranda è , ed inoltre l'integranda è
continua nel dominio perché composizione di funzioni continue.

Dall'analisi del dominio dell'integranda comprendiamo che è un punto singolare (di seconda specie) per , infatti

È chiaro che dobbiamo controllare la convergenza dell'integrale che definisce nell'intorno di , perché se non vi fosse
convergenza non sarebbe nemmeno una funzione. Possiamo studiare la locale integrabilità di mediante il criterio del
confronto asintotico, determinando una stima asintotica per quando :

Poiché è un integrale improprio notevole di seconda specie convergente allora la funzione integrale è ben posta.

Attenzione ora! Comunque si fissi si ha che è una funzione continua nell'intervallo conseguentemente è
definita nell'intervallo di variazione di e dunque esso è contenuto o al più uguale al dominio della funzione integrale:
.

249
eBook edito da YouMath.it acquistato da Navdeep Singh
YouMath.it

Aggiungiamoci pure che per si ha che , dunque anche il valore appartiene al

dominio.

Abbiamo gli strumenti per scrivere il dominio della funzione integrale

Osserviamo che poiché la funzione integranda non è definita nell'intervallo non è possibile che "espandare" ulteriormente il
dominio di .

Se fissiamo ad esempio , ad esempio , allora l'intervallo di integrazione sarebbe su cui però ha problemi di
esistenza.

Occupiamoci della limitatezza, analizzando il limite agli estremi del dominio di , in particolare quando . Impostiamo
il limite

che per definzione di integrale improprio coincide diventa:

Possiamo studiare la convergenza mediante il criterio del confronto asintotico per integrali impropri di prima specie, osservando che
sussiste la seguente stima asintotica per l'integranda

Pij88u9k
Grazie al criterio del confronto asintotico sappiamo che

ha lo stesso comportamento di

ma quest'ultimo è un integrale improprio notevole divergente (negativamente), dunque:

è divergente: possiamo asserire che non è limitata inferiormente.

Procediamo con il calcolo della derivata prima di che possiamo determinare mediante l'applicazione del teorema
fondamentale del calcolo integrale

È evidente che la derivata prima è sempre positiva per dunque è strettamente crescente nel dominio.

250
Navdeep Singh - navdeep99@outlook.it - 146.241.52.118 - 2020-01-04 22:31:14 r1t2e3m4u5k cikelpo
YouMath.it
A causa della monotonia della funzione inoltre, si ha che è un punto di massimo assoluto e il massimo vale
.

Per definizione di massimo assoluto si ha infine che

ossia la funzione integrale è negativa o al più nulla nel dominio. Abbiamo le informazioni che ci servono per disegnare un grafico
qualitativo a meno della concavità

L'esercizio è completo.

Esercizio 10. Dire per quali valori di la seguente funzione definita a tratti è continua
Pij88u9k

dopodiché studiare la funzione integrale definita da:

Svolgimento: cominciamo con lo studio della continuità della funzione

partendo con l'analisi delle condizioni .

La prima condizione, ossia , è equivalente a .

La seconda condizione, ossia è equivalente a .

In entrambi i casi è sufficiente risolvere la disequazione con valore assoluto per determinare la condizione equivalente. In definitiva
si riscrive come segue:

Naturalmente è continua sia per e per perché in entrambi i rami sono composizioni di funzioni
continue. I punti in cui abbiamo dubbi sono i punti di raccordo, ossia i punti in cui cambia la sua espressione analitica.

251
eBook edito da YouMath.it acquistato da Navdeep Singh
YouMath.it
Lo studio della continuità in tali punti avviene ovviamente mediante la definizione stessa di continuità: considereremo il limite destro
e il limite sinistro per che tende al punto di raccordo e per avere continuità dobbiamo pretendere che i due limiti siano finiti e
coincidono con il valore che la funzione assume nel punto di raccordo.

Cominciamo con e consideriamo il limite destro e sinistro di

Il limite destro e il limite sinistro per sono sì finiti ma non coincidono pertanto è un punto di discontinuità a salto
per .

Studiamo la continuità nel punto

Anche è un punto di discontinuità a salto.

Ora ci possiamo dedicare allo studio della funzione integrale

Pij88u9k
Poiché ha esclusivamente discontinuità a salto allora possiamo dire che la funzione integrale è definita su tutto l'asse
reale e dunque il suo dominio coincide con ed è ivi continua.

Per il teorema fondamentale del calcolo integrale, inoltre, la funzione è derivabile in tutti i punti in cui la funzione integranda è
continua: è certamente derivabile in , in e in .

non è derivabile in né in questo perché la sua derivata coincide con la funzione integranda:

e come abbiamo visto presenta discontinuità a salto in e in , conseguentemente ha due punti angolosi,
in e in .

Osserviamo che si annulla per che si candida come punto di massimo o di minimo relativi. Classifichiamo tale punto
mediante lo studio del segno della derivata prima: impostiamo la disequazione e risolviamola su ciascun intervallo di
definizione dei due rami.

Se allora la disequazione diventa:

che ha per soluzione e tenendo conto della conduzione imposta avremo che:

­ se allora è negativa;

­ se allora è positiva.

Abbiamo già le prime informazioni sulla monotonia della funzione integrale che è:

252
Navdeep Singh - navdeep99@outlook.it - 146.241.52.118 - 2020-01-04 22:31:14 r1t2e3m4u5k cikelpo
YouMath.it
­ strettamente crescente per ;

­ strettamente decrescete per .

Se la disequazione diventa

che se risolta conduce a , pertanto risulta essere strettamente crescente sia in sia in .

Lo studio del segno della derivata prima ci dice che è un punto di flesso a tangente orizzontale.

Controlliamo se la funzione integrale presenta asintoti orizzontali, impostando il limite

Il primo integrale converge perché abbiamo una funzione continua su un intervallo chiuso e limitato (e dunque è Riemann -
integrabile). Il secondo integrale converge perché è un integrale notevole del tipo

Il limite esiste finito perché somma di integrali convergent, quindi ammette un asintoto orizzontale di cui è

possibile calcolare esplicitamente l'equazione, è sufficiente risolvere gli integrali così da ottenere
Pij88u9k .

Procediamo allo stesso modo per , anche in questo caso avremo un asintoto orizzontale la cui equazione è

Nota bene: non sempre è possibile determinare esplicitamente l'equazione dell'asintoto perché magari non è possibile risolvere
esplicitamente l'integrale improprio.

Osserviamo ora che e grazie alla monotonia possiamo dire senza dubbi che la funzione integrale è positiva per
mentre è negativa per .

Occupiamoci della derivata seconda grazie alla quale determineremo gli intervalli di concavità e convessità. Il calcolo della derivata
seconda si ottiene ovviamente derivando

Attenzione, nell'espressione della derivata seconda compaiono tre rami a causa del valore assoluto dell'arcotangente. Ad ogni
modo non è difficile mostrare che è:

­ positiva per ;

­ negativa per , per e per .

Grazie allo studio della derivata seconda scopriamo che è:

­ convessa per ;

­ concava per e per e per .

253
eBook edito da YouMath.it acquistato da Navdeep Singh
YouMath.it

Ecco il grafico della funzione

mentre il grafico della funzione integrale è:

Pij88u9k

Possiamo considerare concluso l'esercizio.

Esercizio 11. Studiare la seguente funzione integrale

Svolgimento: come ogni studio di funzione che si rispetti, si comincia sempre dal dominio della funzione. Osserviamo che la
funzione integranda

è definita a patto che il denominatore sia diverso da zero

pertanto . Naturalmente è continua nel dominio perché composizione di


funzioni continue.

Osserviamo che per ogni l'integrale che definisce la funzione integrale esiste finito sicuramente, perché comunque si
fissi l'intervallo di estremi è tale che la funzione integranda sia continua, di conseguenza il teorema
fondamentale del calcolo integrale ci permette di asserire che la funzione integrale è continua in e in
.

Osserviamo inoltre che la funzione integranda è pari, infatti:

254
Navdeep Singh - navdeep99@outlook.it - 146.241.52.118 - 2020-01-04 22:31:14 r1t2e3m4u5k cikelpo
YouMath.it

Possiamo sfruttare questa proprietà per studiare la funzione integrale, giacché è certamente dispari, limitando lo studio per
e poi sfruttare la simmetria centrale.

Studiamo l'integrabilità dell'integranda nei punti singolari, in particolare per . Interverranno i criteri di convergenza per gli
integrali impropri di seconda specie, ed in particolare il criterio del confronto asintotico.

Per valgono le seguenti stime asintotiche:

e dunque la funzione integranda gode della seguente stima asintotica

Poiché converge (è un integrale improprio notevole con esponente minore di ) allora convergerà anche:

Questo ci permette di concludere che la funzione integrale è definita per e per simmetria anche in . In definitiva, il
Pij88u9k
dominio di è .

Continuiamo con lo studio di funzione, determinando l'intersezione con l'asse . Chiaramente interseca l'asse delle ordinate
in , infatti

Ricordiamo che se gli estremi di integrazione coincidono allora l'integrale è zero per definizione.

Studiamo i limiti agli estremi dell'intervallo, considerando il limite

Abbiamo già studiato in precedenza il punto singolare (e il suo simmetrico ), dobbiamo invece controllare la
convergenza per . Dobbiamo controllare la convergenza con i criteri di convergenza dell'integrale improprio di prima
specie.

Per sussistono le seguenti stime

255
eBook edito da YouMath.it acquistato da Navdeep Singh
YouMath.it
Se converge l'integrale improprio

allora converge anche l'integrale . Bisogna usare un po' di ingegno per mostrare la convergenza: è facile notare che
e grazie a tale disuguaglianza possiamo scrivere quanto segue

L'ultimo è un integrale improprio notevole convergente, di conseguenza convergerà anche l'integrale in , dunque:

e in base a quanto scritto, è asintoto orizzontale destro. Per simmetria si ha che

(è una proprietà delle funzioni dispari.)

Calcoliamo la derivata prima della funzione integrale mediante il teorema fondamentale del calcolo integrale. Si ha che nei punti in
cui l'integranda è continua la derivata prima di è uguale all'integranda:

Pij88u9k

Studiamo il segno della derivata prima impostando la disequazione che diventa:

Studiamo separatamente il segno del numeratore e del denominatore, dopodiché costruiremo la tabella dei segni grazie alla quale
otterremo il segno di :

osserviamo infatti che l'esponenziale è una funzione positiva.

Creando la tabella dei segni scopriamo che la derivata prima è:

­ positiva per oppure per ;

­ negativa per .

La funzione integrale è:

­ strettamente crescente per e per ;

­ strettamente decrescente per .

I punti si candidano come punti di non derivabilità, che possiamo classificare mediante il calcolo dei limiti destri
e sinistri.

256
Navdeep Singh - navdeep99@outlook.it - 146.241.52.118 - 2020-01-04 22:31:14 r1t2e3m4u5k cikelpo
YouMath.it

Il limite destro e il limite sinistro della derivata prima esistono ma sono infiniti discordi dunque è un punto di cuspide e in
base al segno della derivata prima possiamo asserire anche che esso è un punti di minimo relativo.

Per la disparità della funzione integrale si ha che è un punto di cuspide ed in base al segno della derivata prima esso è un
punto di massimo relativo.

Possiamo dedicarci allo studio della derivata seconda, che si calcola con le usuali tecniche di derivazione

Studiamo ciascun fattore

Pij88u9k

In definitiva creando la tabella dei segni possiamo asserire che la derivata seconda di è:

­ positiva per

­ negativa per

­ nulla per

Lo studio del segno della derivata seconda ci permette di asserire che sono punti di flesso.

Le informazioni in nostro possesso ci permetto di disegnare qualitativamente il grafico della funzione

257
eBook edito da YouMath.it acquistato da Navdeep Singh
YouMath.it

L'esercizio è concluso.

Esercizio 12. Studiare la seguente funzione integrale tralasciando lo studio della derivata seconda

Svolgimento: partiamo dal dominio della funzione integrale, individuando i punti singolari della funzione integranda

Dobbiamo ragionare sui valori che annullano il denominatore, ossia

Pij88u9k

In entrambi i casi i valori si individuano risolvendo le rispettive equazioni esponenziali. A questo punto studiamo la convergenza
degli integrali impropri corrispondenti:

Concentriamoci sull'integranda e usiamo il criterio del confronto asintotico per gli integrali impropri di seconda specie

dove l'equivalenza asintotica deriva dal limite notevole dell'esponenziale. Il confronto asintotico con gli integrali impropri notevoli ci
dice quindi che l'integrale improprio converge.

Ne deduciamo che appartiene al dominio della funzione integrale e quindi è possibile procedere con le valutazioni di
oltre tale punto, fino a .

Per quanto riguarda l'altro valore

Per essere precisi conviene usare una nota proprietà degli integrali, dopo aver osservato che

Ora ragioniamo sull'integranda. Qui conviene lavorare algebricamente sul fattore del denominatore che genera la singolarità e
sfruttare la definizione di logaritmo

258
Navdeep Singh - navdeep99@outlook.it - 146.241.52.118 - 2020-01-04 22:31:14 r1t2e3m4u5k cikelpo
YouMath.it

effettuiamo un piccolo raccoglimento sfruttando le proprietà delle potenze

dove nell'ultimo passaggio abbiamo sfruttato l'equivalenza asintotica che discende dal limite notevole dell'esponenziale.

Per il confronto asintotico con gli integrali impropri notevoli concludiamo che l'integrale diverge. Da qui deduciamo che la funzione
integrale non esiste in e quindi nemmeno per i valori di ascissa inferiori. In sintesi:

Per capire come avviene la divergenza effettuiamo sin da subito lo studio del segno della funzione integrale mediante lo studio
dell'integranda:

Si tratta di una comunissima disequazione in cui dobbiamo studiare separatamente il segno dei singoli fattori:

Pij88u9k

Il termine in valore assoluto è ovunque positivo, meno che in , per definizione di modulo.

Dal confronto tra i segni si deduce che l'integranda è negativa per e positiva per .

Attenzione: la funzione integrale è certamente nulla per , per un'ovvia proprietà degli integrali definiti

Sull'intervallo l'integranda è negativa e gli estremi di integrazione sono ordinati in modo cresente, quindi la funzione
integrale è negativa su .

Sull'intervallo invece dobbiamo riordinare gli estremi

Essendo l'integranda negativa su tale intervallo, l'integrale con estremi ordinati è negativo, quindi la funzione integrale è positiva. In
definitiva

259
eBook edito da YouMath.it acquistato da Navdeep Singh
YouMath.it
È facile vedere che in la funzione integrale assume un valore negativo, basta fare riferimento al significato geometrico
dell'integrale.

Lo studio del segno ci consente di trarre un'importante considerazione riguardo ai limiti agli estremi del dominio:

­ in abbiamo un asintoto verticale in cui la funzione integrale diverge positivamente da destra;

­ per dobbiamo studiare l'integrale improprio di prima specie

Se studiamo il comportamento asintotico della funzione integranda

Qui si vede facilmente che l'integrale improprio associato a converge grazie al criterio del confronto per gli integrali impropri di

prima specie (confronto, non asintotico); basta procedere per confronto con gli integrali impropri notevoli, ed osservare che ad
esempio

quindi

e quindi
Pij88u9k

Poiché converge (è un integrale improprio notevole di prima specie), possiamo concludere che

ossia abbiamo un asintoto orizzonta destro di equazione .

Procediamo con lo studio della derivata prima. Dobbiamo appellarci al teorema fondamentale del calcolo integrale

A ben vedere abbiamo già effettuato lo studio del segno della derivata prima, basta ricontrollare lo studio dell'integranda e trarre le
dovute considerazioni. La derivata della funzione integrale è:

­ negativa per ;

­ positiva per .

Sui rispettivi integrali la funzione integrale è dunque decrescente e crescente. Ruguardo al punto , l'integranda non è ivi
definita, ma ciò non ci impedisce di studiare la derivabilità di calcolando il limite destro e il limite sinistro della derivata prima

260
Navdeep Singh - navdeep99@outlook.it - 146.241.52.118 - 2020-01-04 22:31:14 r1t2e3m4u5k cikelpo
YouMath.it

Osserviamo che è intervenuta la stima asintotica per la risoluzione del limite.

Calcoliamo il limite sinistro:

Poiché il limite destro e il limite sinistro sono infiniti discordi possiamo asserire che è un punto di non derivabilità per la
funzione integrale, ed in particolare un punto di cuspide. Lo studio è terminato.

Pij88u9k

261
eBook edito da YouMath.it acquistato da Navdeep Singh

Potrebbero piacerti anche